Test br. 1:
Zadatak 1. Na početku ulice nalazi se kuća. U toj kući na šestom spratu živi vozač A, a na sedmom vozači B, C i D, koji su rođena braća vozača A. Vozač A nema više braće.
U stanu vozača A su dvoja vrata i tri prozora.
U stanu vozača B ima onoliko prozora koliko ima vrata u stanu vozača C i onoliko vrata koliko ima prozora kod vozača C.
Stanovi u kojima žive braća vozača D imaju, sve u svemu, onoliko prozora koliko i vrata.
Pitanje: Živi li u toj kući tašta vozača A?
Zadatak 2. Majka je 21 godinu starija od kćerke. Za 6 godina majka će biti 5 puta starija od kćerke. Gdje je otac?
Zadatak 3. Pleme ljudoždera uhvati Ghoula. Njihov poglavica reče: "Napiši jednu izjavu. Ako ona bude istinita ispećemo te i pojesti, a ako bude lažna skuvaćemo te i pojesti." Može li se Ghoul spasiti iz te neugodne situacije?
Zadatak 4. Ako mačka i po za dan i po pojede miša i po, koliko miševa će pojesti 9 mačaka za 9 dana?
Zadatak 1 Ne
Zadatak 2 Otac je mrtav
Zadatak 3 Može
Zadatak 4 6
xrofl
Evo jednog jednostavnog :
Ako motorna testera pravi 10 obrtaja u sekundi a za jedno odrubljivanje glave je potrebno 5 sekundi, kolko ce Letherfejsova motorka napraviti obrtaja ako je body count 7 (osecenih glava naravno) ?
Quote from: akhnaton on 17-10-2009, 15:12:14
Zadatak 1 Ne
Zadatak 2 Otac je mrtav
Zadatak 3 Može
Zadatak 4 6
xrofl
Za prvi i treći zadatak odgovori su tačni, ali nisu objašnjeni. A odgovori na drugi i četvrti zadatak nisu ispravni.
Quote from: Son of Man on 17-10-2009, 15:13:55
Evo jednog jednostavnog :
Ako motorna testera pravi 10 obrtaja u sekundi a za jedno odrubljivanje glave je potrebno 5 sekundi, kolko ce Letherfejsova motorka napraviti obrtaja ako je body count 7 (osecenih glava naravno) ?
Sjajan zadatak! Možda ovo iskoristim za neki kontrolni u školi! :|
1. vozač A je (verovatno) žensko (nisam se udubljivao u rešavanje, ali kad je nelogično pitanje verovatno ima razloga).
2. ovde nešto ne valja. imamo sistem sa dve jednačine, M=C+21 i M+6=5*(C+6). Rešenje ovog sistema je C=-0.75, M=20.25. Ćerka još nije rođena, a ako pričamo o američkom viđenju seksa sa maloletnicom onda je otac u Kanu, prima nagradu za životno delo.
3. "Biću skuvan i pojeden"
4. Za dan i po mačka pojede miša, za devet dana 6 miševa, devet mačaka za devet dana 54 miša.
Hm, kad malo razmislim 0.75 godina je devet meseci. (Budući) otac je u krevetu s (budućom) mamom.
Svaka čast mac! Bez greške. Čista petica! xjap
3. Ghoul se verovatno ne bi izvukao, jer bi im rekao da je "ispećemo te" nepravilno, a "ispeći ćemo te" pravilno.
Quote from: angel011 on 17-10-2009, 18:48:20
3. Ghoul se verovatno ne bi izvukao, jer bi im rekao da je "ispećemo te" nepravilno, a "ispeći ćemo te" pravilno.
:oops:
Dok ne stigne novi test, evo nekih resenih zadataka:
(https://www.znaksagite.com/diskusije/proxy.php?request=http%3A%2F%2Flh3.ggpht.com%2F_dPeotyuHVO0%2FStn4FwMeAHI%2FAAAAAAAAAIA%2FGO9lY34O3bU%2Fsin.jpg&hash=95919305287c49ce15d4bae7306c76a7b918ffc8)
(https://www.znaksagite.com/diskusije/proxy.php?request=http%3A%2F%2Flh5.ggpht.com%2F_dPeotyuHVO0%2FStn4B7Ag61I%2FAAAAAAAAAH8%2FPOvKYXQhnd8%2Fraiz.jpg&hash=812544f03c437728355480968162337f4eaf6a73)
(https://www.znaksagite.com/diskusije/proxy.php?request=http%3A%2F%2Flh6.ggpht.com%2F_dPeotyuHVO0%2FStn38z0JtNI%2FAAAAAAAAAH4%2F0PZPoj8RvM0%2Finfinito.jpg&hash=761971fec70fd5f97659a27b53a026a158810c04)
(https://www.znaksagite.com/diskusije/proxy.php?request=http%3A%2F%2Flh6.ggpht.com%2F_dPeotyuHVO0%2FStn33rvGSlI%2FAAAAAAAAAHc%2FhU3z8fte-Lk%2Ffindx.jpg&hash=2cde66cd18144cfae6f9f124b08e866a890d907e)
(https://www.znaksagite.com/diskusije/proxy.php?request=http%3A%2F%2Flh4.ggpht.com%2F_dPeotyuHVO0%2FStn3f363soI%2FAAAAAAAAAHA%2FFs53sErqDXU%2Fexpanded.jpg&hash=8a4311526d9d162f5000b409fe55f59b92984639)
Želim samo da napomenem da ovaj primjer sa limesom kad iks teži osam nije ispravan. Pomenuti limes zapravo ne postoji.
Quote from: Harvester on 17-10-2009, 20:42:19
Želim samo da napomenem da ovaj primjer sa limesom kad iks teži osam nije ispravan. Pomenuti limes zapravo ne postoji.
O čemu pričaš? Vrednost 1/0 ne postoji, ali limes postoji. Plus beskonačno, ako x teži osmici sa gornje strane, a minus beskonačno, sa donje.
Sine Harvestere, zadatak razvaljuje, ali si ti pglu za matiš. :lol:
Zače, legendo!!! :!:
Ovaj prvi "rijeseni" zadatak i ovaj Peterov su - razvala... :mrgreen:
Quote from: Alex on 17-10-2009, 22:08:43
Quote from: Harvester on 17-10-2009, 20:42:19
Želim samo da napomenem da ovaj primjer sa limesom kad iks teži osam nije ispravan. Pomenuti limes zapravo ne postoji.
O čemu pričaš? Vrednost 1/0 ne postoji, ali limes postoji. Plus beskonačno, ako x teži osmici sa gornje strane, a minus beskonačno, sa donje.
Sam si ga objasnio. Da bi limes postojao, gornji i donji limesi moraju da budu jednaki.
Quote from: Harvester on 18-10-2009, 01:09:00
Quote from: Alex on 17-10-2009, 22:08:43
Quote from: Harvester on 17-10-2009, 20:42:19
Želim samo da napomenem da ovaj primjer sa limesom kad iks teži osam nije ispravan. Pomenuti limes zapravo ne postoji.
O čemu pričaš? Vrednost 1/0 ne postoji, ali limes postoji. Plus beskonačno, ako x teži osmici sa gornje strane, a minus beskonačno, sa donje.
Sam si ga objasnio. Da bi limes postojao, gornji i donji limesi moraju da budu jednaki.
Parti brejkere jedan, ti u sprdnju neveruješ.
Quote from: JasonBezArgonauta on 18-10-2009, 01:30:26
Parti brejkere jedan, ti u sprdnju neveruješ.
ovi matematičari su glupi za svesni humor.
ali im zato odlično ide nenamerni.
nerds!
Quote from: Ghoul on 18-10-2009, 03:43:55
Quote from: JasonBezArgonauta on 18-10-2009, 01:30:26
Parti brejkere jedan, ti u sprdnju neveruješ.
ovi matematičari su glupi za svesni humor.
ali im zato odlično ide nenamerni.
nerds!
It takes one to know one!
Kad smo već kod matematičkog humora:
(https://www.znaksagite.com/diskusije/proxy.php?request=http%3A%2F%2Fphotos-a.ak.fbcdn.net%2Fphotos-ak-snc1%2Fv2451%2F219%2F83%2F1633063574%2Fn1633063574_148442_1346213.jpg&hash=5b76de7dfc909d4c1e11ae5e989b671726eb11bf)
Ili:
(https://www.znaksagite.com/diskusije/proxy.php?request=http%3A%2F%2Fphotos-h.ak.fbcdn.net%2Fphotos-ak-sf2p%2Fv364%2F21%2F92%2F631511603%2Fn631511603_1420414_9067.jpg&hash=6c99620279849a6de9632c9764bc4cdb8dce1855)
Želim samo da napomenem da ovaj primer sa kvadraturom kad smajli teži sunđer bobu nije ispravan.
Zatvorili teorijskog fizičara, praktičnog fizičara i matematičara u 3 različite ćelije i dali im konzervu hrane bez otvarača i ostavili ih tako mesec dana.
Kada su stražari došli posle mesec dana zatekli su sledeću situaciju:
Kod teorijskog fizičara svi zidovi ispisani jednačinama, konzerva otvorena, hrana pojedena i fizičar živ i zdrav. Stražari ga pitaju kako mu je uspelo, a on im objasni kako je izračunao kojom jačinom, pod kojim uglom, u koju tačku..., da baci konzervu od zid da bi se ova otvorila.
Kod praktičnog fizičara zidovi stoje izlupani, krevet rasturen, on sav raščupan, ali konzerva otvorena, hrana pojedena i on živ i zdrav. Stražari ga pitaju kako mu je uspelo, a on im objasni kako je bacao konzervu različitom jačinom pod različitim uglovima sve dok mu nije uspelo da je razbije i otvori.
Kod matematičara ulaze stražari u ćeliju, ćelija uredna, krevet namešten, a matematičar leži mrtav na podu sa konzervom u ruci, a na konzervi čitkim rukopisom napisano:
Hrana je neophodna za život.
Pretpostavimo suprotno...
Nije loše, mada mislim da bi se prije desilo da matematičar lijepo izračuna sve ono što je sračunao i teorijski fizičar i onda da kaže "E lijepo, sve sam izračunao, sad nek fizičari to sprovode u djelo!" i umre od gladi.
When Zombies Attack!: Mathematical Modelling of an Outbreak of Zombie Infection
Abstract:
Zombies are a popular figure in pop culture/entertainment and they are usually
portrayed as being brought about through an outbreak or epidemic. Consequently,
we model a zombie attack, using biological assumptions based on popular zombie
movies. We introduce a basic model for zombie infection, determine equilibria and
their stability, and illustrate the outcome with numerical solutions. We then refine the
model to introduce a latent period of zombification, whereby humans are infected, but
not infectious, before becoming undead. We then modify the model to include the
effects of possible quarantine or a cure. Finally, we examine the impact of regular,
impulsive reductions in the number of zombies and derive conditions under which
eradication can occur. We show that only quick, aggressive attacks can stave off the
doomsday scenario: the collapse of society as zombies overtake us all.
Ceo "naučni rad" u PDF-u ovde (http://www.mathstat.uottawa.ca/~rsmith/Zombies.pdf).
Nego, šta je bilo sa prvobitnom namenom ovog topika - da se testira Ghoulovo znanje matematike?
Quote from: Milosh on 22-10-2009, 00:52:37
Nego, šta je bilo sa prvobitnom namenom ovog topika - da se testira Ghoulovo znanje matematike?
Ghoul nije htio da sarađuje. Ili nije uspio da riješi nijedan zadatak, pa se zato ućutao. :evil:
Quote from: Ygg on 22-10-2009, 01:43:29
nije uspio da riješi nijedan zadatak
Dođavola i ove dvostruke negacije u našem jeziku! Nemaju veze s logikom. Trebalo bi da stoji: "nije uspio da riješi ijedan zadatak" ili "uspio je da riješi nijedan zadatak". 8-)
(https://www.znaksagite.com/diskusije/proxy.php?request=http%3A%2F%2Fi542.photobucket.com%2Falbums%2Fgg429%2FBojan_B%2FRazno%2FMisc-Unknown-45740.jpg&hash=36fc7025e78c0815af1f77f1c0fd214092e34850)
Ha ha ha ha.
Dva tackasta naelektrisanja se nalaze na rastojanju r koje iznosi r puta koren iz 2.
Naelektrisanje prvog tela je q=8C a drugog q=2C. Koliko je naelektrisanje u tacki A koje se nalazi na pola puta izmedju dva naelektrisanja?
Quote from: kresha on 22-10-2009, 21:17:24
Dva tackasta naelektrisanja se nalaze na rastojanju r koje iznosi r puta koren iz 2.
Naelektrisanje prvog tela je q=8C a drugog q=2C. Koliko je naelektrisanje u tacki A koje se nalazi na pola puta izmedju dva naelektrisanja?
Ti bi da ti drugi rade domaći a? ;)
Ocekujem da ce neko do ponedeljka da resi :!:
Ne traži se naelektrisanje u tački A, valjda, nego jačina i smer električnog polja.
Pošto su tela istog tipa naelektrisanja, a na različitim krajevima, to se vrednosti njihovih polja prosto oduzimaju. Jednačina polja je E = kq/(r^2), gde je k elektrostatička konstanta, q naboj, r rastojanje. U našem slučaju je
E1 = k 8C / (r^2 / 2), E2 = k 2C / (r^2 / 2)
Jačina polja je E = E1 - E2 = k 6C / (r^2 / 2) = k 12C / r^2
Meni u zadatku pise naelektrisanje u tacki A.
Ali i ovo je super, bar cu nesto da napisem :)))
Aman, ko se ovde testeriše? Ygg, reaguj!
Ma ovo nije ni matematika ni logika, nego fizika. To tek nema veze sa Ghoulom.
Quote from: JasonBezArgonauta on 23-10-2009, 14:29:29man, ko se ovde testeriše? Ygg, reaguj!
samo se vi testirajte, I'm so outta here! :)
Quote from: kresha on 22-10-2009, 21:17:24
Dva tackasta naelektrisanja se nalaze na rastojanju r koje iznosi r puta koren iz 2.
Naelektrisanje prvog tela je q=8C a drugog q=2C. Koliko je naelektrisanje u tacki A koje se nalazi na pola puta izmedju dva naelektrisanja?
Quote from: mac on 23-10-2009, 11:46:56
Ne traži se naelektrisanje u tački A, valjda, nego jačina i smer električnog polja.
Pošto su tela istog tipa naelektrisanja, a na različitim krajevima, to se vrednosti njihovih polja prosto oduzimaju. Jednačina polja je E = kq/(r^2), gde je k elektrostatička konstanta, q naboj, r rastojanje. U našem slučaju je
E1 = k 8C / (r^2 / 2), E2 = k 2C / (r^2 / 2)
Jačina polja je E = E1 - E2 = k 6C / (r^2 / 2) = k 12C / r^2
Quote from: kresha on 23-10-2009, 13:17:16
Meni u zadatku pise naelektrisanje u tacki A.
Ali i ovo je super, bar cu nesto da napisem :)))
Hm... Ako se traži jačina električnog polja u pravu je
mac. A ako se traži naelektrisanje, onda zadatak baš i nema puno smisla. U takvoj vrsti zadataka obično su dva tačkasta naelektrisanja fiksirana a treće je u ravnoteži sa njima, pa se traži koliko je njegovo naelektrisanje (ili je njegovo naelektrisanje poznato pa se traži naelektrisanje nekog od ova dva fiksirana). Ako bi ova dva krajnja naelektrisanja bila fiksirana a ovo između u ravnoteži s njima, onda ono ne bi moglo biti na sredini, sem ako mu naelektrisanje nije nula. Valjda. Pa bi odgovor bio da je naelektrisanje u tački A nula i da ne zavisi od rastojanja koje nam je onda bezveze dato.
A možda si ti Krešo pogrešno zapisao taj zadatak? :?
Quote from: JasonBezArgonauta on 23-10-2009, 14:29:29
Aman, ko se ovde testeriše? Ygg, reaguj!
Ma nema veze! Bitno je da se ovdje popularizuje matematika & fizika & logika. :lol:
Ima da ovaj topik postane popularniji od
Best topika EVER! :evil:
xcheers
Quote from: Ghoul on 23-10-2009, 15:08:38
samo se vi testirajte, I'm so outta here! :)
Očigledno pratiš ovaj topik. To je dobar početak, vremenom ćeš početi i da aktivnije učestvuješ u njemu. :mrgreen:
ma samo prelećem pogledom da vidim da krišom ispod ovih bezveznjačkih formula i brojki i ostalih gluposti niste prokrijumčarili nešto što je actually interesantno, znajući da mene matematika zanima manje čak i od igrica.
Paranoja.
Evo jedno pravo pitanje:
Dokle zec trči u šumu?
Quote from: scallop on 23-10-2009, 18:03:53
Paranoja.
Evo jedno pravo pitanje:
Dokle zec trči u šumu?
say it in english, pa da testiramo tvoje znanje engleskog!
Evo nešto teže:
Kako izmeriti visinu bandere sa štopericom. (ovo je za one sa tehničkim fakultetom)
Ko daje ygg-u za pravo da popularise fiziku?
Ovde očigledno svako sebi uzima za pravo šta oće.
Quote from: scallop on 23-10-2009, 18:03:53
Paranoja.
Evo jedno pravo pitanje:
Dokle zec trči u šumu?
valjda dok ne stigne do nje, posle toga trci KROZ sumu:)
Quote from: scallop on 23-10-2009, 18:03:53
Paranoja.
Evo jedno pravo pitanje:
Dokle zec trči u šumu?
Do polovine šume. Nakon toga trči
iz šume.
Quote from: Ygg on 23-10-2009, 21:35:48
Quote from: scallop on 23-10-2009, 18:03:53
Paranoja.
Evo jedno pravo pitanje:
Dokle zec trči u šumu?
Do polovine šume. Nakon toga trči iz šume.
Tačno. That's my man.
Ako je tako, kad zeka trči po šumi?
a je l' taj zeka juri papagaja?
Quote from: belisarius on 23-10-2009, 19:57:18
Ko daje ygg-u za pravo da popularise fiziku?
Nisam rekao da ja popularišem fiziku, već da se fizika populariše na ovom topiku. xtwak
A ovaj zadatak iz fizike je gradivo osnovne škole, tako da to potpada pod maj fah. :lol:
S'm ti nastavi, Ygg. Meni ništa nije falilo da moj topik "Šta smo dobro krkali ovih dana" isprati 25.000 poseta.
Nego, šta je sa banderom?
Johne, zeka trči po šumi kad ga lovi sovuljaga.
Quote from: Ygg on 23-10-2009, 16:31:04
Quote from: JasonBezArgonauta on 23-10-2009, 14:29:29
Aman, ko se ovde testeriše? Ygg, reaguj!
Ma nema veze! Bitno je da se ovdje popularizuje matematika & fizika & logika. :lol:
Ima da ovaj topik postane popularniji od Best topika EVER! :evil:
xcheers
Best topik EVER nije best topik EVER zbog popularnosti nego zbog kvaliteta.
Skalope, nisi bio najprecizniji. Da li se pored štoperice mogu poneti i merdevine i teg od kilogama, na primer? Ili samo štoperica?
Mislim da se podrazumeva da možeš da se popneš na banderu i bez merdevina.
Klizave su bandere. Ja bez merdevina nikako, još i sa tegom... Ajde, štopericu mogu i u džep.
Mislim da može i samo štopericom, bez penjanja na banderu. Možda tako što se baci štoperica vertikalno uvis do visine bandere i onda hvata. Uključi se u momentu bacanja, a isključi u momentu hvatanja. A onda iz formule za slobodan pad možemo dobiti visinu. (Vrijeme potrebno da tijelo izbačeno uvis dođe do neke visine jednako je vremenu potrebnom za slobodan pad sa te visine, pa dobijeno vrijeme treba prepoloviti).
A i problem s penjanjem na banderu je kako zaustaviti štopericu kad padne na zemlju, ako si na banderi. Bila bi potrebna dva čovjeka.
Pa ako je to imalo normalna štoperica, zaustaviće se sama tako što će da se razbije o beton.
@ Ygg
Kilavo rešenje. Nemoguće je baciti štopericu idealno vertikalno uvis i pri tome dosegnuti visinu bandere. Drugo, trebalo bi na izračunato dodati i svoju visinu u predelu ruku (opet je to aproksimativno) jer nećeš hvatati štopericu na samoj zemlji, niti je sa zemlje možeš baciti.
Pih... sitnice. Koja ste vi zakerala! xfrog
E ovo je matiš
(https://www.znaksagite.com/diskusije/proxy.php?request=http%3A%2F%2Fwww.funxite.com%2Fmedia%2F9773-math.jpg&hash=5202cebf284c5a0d276167e1112f1c7b6b281d80)
Quote from: Harvester on 24-10-2009, 00:47:06
Pa ako je to imalo normalna štoperica, zaustaviće se sama tako što će da se razbije o beton.
Ygg plus Harvester.
Ajd da svoj 666. post utrošim pametno. :mrgreen:
Evo zadatka čiji je autor navodno Albert Einstein lično. Legenda kaže da je on tvrdio da samo 2% ljudi može da ga riješi za manje od 15 minuta.
Ovakvi zadaci zovu se integrami i poznati su svakom enigmati. Recimo, u hrvatskoj Kviskoteci postojala je (i vjerovatno postoji još uvijek) u svakom broju redovna rubrika sa zadacima ovog tipa, samo obično laganijim.
Zadatak glasi:
Uz rijeku je 5 kuća u 5 različitih boja. U svakoj od tih kuća živi čovjek različite nacionalnosti, s različitim kućnim ljubimcem, puši svoju vrstu cigareta i naviknut je na svoje omiljeno piće!
Poznati podaci:
1. Englez živi u crvenoj kući.
2. Švedjanin drži psa.
3. Danac pije čaj.
4. Zelena kuća je po redu odmah prije bijele.
5. Vlasnik zelene kuće pije kafu.
6. Osoba koja puši "Pall Mall" gaji ptice.
7. Vlasnik žute kuće puši "Dunhil".
8. Čovjek koji živi u kući u centru pije mlijeko.
9. Norvežanin živi u prvoj* kući.
10. Čovjek koji puši "Blend" živi pored onog koji drži mačku.
11. Onaj koji drži konje živi pored onog koji puši "Dunhil".
12. Čovjek koji puši "Blue Master" pije pivo.
13. Njemac puši "Prince".
14. Norvežanin živi pored plave kuće.
15. Čovjek koji puši "Blend" ima susjeda koji pije vodu.
Einsteinovo pitanje: Ko gaji ribe?
*Mala napomena. Čisto da ne bude zabune. Smatramo da su kuće poredane s lijeva na desno: dakle, prva kuća je prva s lijeve strane a posljednja je ona skroz desno.
Integrame je mnogo jednostavnije rješavati ako podatke prikažete tabelarno. Ja sam bio dobar pa sam napravio tabelu, pa ko hoće može da je isprinta i da pokuša zadatak riješiti pomoću nje.
(https://www.znaksagite.com/diskusije/proxy.php?request=http%3A%2F%2Fi36.tinypic.com%2F18kj2b.jpg&hash=2291d4a8b3c929833701825a8df3f106d4454a8b)
Glupo. Zar "prince" ne bi trebalo da puši Danac? To su danske pljuge. Malo je bezveze da ih puši Švaba. "Blue master" su norveške, valjda njih puši Norvežanin.
Pa dobro, ovo nisu tipični predstavnici svojih nacija. Mislim da se dobije i da Njemac ne pije pivo, niti Englez čaj.
Norvezanin gaji ribe.
Quote from: Ghoul on 01-11-2009, 17:45:20
Norvezanin gaji ribe.
Ne. Pogriješio si negdje.
Švabo!
Ovo me je inspirisalo. Odlučio sam da napišem naučni rad o integramima, tj. da sve svedem na matematičke pojmove i tako ovoj zanimljivoj igri oduzmem svu draž :evil: !
Jeste. švabo. :|
Pa ovo je čisto matematička igra, zato i jeste zanimljiva! :!:
Ajd sad objasni otkud Švabo...
Pa da. Zapravo je u integramu zadan neki skup U koji je jednak uniji šest skupova (u ovom slučaju skupovi nacionalnosti, kuća, rednih brojeva, ljubimaca, pića i cigareta, koje ćemo da označavamo sa N, K, R, LJ, P i C respektivno) od kojih svaki ima po pet elemenata i traži se da se odredi neka relacija R na tom skupu. Skup U ima dakle trideset elemenata, pošto svaki od ovih skupova ima po pet elemenata. Dekartov proizvod tog skupa sa samim sobom (oznaka UxU) je skup svih uređenih parova (a,b) gdje su obje komponente iz skupa U. Dakle, taj proizvod ima devetsto elemenata. Relacija na nekom skupu je, kao što znamo, podskup Dekartovog proizvoda tog skupa sa samim sobom i mi treba da odredimo tu relaciju. Prva olakšavajuća okolnost je što znamo da će ona imati manje od devetsto uređenih parova. Druga olakšavajuća okolnost jeste to što su uslovi zadani vrlo lukavo tako da ta relacija bude relacija EKVIVALENCIJE.
Naime, tu relaciju možemo da formulišemo na sljedeći način: Uređeni par (a,b) pripada relaciji R (ili: komponente a i b su u relaciji R) ako i samo ako postoji takav element c da važi: (a,c) je u R i (b,c) je u R. U ovom primjeru jedna od mogućih formulacija bi bila: Komponente a i b su u relaciji R ako i samo ako "pripadaju istoj kući" (pojam pod navodnicima nije najprecizniji, ali pretpostavlja se da se zna šta to znači). Primjera radi, da li su kafa i pas u relaciji, tj. da li se oni nalaze u istoj kući? Nakon postupka koji ćemo objasniti docnije dolazimo do informacije da je pas ljubimac pete kuće, a da se kafa pije u četvrtoj, te nijedan od parova (kafa,pas) i (pas,kafa) ne pripada našoj relaciji. Iz formulacije sa "pripadnošću istoj kući" potpuno jasno se vidi da je R relacija ekvivalencije. Refleksivnost očigledno važi jer niko ne može da živi u dvije kuće istovremeno (po uslovima zadatka). Simetričnost je takođe ispunjena jer redoslijed komponenti nije od značaja. Npr: Ako su a i b u relaciji R, tj. (a,b) pripada R, tada a i b žive u istoj kući, tj. a živi u kući K i b živi u kući K, tj. (zbog komutativnosti konjunkcije), b živi u kući K i a živi u kući K, tj. b i a žive u istoj kući K, tj. b i a su u relaciji R (odnosno (b,a) pripada R). Tranzitivnost: Ako parovi (a,b) i (b,c) pripadaju R, znači da a i b žive u istoj kući i b i c žive u istoj kući, tj. a živi u nekoj kući K, b takođe živi u nekoj kući K, b živi u nekoj kući L i c živi u toj kući L. Pošto b može da bude samo u jednoj kući, slijedi da je K=L, tj. c takođe živi u kući K, baš kao i a, pa su a i c u relaciji, tj. (a,c) pripada R.
Ova relacija ekvivalencije, kao i ostale, particioniše kompletan skup na dijelove koji su međusobno disjunktni a njihova unija je jednaka kompletnom skupu. U tim dijelovima, tzv. particijama, mi ćemo da dobijemo sve kombinacije uređenih parova oblika (a,b) gdje su a i b komponente koje egzistiraju u istoj kućici.
To je otprilike postavka problema, a poslije ćemo da opišemo njegovo rješavanje :evil:
Ovo je bio odgovor na Yggov post.
mac, suština je u sljedećem: svi uslovi koji su navedeni gore služe ili da nam eksplicitno navedu koje komponente su u relaciji ili da nam eliminiše neke komponente. primjera radi: Englez živi u crvenoj kući, znači uređeni par (Englez,crvena kuća) pripada R. Pošto svaka komponenta može da bude u relaciji sa samo jednom komponentom iste vrste (zbog particionisanja), slijedi da niko osim Engleza ne živi u crvenoj kući, te iz početnih 900 uređenih parova brišemo npr. parove (Norvežanin, crvena kuća), (Šveđanin, crvena kuća) itd. Takođe brišemo parove (Englez,plava kuća), (Englez, zelena kuća) itd. Kad ovim postupkom eliminišemo dovoljan broj parova, dolazimo lako do rješenja.
Harvi je to fino objasnio, samo previše matematički, rekao bih. ;)
@mac: Najlakše je riješiti zadatak preko tabele. Npr. dato je da Englez živi u crvenoj kući. To u tabeli možemo označiti tako što stavimo znak plus u polje u kojem se ukrštaju red ENGLEZ i kolona CRVENA. Možemo odmah staviti i minuse u sva polja u redu i koloni u kojem se nalazi to polje (unutar kvadrata 5x5 naravno), jer ako Englez živi u crvenoj kući, onda sigurno ne živi u kući neke druge boje i, ako u crvenoj kući živi Englez, onda u njoj sigurno ne živi neko druge nacionalnosti. Slično uradimo i za ostale uslove, s tim da neke uslove ne možemo odmah iskoristiti već tek kasnije. Npr. uslov 10. (Čovjek koji puši "Blend" živi pored onog koji drži mačku) nam u početku ne govori ništa osim da onaj koji ne puši Blend ne drži mačku, tek kasnije će taj uslov još doći do izražaja.
I tako, koristeći uslove zadatka i malo kombinatorike i zdrave logike može se postepeno popuniti cijela tabela plusevima i minusima. Dakle, tačno je određeno ko gdje živi, koje piće voli, kojeg ima kućnog ljubimca itd.
Okačiću malo kasnije i tabelu s rješenjem (tj. popunjenu), sad odoh da dovršim spremanje večere.
Znam ja kako se rešavaju ovi problemi (ja ih znam kao detektivske probleme). Mislio sam da Harvi objasni rečima kako je došao do konkretnog rešenja. Uzgred, nešto mi se javlja ovaj Ajnštajnov zadatak, i to da konačne tabele nisu potpuno popunjene.
Mac, meni je trebalo blizu dva sata da riješim ovaj problem, pošto integrame nikad prije nisam rješavao (a možda i zato što nisam pisao na papiru nego editovao sliku s tabelom u Paintu :evil:). Nema šanse da se sad sjetim postupka jer je isti vrlo dug i kompleksan.
Riječ-dvije o tabeli: Vidimo da tabela ima po pet grupa pojmova sa lijeve i gornje strane i da je dimenzija 25x25, s tim da se neka polja ne popunjavaju. Za predstavljanje čitavog Dekartovog proizvoda trebala bi tabela sa po 6 grupa pojmova lijevo i gore i ukupnih dimenzija 30x30. Međutim, po jedan pojam je izbačen gore i lijevo zbog ponavljanja (npr. nema potrebe da imamo parove tipa (životinja,životinja)). U ovoj "redukovanoj" tabeli takođe je uzeta u obzir simetričnost, pa se npr. odnos između ljubimaca i cigareta javlja (kako je i red) samo jednom - cigarete su četvrte odozgo, a ljubimci drugi s lijeva. To je ovaj "veliki" kvadrat koji se sastoji od 25 malih kvadrata. Odmah dijagonalno od njega (jedno mjesto dole i desno) bi se nalazio ponovo taj isti odnos, samo sa obrnutim redoslijedom, pa tog kvadrata nema.
Dakle, umjesto kvadratne dobijamo, uslovno rečeno, trouglastu tabelu u kojoj treba da popunimo svega petnaest velikih kvadrata, odnosno 15x25=375 polja. Ako su a i b u relaciji R (tj. (a,b) pripada R), ucrtavamo odgovarajući znak u onom redu u kom se nalazi pojam a i u onoj koloni u kojoj se nalazi pojam b. To neka bude recimo znak 1, što označava tačno. Ostala polja u tom redu i toj koloni u tom istom velikom kvadratu označimo nulama. Zašto? Pa, ako je pojam a u relaciji sa pojmom b iz neke grupe, onda on ne može biti u relaciji sa još jednim pojmom iz te grupe (to je uslov zadatka). Dakle, mi iz naše relacije R brišemo sve uređene parove tipa (a,b1), gdje je b1 različito od b.
Naročito je interesantno kretanje kroz ovu tabelu. Npr. ako znamo da se Blue Master puši u bijeloj kući, da je bijela kuća peta po redu i da u njoj živi Šveđanin, krenuvši od polja koje odgovara Blue Masteru i bijeloj kući (prva kolona, sedmo polje odozdo) možemo da dođemo do pete kuće idući prema gore, zatim odatle krenemo desno i završimo kod Šveđanina. Na osnovu tog pravougaonog kretanja možemo da zaključimo da Šveđanin puši Blue Master i to upišemo na odgovarajuće mjesto u tabeli. Usput možemo da pokupimo i razne druge podatke - npr. ako slučajno na našoj putanji prema Šveđaninu nabasamo na jedinicu tamo gdje se ukrštaju pivo i kuća broj pet, znamo odmah i da Šveđanin pije pivo.
Ovo pravougaono kretanje kroz tabelu je zapravo korišćenje svojstva tranzitivnosti relacije R. Ako su parovi (a,b) i (b,c) u relaciji R, tada je u njoj i par (a,c). Primjer: Ako Šveđanin pije pivo (par (Šveđanin,pivo) je u R) i ako Šveđanin živi u petoj kući (par (peta kuća, Šveđanin) je u R), tada je i par (pivo, peta kuća) u R.
Iz ovoga se lako vidi kako se jednostavniji uslovi tipa "Norvežanin živi u prvoj kući" prevode u uređene parove u relaciji.
Ali šta je sa komplikovanijim uslovima tipa "Čovjek koji puši Blend živi pored onog koji drži mački" ? Oni nam ne daju direktno parove koji pripadaju relaciji, nego neke parove koji NE pripadaju relaciji i još neke implikacije koje mogu da se koriste u kasnijem računu. Ako čovjek koji puši Blend živi pored onog koji ima mačku, znamo odmah da par (Blend,mačka) ne pripada našoj relaciji. Ovaj uslov nam takođe daje i sljedeći niz implikacija:
Ako par (Blend,prva kuća) pripada R, tada par (mačka,druga kuća) pripada R (tj. mačka živi u drugoj kući).
Ako par (Blend, druga kuća) pripada R, tada ili par (mačka,prva kuća) ili par (mačka,treća kuća) pripada R.
Na sličan način dobijamo još tri implikacije. Ovaj uslov postaje koristan kasnije kad otkrijemo u kojoj je kući Blend - to nam pomaže da suzimo broj mogućnosti za mačku.
Evo i rješenja. Ja sam pisao pluseve i minuse a Harvi je pričao o jedinicama i nulama.
Inače, Harvi je pojasnio kako se tabele popunjavaju. Objasniti riječima čitav postupak i dolazak do rješenja trajalo bi strašno dugo. Pogotovo što se tek na samom kraju otkrije ko to drži ribe, pa je potrebno popuniti maltene cijelu tabelu.
Lako je provjeriti da ovo jeste rješenje (jer zadovoljava sve uslove iz zadatka) ali bez popunjavanja tabele ne može se vidjeti da je to rješenje jedinstveno.
(https://www.znaksagite.com/diskusije/proxy.php?request=http%3A%2F%2Fi35.tinypic.com%2Fxmkcw5.jpg&hash=d5894c2c97479fa8432e37777ce49c5bf5ba3fa7)
Matematički vicevi! xnerd
(Besramno kopirani sa koleginog statusa na fejsbuku)
Logic is a systematic method for getting the wrong conclusion... with confidence.
A topologist is a person who doesn't know the difference between a coffee cup and a doughnut.
Q: Why do mathematicians, after a dinner at a Chinese restaurant, always insist on taking the leftovers home?
A: Because they know the Chinese remainder theorem!
Life is complex: it has both real and imaginary components.
Q: What is sour, yellow, and equivalent to the axiom of choice... A: Zorn's lemon...
Two mathematicians are studying a convergent series. The first one says: "Do you realize that the series converges even when all the terms are made positive?" The second one asks: "Are you sure?" "Absolutely!"
Q: What is purple and commutative? A: An abelian grape...
Q: What is normed, complete, and yellow? A: A Bananach space...
Q: How does a mathematician call his dog? A: Cauchy - because it leaves a residue at every pole...
Ovaj sa Košijem je sjajan, a ostali su onako.
vi, bre, matematičari, skroz ste izgubljeni slučajevi.
čak vam ni vicevi nisu smešni!
(tj, da bi bili smešni nekome, taj neko prvo mora da bude matematičar!)
Jedu mi se krofne.
Quote from: Harvester on 29-12-2009, 22:26:33
Jedu mi se krofne.
onda je trebalo da budeš pajkan a ne matematičar!
svaki pravi drot konzumira krofne
Opet frapantno nepoznavanje matematike, tj. u ovom slučaju logike. Ako svi policajci vole krofne, iz toga ne slijedi da su svi koji vole krofne policajci.
Ghoule, polako počinjem da sumnjam da će od tebe ikad biti matematičar. :roll:
Quote from: Ygg on 29-12-2009, 23:02:06
Opet frapantno nepoznavanje matematike, tj. u ovom slučaju logike. Ako svi policajci vole krofne, iz toga ne slijedi da su svi koji vole krofne policajci.
Ghoule, polako počinjem da sumnjam da će od tebe ikad biti matematičar. :roll:
Opet frapantno nepoznavanje psihologije.
Ako harvovom fetišu prema krofnama dodamo njegovo sporo shvatanje i stalno kašnjenje i kaskanje za događajima, 'otkrivanje amerike' koju su svi pre njega već otkrili, opsesiju trešom i neukusom, jeftinom zabavom (video igrice!), te patološku opsesiju sisama, onda vidimo da se harv savršeno uklapa u psihološki profil PAJKANA.
prema tome, moja aluzija je zasnovana ne samo na goloj logici, kako kolega ig naivno zamišlja, već pre svega na primenjenoj
dubinskoj psihologiji.
ali, ig, polako počinjem da sumnjam da će od tebe ikad biti psiholog.
:evil:
Quote from: Ghoul on 29-12-2009, 23:08:44
ali, ig, polako počinjem da sumnjam da će od tebe ikad biti psiholog.
:evil:
Ovo ću shvatiti kao kompliment. :lol:
Quote from: Ygg on 29-12-2009, 23:46:30
Quote from: Ghoul on 29-12-2009, 23:08:44
ali, ig, polako počinjem da sumnjam da će od tebe ikad biti psiholog.
:evil:
Ovo ću shvatiti kao kompliment. :lol:
ali zato ako nastaviš da se družiš s harvom, i naročito, da zajedno s njim gledaš filmove, imaš dobre šanse da postaneš predmetom proučavanja psihologa i psihijatara!
Ha! Polako počinjem da sumnjam da će od tebe ikad biti poznavalac naših filmskih večeri. Naime, neka od najvećih zlodjela koja smo gledali donio je upravo Ygg.
Quote from: Harvester on 30-12-2009, 00:09:06neka od najvećih zlodjela koja smo gledali donio je upravo Ygg.
u to sam apsolutno siguran!
Quote from: Ghoul on 30-12-2009, 00:12:36
Quote from: Harvester on 30-12-2009, 00:09:06neka od najvećih zlodjela koja smo gledali donio je upravo Ygg.
u to sam apsolutno siguran!
Doduše, kad se sjetim Kratkofila i onih tvojih Danaca "s kojima ne možeš pogriješiti"... :evil:
An infinite number of mathematicians walk into a bar. The first one orders a beer. The second orders half a beer. The third, a quarter of a beer. The bartender says "You're all idiots", and pours two beers.
Sjajno! xrofl
A evo još jedan vic koji s matematičarom:
Jedan matematičar je došao na konjske trke i tom prilikom uložio je sav novac kladeći se na konja broj 5. Uradio je to imajući u vidu da je rođen 5. maja 1955. godine, a tog dana je i njegova peta žena rodila peto dijete, dugačko 55 centimetara... Konj na koga se kladio stigao je peti.
Quote from: Melkor on 09-02-2010, 04:00:27
An infinite number of mathematicians walk into a bar. The first one orders a beer. The second orders half a beer. The third, a quarter of a beer. The bartender says "You're all idiots", and pours two beers.
:!:
У совјетској Русији, чувеном математичару дође пријатељ и каже му да ће сутра тајна служба да га ухапси и осуди - сад стрељање или Сибир - још се не зна.Пита он шта да ради...пријатељ му каже да побегне што дубље у унутрашњост, да се пријави у неки колхоз као неписмени радник, да оре и копа једно две године, па да се пријави у вечерњу школу - то комунисти воле. На крају ће завршити као наставник у некој школи и нико га неће приметити.
Он тако и уради. После две године, дођу да питају ко хоће у вечерњу школу, он дигне своју жуљевиту руку и позову га. Ко буде добар у школи - може и у партију да уђе!!!
Крене он у школу. Језик, прво вуку косе црте два часа, па усправне два часа...све он то преживи, али дође математика.
Први час бројеви од један до десет. Једно пети час, наставница напише на табли, после објашњења од пола сата о сабирању, 1+1=? и пита ко ће ово да реши. Он полуди и јави се. Крене са интегралима, диференцијалима, лимесима, вектори, векторска поља...
На крају добије резултат 1+1=3. Запањен стане, и пита - где сам бре погрешио? Из задњег реда се стидљиво дигне рука једног дрвосече који каже - код Кошијевог критеријума конвергенције низова!
jel si taj primer uzeo iz pavićeve knjige predeo slikan čajem ?
Ne, dobio sam ga u nekom od par desetina "zanimljivih i humoristickih" mailova koji stizu nedeljno.
Jeste, priča je očigledno pokradena iz Pavićevog
Predela slikanog čajem. Priča se zove
Šala o Fjodoru Aleksejeviču Razinu.
To je priča o jednom velikom matematičaru koji je živio u Moskvi, negdje u Staljinovo vrijeme. Pošto je on bio čuven profesor na univerzitetu tražili su od njega da se učlani u komunističku partiju. On je to nevoljno na kraju i uradio i na prvih nekoliko sastanaka samo je ćutao i zapisivao nešto u svoju bilježnicu, a onda se jednom javio za riječ i krenuo je da matematički objašnjava kako je to što oni tvrde u svojim govorima ustvari nemoguće.
QuoteI to je na sastanku rekao svojim neiskvarenim jezikom brojki, podvukavši da ono što traže drug A iz komiteta i drugarica B iz pratećih službi, ne može da u rezultatu da (kako oni očekuju) C, nego Y, pa prema tome da bi se dobilo željeno C, neophodno je i logično menjati baš ono što oni...
Itd. Uglavnom, na kraju je naš matematičar pobjegao iz Moskve u neki udaljeni gradić i tamo - čistio snijeg. Ubzo je proglašen za najboljeg čistača snijega u okrugu i onda su, gle čuda, opet od njega tražili da uđe u komunističku partiju. On se branio time da je nepismen, te su ga poslali na tečaj za nepismene. I tu dolazi do sjajnog finala priče:
QuotePotom se okrenula tabli [učiteljica], iz uha izvukla komadić krede i počela čas matematike.
— Jedan plus jedan — pisala je i naglas sricala Natalija Filipovna — jedan plus jedan jesu dva! I to i u ponedeljak i u utornik, upamtite. I juče su bili i biće vo vjek i vjekov dva i samo dva.
U sobi je bilo toplo, peć je počela da šeta ko puštena s lanca, svi su sricali: jedan plus jedan jesu dva, Fjodor Aleksejevič i sam je uzeo plajvaz da pribeleži ono sa table i tada ne izdrža. On tek sada shvati da se, otkad je uzeo lopatu da čisti sneg, ne znoji više i da sve to neisceđeno mora nekud iz njega napolje. I prvi put za sve ove godine ne izdrža. Ustao je odlučno, odmah udario glavom o tavanicu, izišao na tablu, obratio se svojim predašnjim samouverenim glasom Nataliji Filipovnoj koja je nemo gledala u njega i rekao na zaprepašćenje svih prisutnih:
— To je matematika XIX stoleća, draga Natalija Filipovna. Dozvolite da to primetim. Današnja, moderna matematika drugačije gleda na stvari. Ona zna da jedan i jedan ne moraju uvek biti dva. Dajte mi tu kredu za časak, pa ću vam odmah dokazati.
I Fjodor Aleksejevič je počeo urođenom brzinom ispisivati po tabli brojeve. Jednačina za jednačinom se redala, u prostoriji je vladao tajac, profesor je prvi put posle toliko godina ponovo radio svoj posao, doduše, onako pognut nije imao najbolji pregled brojki, kreda je čudno nekako škripala i odjednom rezultat je ispao sasvim protiv očekivanja Fjodora Aleksejeviča opet 1+1=2.
— Trenutak! — uzviknuo je Fjodor Aleksejevič — nešto nije u redu, samo trenutak, odmah ćemo videti gde je greška — a po glavi mu se motala besmislica: Sve izgubljene partije karata čine celinu! i od nje nije mogao da računa. Misli su grmele u njemu i grmljavina misli zaglušivala je sve ostalo.
Ali, njegova besprimerna umešnost išla mu je naruku, on je odmah znao gde će naći omašku i poleteo kredom po redovima ispisanih brojeva, sa kojih se već krunio beli prah.
I u tom času ceo razred, njih dvadeset četvorica, svi sem Natalije Filipovne Skargine, počeli su uglas da mu šapuću rešenje:
— Plankova konstanta! Plankova konstanta!
Ma, bre, sta vam je, to je Pavic pokrao matori ruski vic :D
(https://www.znaksagite.com/diskusije/proxy.php?request=http%3A%2F%2Fi146.photobucket.com%2Falbums%2Fr255%2Ffightclub2405%2F205-1.jpg&hash=b8b3093cdc0b164c6b994b53974597e664349159)
Nature by Numbers (http://www.youtube.com/watch?v=kkGeOWYOFoA#normal)
Predivno urađeno!!! Hvala Melkore! xjap
(https://www.znaksagite.com/diskusije/proxy.php?request=http%3A%2F%2Fwww.smileyvault.com%2Falbums%2Fsquare%2Fsquare-wink.gif&hash=79a1534cb5b7fb67241703250668453d106021c2)
da ste na vreme otvorili ovaj topik, mozda bi ghoul ponudio resenje poenkareove hipoteze i dobio/odbio milion zelembaca. :idea:
Praktična primjena Bajesove formule! :D
(https://www.znaksagite.com/diskusije/proxy.php?request=http%3A%2F%2Fspikedmath.com%2Fcomics%2F226-bayesian-inference.png&hash=b5f724b917016e89b7e25ca336b02437db429ae4)
Evo jedan logicki problemcic koji mi je neki direktor postavio kad sam isao na razgovor za poslovnu praksu. Problem je dakle trebalo resiti usmeno sto je znatno teze, pogotovu ako imate tremu na takvim razogovrima i glasi ovako: kazaljka na casovniku pokazuje 15.15h. Koliko je razmak u stepenima izmedju dve kazaljke?
Sigurno trik pitanje s obzirom da časovnici sa kazaljkama obično idu samo do 12 :lol:
Prosto je to, samo većina ljudi se zaleti i kaže 0, ne razmišljajući da i mala kazaljka prevali put za tih petnaest minuta...
@ Melkor "Nature by Numbers"
Ulepšalo mi dan! xcheers
Za petnaest minuta mala kazaljka predje četvrtinu od jednog sata, to jest četvrtinu od dvanaestine kruga, to jest 360 / 12 / 4 = 7.5 stepeni. Toliki je i razmak između kazaljki.
@Ygg
Promakao mi ovaj strip. Dobra fora, ali pogresno uradjen zadatak. Feler je recenica:"I compliment her eyes once a week". To daje vjerovatnocu 4/28 za totalitet B, a ne 4/30. Onda je u Bajesovoj formuli najbolje totalitet dobiti kao mnozilac 28 i 30 (ili NZS, svejedno, ali ovako je lakse), pa uspjesni ishodi eksperimenta postaju 120 i 112 (za date totalitete). Da se pitam, ja bi jos ubacio i naocare i frizuru kao minus (mozda kao jednu od disjunktnih suma parcijalnih vjerovatnoca u brojiocu?).
Ovo za kazaljke sam znao, a postoji dobra knjiga "How would you move mount Fuji?" koja u sebi ima zagonetke ovog tipa (koristi se i kao priprema za programerske intervjue). Zagonetka koje se sjecam:"Dato je 8 bilijarskih kugli i jedna vaga. Sedam kugli su iste tezine, a jedna je malo teza. Koliki je minimalan broj mjerenja da se jednoznacno odredi koja kugla je najteza?".
dva merenja.
to je stari zadatak, ali ima nadogradnju.
Šta ako ne znamo da li jedna kugla lakša ili teža, a da opet bude iz dva merenja. E, ovo je već horor.
A uz to i nemoguće. Mislim da je 5 kugli maksimum za dva merenja ako je nepoznata težina defektne kugle, a za 8 kugli ti treba tri merenja.
Edit: Ma nije ni 5 nego 4.
Danasnji poslodavci traze resavanje logickih testova i ovakvih pitanja ( i to ne samo za programere:) ) pa ako neko zna jos neke...
Quote from: Truman on 21-07-2010, 12:27:44
Evo jedan logicki problemcic koji mi je neki direktor postavio kad sam isao na razgovor za poslovnu praksu. Problem je dakle trebalo resiti usmeno sto je znatno teze, pogotovu ako imate tremu na takvim razogovrima i glasi ovako: kazaljka na casovniku pokazuje 15.15h. Koliko je razmak u stepenima izmedju dve kazaljke?
Hoću li ikada susresti takvog poslodavca, koji traži inteligenciju?
haha, pa ne znam na šta misliš..?
Ozbiljnije firme traže to.
Quote from: BladeRunner on 21-07-2010, 14:49:56
@Ygg
Promakao mi ovaj strip. Dobra fora, ali pogresno uradjen zadatak. Feler je recenica:"I compliment her eyes once a week". To daje vjerovatnocu 4/28 za totalitet B, a ne 4/30. Onda je u Bajesovoj formuli najbolje totalitet dobiti kao mnozilac 28 i 30 (ili NZS, svejedno, ali ovako je lakse), pa uspjesni ishodi eksperimenta postaju 120 i 112 (za date totalitete). Da se pitam, ja bi jos ubacio i naocare i frizuru kao minus (mozda kao jednu od disjunktnih suma parcijalnih vjerovatnoca u brojiocu?).
Da. Računato je da mesec ima 4 sedmice. Ako se računa precizno, dobije se na kraju vjerovatnoća od nekih 70%. Ali poenta stripa i dalje stoji, Bajesova formula ti povećava šansu da imaš seks sa djevojkom. :)
A ovo sa kazaljkama i bilijarskim kuglama je lagano. Takve zadatke radim sa klincima u osnovnoj školi na dodatnoj nastavi iz matematike.
Evo nekoliko zadataka (na fazon ovog s bilijarskim kuglama) koje sam zadavao klincima na dodatnoj:
Quote1. Располажемо са 9 килограма брашна и тегом од 250 грама. Како ћемо са три мјерења одмјерити тачно 2 килограма брашна?
2. Трговац треба да помоћу теразија препакује 20 килограма шећера у кесе од 2 килограма. Међутим, када је требало да започне паковање, утврдио је да нема одговарајући тег. Заправо, имао је само један тег од 3 килограма и један тег од 7 килограма. Међутим, снашао се и са 9 мјерења препаковао шећер у 10 кеса по 2 килограма. Како је то учинио?
3. У врећи се налази шећер у праху. Располажемо теразијама са два таса и само једним тегом од 1 грам. Како ћемо са десет мјерења измјерити 1 килограм шећера?
4. Од 9, по изгледу једнаких жетона, један је неисправан - мало је лакши од осталих. Како ћемо са два мјерења, на теразијама без тегова, открити који је жетон неисправан?
5. Показати да се са само 5 мјерења, на теразијама без тегова, од 100 по изгледу једнаких жетона, може открити жетон који је мало лакши од осталих 99 (исправних и једнаких по тежини).
6. Од 4 медаљона, потпуно једнаког изгледа, 3 су златна и 1 позлаћен. Позлаћени се разликује од златних само у маси. Са колико се најмање мјерења на теразијама без тегова, може открити позлаћени медаљон и да се још утврди да ли је лакши или тежи од осталих?
7. Између четири наизглед једнака новчића један је неисправан, тј. разликује се по маси од осталих, али није познато да ли је нешто лакши или тежи од исправних новчића. Поред тога, имамо на располагању још један новчић, за који сигурно знамо да је исправан. Помоћу највише два мјерења на теразијама са два таса треба пронаћи неисправан новчић и утврдити да ли је тежи или је лакши од исправних.
8. Између четири наизглед једнака новчића, два су неисправна а два исправна. Исправни новчићи су једнаке масе, а неисправни су такође једнаке масе међу собом, али нешто лакши од исправних. Како ћемо са два мјерења на теразијама са два таса утврдити који су новчићи исправни, а који неисправни?
Da skratimo priču: imaš N novčića ispravne težine, i M lakših novčića, i neograničenu količinu dodatnih ispravnih novčića. Koliko je merenja potrebno da se pronađe K od ovih M lakših novčića?
Nemam rešenje, samo pitam.
N+K-1 mjerenja. :!:
Bolje rješenje od ovoga ne uspjevam da smislim. :(
ovo mi je previše aptraktno, umesto neodređenih promenljivih više volim konkretne stvari poput one koju sam vam zadao.
Pa umjesto slova uzmeš konkretne brojeve. Npr. imaš 20 novčića i znaš da je među njima 12 ispravnih i 8 neispravnih. Neispravni novčići imaju manju težinu od ispravnih (težina im je međusobno jednaka). Osim toga imaš sa strane još ispravnih novčića u neograničenom broj (koliko god ti treba). Koliko je najmanje mjerenja na terazijama bez tegova potrebno da otkriješ 5 neispravnih novčića?
Meni ne pada na pamet nikakav bolji algoritam osim da poredimo jedan po jedan novčić sa ispravnim novčićem. Jer ako na terazije stavimo dvije hrpe sa jednakim brojem novčića i dobijemo da je lijeva hrpa lakša od desne, to samo znači da na lijevoj hrpi ima više neispravnih novčića nego na desnoj, a to nam ne govori mnogo. Valjda. :(
mac
QuoteDa skratimo priču: imaš N novčića ispravne težine, i M lakših novčića, i neograničenu količinu dodatnih ispravnih novčića. Koliko je merenja potrebno da se pronađe K od ovih M lakših novčića?
Nemam rešenje, samo pitam.
Svi junaci nikom ponikose, i u crnu zemlju pogledase... :). Nemam ni ja instant resenje, ali cu da razmislim pa da vidim da li mogu da dodjem do njega. Evo jos jednostavnije verzije: imamo N laksih i M tezih novcica. Koliko je minimalno mjerenja potrebno da se iz skupa N+M izdvoje podskupovi N i M? Pretpostavlja se da je razlika u masi laksih i tezih novcica veoma velika (kako bi se eliminisala situacija da bilo koja kombinacija masa daje slucajnu ravnotezu - e ovo bi bio komplikovan zadatak!). Inace, za ygg-ovo rjesenje mislim da nije minimalno (sto ne znaci da ja mogu bolje). Uporno mi na pamet pada Ramzeova teorema - nesto mi govori da moze da se iskoristi, ali trebalo bi prelistati par svesaka da vidim sta. Ko provali - neka javi! Pozdrav.
Treba krenuti od nekih malih brojeva i onda nekako pomocu indukcije.
Ala sam ga sredio!
Dok očekujemo da neko riješi ovaj naš problem evo malo zabave:
(https://www.znaksagite.com/diskusije/proxy.php?request=http%3A%2F%2Fspikedmath.com%2Fcomics%2F272-patch-notes.png&hash=4cedd083cb849e74ada45f5a9cd2be18284b486a)
Da, indukcija, ali svaki dokazani slučaj nije definisan samo parom utvrđenih novčića (n, m), nego nekakvim komplikovanijim skupom skupljenog saznanja o novčićima.
Quote from: BladeRunner on 21-07-2010, 14:49:56
Zagonetka koje se sjecam:"Dato je 8 bilijarskih kugli i jedna vaga. Sedam kugli su iste tezine, a jedna je malo teza. Koliki je minimalan broj mjerenja da se jednoznacno odredi koja kugla je najteza?".
uzmes sest kugli i stavis po tri na svaki tas. Uzmes trojku koja je teza i od nje uzmes proizvoljno 2 kugle koje stavis na po tas. I onda je jasno koja je teza, a ako nisu te dve onda je 3. A ako nije medju tih sest uzmes one tri i od njih dve raspodelis kao korak dva u prethodnom. Znaci dva merenja. Je l tacno?
Jes
stvarno nije tesko. A sta je ono rekao Boban, sto je zakomplikovao?
To je korektno matematičko razmišljanje. Kad rešiš lak problem onda probaj da rešiš i teži, i teži, sve dok ne dođeš do nerešivog. E taj problem je zapravo najinteresantniji.
Nisam bio nesto dobar matematicar u skoli. Mada mislim da je ovo vise logika nego matematika jer kad mi kazes matematika obicno pomislim na integrale i tu visoku matematiku...
Nego da vidimo sta je boban rekao...
Quote from: Boban on 21-07-2010, 14:59:39
dva merenja.
to je stari zadatak, ali ima nadogradnju.
Šta ako ne znamo da li jedna kugla lakša ili teža, a da opet bude iz dva merenja. E, ovo je već horor.
Razmislicu kad napisem ovo jer me prvo zanima - da l resenje ovog problema uopste i postoji?
Ne
Edit: to jest rešenje (odgovor) je da kuglica ne može da se odredi sa dva merenja.
Znaci, opet sam ispao smeker :)
(https://www.znaksagite.com/diskusije/proxy.php?request=http%3A%2F%2Fi146.photobucket.com%2Falbums%2Fr255%2Ffightclub2405%2Fblog%25202%2Fpigeons.jpg&hash=f359dfc42a32372fb8fddf8cf7b33860f5be278c)
(https://www.znaksagite.com/diskusije/proxy.php?request=http%3A%2F%2Fbitcast-a-sm.bitgravity.com%2Fslashfilm%2Fwp%2Fwp-content%2Fimages%2FZZ1E79F21E-550x687.jpg&hash=fe600319137da13a6f4ec0704ac89b4f7b9f1caa)
Prvo je Matrix, drugo bi moglo biti Sins, četvrto Mighty Ducks, peto Iron Cross, deseto Beauty and Beast.
Sedmo bi moglo da bude Sin City.
Tri vjerovatno Great balls of fire, ako se dobro sjecam topologije i Analize 2 :). Sest me nervira... mozda Social Network?
Odgovor:
1: The Matrix
2: Signs
3: The Fireball, or Ball of Fire, or Great Balls of Fire
4: Duck Soup, or The Mighty Ducks
5: Cross of Iron, or Iron Man 2
6: A Man Apart, or A Man Alone, or Social Network
7: Sin City
8: Heat
9: GoldenEye
10: Beauty and the Beast, or Angels and Demons
Nismo losi, nismo losi... 8-). Pozdrav (i cestitke za maca).
melkor, uzeo sam ti sliku i otvorio temu na Krstarici o tome. Pa ću da poredim čiji su forumaši pametniji. 8-)
ql, ionako nije moja :)
A ja, pametna, cijelo vrijeme mislim da je odgovor slika koja se, eto, samo meni nece ucitati... :x :oops:
Quote from: Trasharella on 24-03-2011, 00:17:03
A ja, pametna, cijelo vrijeme mislim da je odgovor slika koja se, eto, samo meni nece ucitati... :x :oops:
To treba računati kao skriveno (i kvarno) jedanaesto pitanje ;)
Osmo je Heat :)
(https://www.znaksagite.com/diskusije/proxy.php?request=http%3A%2F%2Fspikedmath.com%2Fcomics%2F424-the-numbers-quiz.png&hash=b219417099d2cfac476774e0542b0f9b9d95d642)
Ygg, nije lako, naročito kad ima 3 628 800 mogućih rešenja :(
Pa dobro, neki brojevi se daju lako izračunati a neke nije problem izguglati, tako da nema baš 10! mogućnosti. :lol:
Ustvari, brojevi su već poredani po veličini, tj. poredak je: a, b, c, d, e, f, g, h, i, j.
(https://www.znaksagite.com/diskusije/proxy.php?request=http%3A%2F%2Fi54.tinypic.com%2Fs5wdhz.png&hash=6f957062f35a4bc80d7c8ddd88321c5890ef6d1f)
Razne logičke igre na sledećem sajtu: http://www.logicgamesonline.com/ (http://www.logicgamesonline.com/)
Uspeo sam da rešim Daily Expert Netwalk sa 92 poteza za nekih 10 minuta (prvi put igrao). Problem sa expert težinom je da je matrica wrappovana (što nisam znao na samom početku).
Au, ja sam za ova Netwalk bas glup...
Jednom kad provališ stvari rešenja budu laka za početnu i srednju težinu, jer tu cevi ne mogu da štrče izvan matrice. Imaš i kratko uputstvo da te malo pogura, http://www.logicgamesonline.com/netwalk/tutorial.html (http://www.logicgamesonline.com/netwalk/tutorial.html)
Beginner i medium nivo su sasvim lagani ali za prvu igru na expert nivou potrošio sam pune 34 minute!!! :cry:
Popravljam se! 6:53!!! :!:
QuotePovodom velikog uspeha koji su mladi matematičari Srbije ostvarili na 52. Međunarodnoj olimpijadi iz matematike održanoj u Amsterdamu (četiri medalje), učenicima koji su osvojili odličja i njihovim profesorima, ministar prosvete i nauke prof. dr Žarko Obradović uputio je srdačne čestitke.
(https://www.znaksagite.com/diskusije/proxy.php?request=http%3A%2F%2Fwww.blic.rs%2Fdata%2Fimages%2F2011-07-25%2F161783_matematicka-gimnazija_f.jpg%3Fver%3D1311592770&hash=d091b70f7808992d2347564fa70ffbf6d0c380be)
Za ostvareni uspeh na Međunarodnoj olimpijadi u Holandiji, Ministarstvo prosvete i nauke će učenike i njihove profesore nagraditi sa sedmodnevnim pripremama na Zlatiboru za naredno takmičenje.
Čestitajući mladim matematičarima na osvojenim medaljama, ministar Obradović je istakao da je ostavreni uspeh još jedan u nizu izvanrednih rezultata koje naši mladi talenti postižu širom sveta na prestižnim takmičenjima i da to predstavlja potvrdu izuzetne nadarenosti, ali i ogromnog rada i zalaganja naših učenika, kao i predanog angažovanja i posvećenosti njihovih profesora, truda i brige roditelja i odgovornosti čitavog društva.
Ministar Obradović je naglasio da Ministarstvo prosvete i nauke podržava talentovane i vredne učenike, pre svega obezbeđujući uslove za njihovo kvalitetno obrazovanje i napredovanje, kao i omogućavajući njihovo učešće na takmičenjima na kojima mogu da iskažu rezultate svog rada i talenta.
Na Olimpijadi je Teodor fon Burg osvojio zlatnu medalju, Igor Spasojević srebrnu medalju, kao i Stevan Gajović, dok je Rade Špegar osvojio bronzu. Filip Živanović je dobio pohvalu.
Njih petorica su učenici Matematičke gimnazije u Beogradu.
Na Olimpijadi je učestvovao i Stefan Mihajlović, učenik gimnazije ,,Svetozar Marković" iz Niša.
U pratnji učenika su bili lideri reprezentacije profesori Đorđe Krtinić i Dušan Đukić.
Na Olimpijadi su učestvovali učenici iz 105 zemalja.
http://www.blic.rs/Vesti/Beograd/267865/Ucenicima-Matematicke-gimnazije-poklon-i-pohvale-od-ministra-prosvete (http://www.blic.rs/Vesti/Beograd/267865/Ucenicima-Matematicke-gimnazije-poklon-i-pohvale-od-ministra-prosvete)
A evo i reakcije News.net-a (http://www.njuz.net/vise-hiljada-beogradana-okupice-se-slucajno-u-centru-grada-u-isto-vreme-kada-mladi-matematicari-donesu-cetiri-medalje-iz-holandije/)!
QuoteBEOGRAD, 25. jul 2011, (Njuz) – Olimpijske šampione u matematici, reprezentativce Srbije u ovom sve popularnijem sportu, po povratku iz Holandije dočekaće hiljade slučajnih prolaznika u centru Beograda.
Da podsetimo, Teodor fon Burg (zlato), Igor Spasojević i Stevan Gajović (srebro), Rade Špegar (bronza) je četverac koji je nastupajući pod srpskom zastavom oduvao i favorite poput Indije i SAD.
Nekoliko desetina rođaka dočekalo je naše matematičare već na aerodromu Nikola Tesla odakle će se posebnim autobusom sa natpisom ,,Lasta" uputiti u centar Beograda. Dolazak u Skupštinu se očekuje oko 17 časova gde ih niko neće dočekati niti sprovesti do balkona.
- Ovo je veliki uspeh za sve nas koji nismo prstom mrdnuli da bi ova deca imala kakve takve uslove za pripreme i napredak – rekao je ministar prosvete i nauke dr Žarko Obradović. – Ovde smo da obećamo da nameravamo da nastavimo dalje sa potcenjivanjem svega što nas ne stavlja na naslovne strane novina i u VIP boksove na utakmicama.
Sve televizije sa nacionalnom frekvencijom će prenositi događaj uživo uz prekide radi prenosa prijateljske fudbalske utakmice Javor – BSK Borča, izveštaja o kupanju i češljanju psa porodice Đoković i turske serije ,,Golum".
http://www.cracked.com/article_19360_7-questions-you-didnt-know-could-be-answered-with-math.html (http://www.cracked.com/article_19360_7-questions-you-didnt-know-could-be-answered-with-math.html)
Ovo su klinci (8-9 raz.) dobili u prvom kolu Turnira gradova:
Iz grada A su u 10 sati krenuli biciklista i pešak, ka gradu B. Iz smera B ka A idu zaprežna kola i automobil. Svi nabrojani se kreću konstantnom (svojom) brzinom. Biciklista posle nekog vremena sreće zapregu, a posle isto toliko vremena – automobil. Od tog trenutka, automobil posle nekog vremena nailazi na pešaka, a posle istog tog (drugog) vremena, prolazi pored zaprege. Pešak i automobil su se sreli u 11 sati. Kada su se mimoišli pešak i zaprega?
Problem mi je: dok sam rešavala, nekako sam dokazala da pešak i zaprega idu istom brzinom (ili nisam?). Tada je rešenje očigledno i dalje mogu da dobijem odnose svih brzina (nepotrebno, ali me je zanimalo da li je logično). Ima li neko strpljenja da napiše jednačine koje ovo dokazuju, ili opovrgavaju?
Quote from: Midoto on 26-10-2011, 15:23:48
Ovo su klinci (8-9 raz.) dobili u prvom kolu Turnira gradova:
Iz grada A su u 10 sati krenuli biciklista i pešak, ka gradu B. Iz smera B ka A idu zaprežna kola i automobil. Svi nabrojani se kreću konstantnom (svojom) brzinom. Biciklista posle nekog vremena sreće zapregu, a posle isto toliko vremena – automobil. Od tog trenutka, automobil posle nekog vremena nailazi na pešaka, a posle istog tog (drugog) vremena, prolazi pored zaprege. Pešak i automobil su se sreli u 11 sati. Kada su se mimoišli pešak i zaprega?
Problem mi je: dok sam rešavala, nekako sam dokazala da pešak i zaprega idu istom brzinom (ili nisam?). Tada je rešenje očigledno i dalje mogu da dobijem odnose svih brzina (nepotrebno, ali me je zanimalo da li je logično). Ima li neko strpljenja da napiše jednačine koje ovo dokazuju, ili opovrgavaju?
Aj proveri postavku zadatka još jednom.
Iz ovog prvoge dela "
Biciklista posle nekog vremena sreće zapregu, a posle isto toliko vremena – automobil" se, na prvi pogled, zaključuje da je zaprega brža od automobila, ali dobro to nije ni toliko sporno, ipak je moguće. Al ovaj deo je me buni "
Od tog trenutka, automobil posle nekog vremena nailazi na pešaka, a posle istog tog (drugog) vremena, prolazi pored zaprege". Pošto je zaprega brža od automobila ne vidim kako bi on nju mogao da obiđe s obzirom da se krenuli iz mesta B.. ili možda nisu krenuli iz mesta B (jer ipak to nije tačno navedeno u zadatku) iz čega jedino zaključujem da se zaprega nalazila nagde na relaciji B-A, dok je automobil (koji se kreće brže od nje) bio iza nje (odnosno bliži gradu B), pa je u jednom momentu prestiže. Jel to slučaj ili...?
Zaprega je bliža (navodi se samo da iz istog smera idu ka A) - zato ona prva sreće biciklistu.
Evo početnog položaja, u 10 sati:
Biciklista i pešak (u gradu A; idu ka B) Zaprega (ide ka A) Automobil (ide ka A i dosta je daleko :) )
Tako je. Ja sam juče na poslu nacrtao skicu, i postavio jednačine, i na brzinu pokušao da rešim, ali bez uspeha. Imam utisak da ili nedostaje neki podatak, ili u rezultatu učestvuju odnosi brzina nekih učesnika, ili se nisam dovoljno udubio u jednačine. Inače, evo jednačina, pa nek drugi lome glavu:
2T1 + T2 = 1h
vb * 2T1 = vp(2T1 + T2) + va*T2
vb * T1 = vp(2T1 + 2T2) + vz(T1 + 2T2)
va*T2 = vp*T2 + vz(T1 + 2T2) + vb*T1
T2 = ?
Hvala, vrtela sam se u tom krugu i odustala, jer nisam dolazila do rešenja, sem onog pominjanog - kada sam na osnovu nekih skica dobila da pešak i zaprega idu istom brzino. No, kada sam deci pokušala da objasnim kako sam došla do rešenja (10:40), nisam uspela da ponovim podvig, a papire sam pobacala (baksuzno).
U svakom slučaju, kada ni Mac ne dolazi lagano do rešenja, zadatak istinski nije jednostavan.
Taman sam hteo da kažem da po mojoj skici rešenje mora da bude posle 11h, ali sad vidim da sam pogrešio u skici, a to znači da su i jednačine pogrešne. Možda ipak postoji rešanje :-)
Pa ja prvo nisam pazljivo procitao zadatak, tj predpostavio sam da automobil i zaprega krecu iz B, pa sam onda njihov predjeni put izrazavao preko predjenog puta ovih koje susrecu, pa sam tu nesto motao dok nisam dosao do ovog dela kad automobil obialzi zapregu, e tu sam skontao da mi nesto ne valja :) Probacu opet kasnije, sad mi mrsko...
Evo nadam se dobrih jednačina:
2T1 + T2 = 1h
vb2T1 = vp(2T1 + T2) + vaT2
vbT1 = vpTx + vz(Tx - T1)
vaT2 = vp(2T1 + T2 - Tx) + vz(2T1 + 2T2 - Tx)
va2T2 = vbT1 + vz(T1 + 2T2)
Tx = ?
Macu, a koje si ti skole zavrsio? :mrgreen:
Mene je učila ulica. Prvo je tu bila Ulica Sezam, pa PC magistrala, i na kraju Information superhighway :-)
:-|
ovo je zanimljivo!
Ove jesu tačne, s tim što ja nisam ubacivala treće vreme, nego razliku između puteva bicikliste i automobila. Ali: postoji 5 jednačina, a 7 nepoznatih (3 vremena i 4 brzine).
Nedostaje jedna jednačina za koju sam mislio da je redundantna. Evo nadam se kompletnog sistema dobrih jednačina:
2T1 + T2 = 1h
vb2T1 = vp(2T1 + T2) + vaT2
vbT1 = vpTx + vz(Tx - T1)
vaT2 = vp(2T1 + T2 - Tx) + vz(2T1 + 2T2 - Tx)
va2T2 = vbT1 + vz(T1 + 2T2)
vz(Tx - T1) + vbT1 = vp(2T1 + T2 - Tx) + vaT2
Tx = ?
Zahvaljujem. Ne znam samo kako se deca iskobeljavaju iz ovoga, ali - neka se znoje.
Ja sam se juče zakopao u tim jednačinama, i kad kod sam pronašao novu ona je pretumbavanjem davala jednu od starih, pa sam se smorio. Inače, mac mislim da bi trebala još jedna jednačina, jer koliko se ja sećam za određen broj nepoznatih treba isti taj broj jednačina, al ja više ništa ne vidim i samo presipam iz jedne u drugu.
Ali zato mi je danas pala na pamet druga metoda za rešavanje: grafička. X osa za vreme t, a Y za pređen put s; nacrtati prave koje predstavljaju brzine i naći tačku preseka između prave za zapregu i za pešaka.
Evo kako mi izgledaju ose:
X
0----t1----t2---t3---t4
Y
0----s1----s2--s3------s4
Tačke na osi stavljene su proizvoljno (ove crtice na osama "-" predstavljaju kvadrate na papiru, a mesto gde dođe oznaka (t1,t2...) su granice između kvadrata (ovo pojašnjavam ukoliko neko želi da crta, a mislim da neće biti takvih:P)), bitno je da su ispoštovane relacije
0-t1 = t1-t1, vreme t1 iz jednačina (4 kocke/crtice)
t2-t3 = t3-t4, vreme t2 (3 kocke/crtice)
0-s2 = s2-s4, pređeni putevi bicikliste do susreta sa zapregom i autom (8 koc/crt)
s4-s3 = s3-s2, pređeni putevi auta do susreta sa pešakom i zapregom (6 koc/crt)
Dužine puteva sam nanosio ovim redom: s2, s4, s3, s1, ali sam ih onda obeležio ovako od početka ose.
Kod crtanja bitno je još i da je
0-t1 = t1-t1 > t2-t3 = t3-t4
0-s2 = s2-s4 > s4-s3 = s3-s2
Veličine ovih delova nisu bitne, koliko kod da se promene položaji prava će se takođe menjati tako da će proporcija biti očuvana.
Koordinate za prave
Vb (t1,s2) (t2,s4)
Va (t2,s4) (t3,s3) (t4,s1)
Vz (t1,s2) (t4,s1)
Vp (t3,s4), sve ove koordinate su iz zadatka na osnovu preseka susreta vozila jedno sa drugima, ali Vp ima i kordinatu (0,0) (kao i Vb)
Mesto preseka prava Vp i Vz daje tačku (tx,sx). Znači treba naci jednačine prave Vp i Vz, izjednačiti ih po y (što predstavlja sx) i jednačina se samo svede po tx. Dalje prebacivanje treba svesti da jednačina zavisi samo po 2t1+t2=1h i onda dobiti neki kojeficijent uz ovih 1h, što će biti vreme proteklo od 10h. E sad, teži deo je što se treba izboriti sa silnim računanjem i sve to lepo svesti da se ostale nepoznate stvari skrate... što meni nije uspelo iz nedostatka živaca :)
Ali, može sledeća metoda koja nije validna, a to je lenjirom izmeriti tx i t3 (tj. duž 0-t3), pa se vreme dobija kao 60min*(tx/t3) i meni se taj rezultat mota oko 40 min, jer tu ima i greška oko netačnog merenja tx, ali kontam da u nekom programu može da se dobije tačan odnos tx/t3.
Ono što me muči jeste da sam i ja dobio rešenje kao Midoto, tj 10:40, ali ona je dobila da je Vp=Vz, što ja nisam uspeo da dobijem ni iz jednačina (bar iz onog prvog seta nepotpunih jednačina), a ni sa grafika, na kome se isto vidi da su brzine Vp i Vz različite...
Btw žao mi dece :)
Jednačine koje je ispisao mac su ispravne i zadatak se može riješiti pomoću njih, s tim da nam čak nisu sve neophodne. Druga i šesta su višak (šesta se može dobiti kombinacijom preostalih, pa je višak svakako).
Dakle, trebaju nam jednačine:
2T1 + T2 = 1h
vbT1 = vpTx + vz(Tx - T1)
vaT2 = vp(2T1 + T2 - Tx) + vz(2T1 + 2T2 - Tx)
va2T2 = vbT1 + vz(T1 + 2T2)
Druga, treća i četvrta jednačina se malo preurede:
(vb + vz)T1 = (vp +vz)Tx
2(vp + vz)T1 + (vp +vz)T2 = (va - vz)T2 + (vp +vz)Tx
(vb + vz)T1 = 2(va - vz)T2
Iz prve i treće jednačine se dobije:
(va - vz)T2 = 1/2 (vb + vz)T1 = 1/2 (vp +vz)Tx
Kad se to uvrsti u drugu jednačinu dobije se:
(vp + vz)(2T1 + T2) = 1/2 (vp +vz)Tx + (vp +vz)Tx
Odnosno:
1 = 3/2 Tx, tj. Tx = 2/3 h = 40 min
:shock:
Vas trojica me plašite.
Znao sam da postoji elegantnije rešenje, al što jednostavno kad može koplikovano :)
Još dva - hvala. Upravo sam dobila opkladu (protiv sina) da će neko odavde do danas u 19h imati: tačno vreme uz ispravan postupak. Ja stvarno nisam uspevala da dokažem, čak sam se upetljala u vrlo komplikovane jednačine odnosa različitih brzina, mada mi je bilo jasno da deca tog uzrasta još ne vladaju "matematičkim alatima" da bi se odatle izvukla (dakle - pogrešan mi je pristup rešavanju). A ovo rešenje je stvarno fino.
Zgg je kvarni matematičar koji je sve ovo znao još kao klinac. xrotaeye
Quote from: Midoto on 28-10-2011, 19:49:28
Još dva - hvala. Upravo sam dobila opkladu (protiv sina) da će neko odavde do danas u 19h imati: tačno vreme uz ispravan postupak. Ja stvarno nisam uspevala da dokažem, čak sam se upetljala u vrlo komplikovane jednačine odnosa različitih brzina, mada mi je bilo jasno da deca tog uzrasta još ne vladaju "matematičkim alatima" da bi se odatle izvukla (dakle - pogrešan mi je pristup rešavanju). A ovo rešenje je stvarno fino.
Pogresan pristup je i mene zeznuo, jednacine su mi bile po dva reda, a u sebi sam se uporno pitao "kako oni ocekuju da dete 8. razred ovo resi?" :)
Ajde sad daj jos jedan zadatak, kladi se sa detetom, pa da opeljesimo klinca :evil:
Evo dokaza i da je vp = vz:
2T1 + T2 = 60min
Tx = 40min
vz(Tx - T1) + vbT1 = vp(2T1 + T2 - Tx) + vaT2
vz(Tx - T1) + vpTx + vz(Tx - T1) = vp(2T1 + T2 - Tx) + vp(2T1 + T2 - Tx) + vz(2T1 + 2T2 - Tx)
2vz(Tx - T1) = 2vp(2T1 + T2 - Tx) + 2vz(T1 + T2 - Tx)
vz(2Tx - T2 - 2T1) = vp(2T1 + T2 - Tx)
vz(80 - 60) = vp(60 - 40)
vz = vp
Quote from: Dr00d on 28-10-2011, 21:46:46
Ajde sad daj jos jedan zadatak, kladi se sa detetom, pa da opeljesimo klinca :evil:
Uvek sam za pelješenje samouverene balavurdije, ali je dotični postao sumnjičav. Ne bih ga više tako lako navukla. Čekaj dok malo zaboravi ovo. A i da Rusi smisle još gadnih zadataka. Ovo drugo kolo je bilo lakše, pa mi nisu dali materijal (zapravo - meni jesu, ali ih je on rešio).
Mac, i ja sam upravo dokazivala da sam u startu bila u pravu, samo se mnogo nerviram što sada do toga mogu da dođem tek na kraju. Verovatno je bio samo - srećan pogodak.
Da, jeste Vz=Vp
Ja sam dosao do
Vz = (Va*2t2-Vb*t1) / (t1+t2)
Vp = (Vb*2t1-Va*t2) / (2t+t2)
Ali to nikao nisam mogao da prebacim na jednakost, a dalje sledi ubacivanje tx sto je napravilo jos vecu zbrku. Posle sam presao na graficku metodu i odustao od ovoga. I sad gledam grafik, tj dva grafika, pogresno sam zakljucio da ce prave sa istim brzinama ali obrnutih smerova kretanja praviti, pa moze se reci, jednokostranican troguao sa X osom, ali ova simetricnost se gubi zbog proizvoljno uzetih t1,t2... s1,s2... Gledam durgi grafik koji sam posle nacrtao da proverim da li ce i on dati 40 min sa drugacijim vrednostima za t1,t2.. kod njega prave prave skroz drugacije uglove, ali ipak daje 40 min. Jbga, moja pogresna predpostavka. Ovo je ispalo kao zadatak iz nacrtne geometrije. Aaaaa ne mogu vise da mislim o ovom zadatku glava me boli xrotaeye Bacam sve ove isvrljane papire..
Kolko kinte je bilo u igri?
Nije baš toliko težak zadatak, čini mi se da su se mimoišli u 10:40. Pokušaću da skiciram i da okačim rešenje, pa ćete videti o čemu govorim.
Hm, vratili se i Ig i Mušet, da li to znači da i Rejnoldsa možemo da očekujemo i to u matematičarskom naponu? :lol:
Quote from: Dr00d on 28-10-2011, 23:05:08
Kolko kinte je bilo u igri?
Sitne pare (100 din, koji su na njegovo insistiranje prerasli u 150), ali mnogo njegove samouverenosti. A siroma' je novčano ufitiljio, jer sve petice iz obe matematike i iz fizike, potru dvojke iz predmeta koji treba da se nauče. Računam da je dovoljno pametan da shvati vrednost rada, pre gimnazije. Roditeljski optimizam!
Evo objašnjenja, pa ću okačiti i skicu:
put je linerana funkcija vremena, sa koeficijentom pravca koji odgovara brzini svakog od njih, pa ako prikažemo grafički pređeni put i presečne tačke ovih grafika, dobijamo donju sliku (B, P, A, Z su biciklista, pešak, auto i zaprega).
Vremena sam obeležila kao i vi, sa T1 i T2, kako bismo se razumeli.
E, sad je još potrebno pokazati (vrlo lako!) da je mesto susreta ZP (zaprege i pešaka) težište trougla čija sam temena obeležila crveno, pa ono deli težišnu duž u odnosu 2:1, a po Talesu i na vremenskoj osi imamo isti odnos, dakle susret se odigrao u 10:40
Nisam baš sigurna da sam dovoljno jasna, ali se nadam da će me razumeti bar ovi koji su pokušavali da reše zadatak algebarskim putem :)
Sve u svemu, trebalo je malo maštovitije pristupiti rešavanju zadatka :lol:
(https://www.znaksagite.com/diskusije/proxy.php?request=http%3A%2F%2Fi44.tinypic.com%2F2lq143.jpg&hash=97ee3eea5355cd43648d42f2e49307defc7ea699)
Uh, tek sada sam videla da je Ygg već rešio zadatak :-x
Neću da hvalim ovo moje rešenje iako pretpostavljam da su očekivali da će zadatak tako rešiti i dete u osmom razredu :)
Quote from: Jevtropijevićka on 29-10-2011, 00:27:35
Hm, vratili se i Ig i Mušet, da li to znači da i Rejnoldsa možemo da očekujemo i to u matematičarskom naponu? :lol:
:)
Ja ne poznajem Rejnoldsa, pa ne mogu da odgovorim :( ali sam malopre videla zadatak i zainteresovao me je. Mislila sam da ga, avaj, niko do sada nije rešio :-x
Odlično rješenje Mouchette! I veoma elegantno!!! Ja sam vidio mac-ove jednačine i krenuo od njih, mada mi se sve vrijeme vrzmalo po glavi kako mora postojati i neko jednostavnije rješenje, jer to su ipak zadaci za osnovce, nema šanse da se oni izbore sa onim jednačinama.
I ja sam se vratio samo ovdje. Ipak sam ja pokrenuo ovaj topik, pa sam na neki način odgovoran za njega! Na ostatku foruma me i dalje nema. :lol:
Jako mi je lepo rešenje. Ipak, mislim da se očekivalo algebarsko rešenje, jer dete nema predstavu kakav je odnos brzina zaprege, bicikla i pešaka, jedino zna da je automobil bitno brži od svih (a ni to ne mora da znači, ako su gledali Tour de France). U slučaju da pokušam da skiciram da je, napr. zaprega brža od bicikliste (rekreativca koji uživa u okolišu :) ), ne mogu da dobijem traženo rešenje, pa ni ako je pešak puno brži od zaprege, ili obrnuto.
Quote from: Jevtropijevićka on 29-10-2011, 00:27:35
Hm, vratili se i Ig i Mušet, da li to znači da i Rejnoldsa možemo da očekujemo i to u matematičarskom naponu? :lol:
:D
Quote from: Midoto on 29-10-2011, 01:17:26
Jako mi je lepo rešenje. Ipak, mislim da se očekivalo algebarsko rešenje,
Hvala Ygg i Midoto :) Sigurna sam da su sastavljači očekivali baš ovakvo rešenje jer učenici, koliko ja znam, uče grafik linearne funkcije baš u osmom razredu. Takođe sam sigurna da se sa sistemima sreću tek na kraju osmogodišnjeg školovanja, i to isključivo sa sistemima sa dve linearne jednačine i dve nepoznate, a taj algebarski pristup je ovde mnogo komplikovaniji.
Ovo je više logička zavrzlama koja ne zahteva preveliko poznavanje matematike, koliko slobodu i maštovitost.
Inače, baš mi je drago što sam rešila zadatak za osmaka :!: U tom uzrastu sam bila na saveznom takmičenju matematičara bivše SFRJ, među desetak najboljih iz svih republika. Eto, nisam još za bacanje :lol:
Ne mešam se mnogo u matematiku koju uče u Mg (sa osnovcima), samo me je iznenadilo kada sam videla da su na početku osmog razreda radili trigonometriju. Nemam ništa protiv toga, čak mi je i logičnije da to prvo prorade sa nastavnicima matematike, nego da se prvi put sa tim sretnu u fizici.
Savezno takmičenje - znači 1. republička? Kako si prošla?
Midoto, ne, nisam bila baš prva u republici, ali sam bila jedna od nekoliko najboljih koji su putovali iz Srbije. S obzirom da sam iz malog mesta i da nisam imala nikakvog mentora, i ovo je uspeh. Razmišlajala sam da upišem MG, ali znaš već, bila sam dete, još iz provincije, i te stvari... Doduše, zvali su me kasnije na neke letnje škole matematičara, Šuplja Stena i tome slično.
Quote from: Mouchette on 29-10-2011, 02:10:18
Razmišlajala sam da upišem MG, ali znaš već, bila sam dete, još iz provincije, i te stvari... Doduše, zvali su me kasnije na neke letnje škole matematičara, Šuplja Stena i tome slično.
Naravno da su te zvali. Biti među najboljima na nivou nekadašnje Jugoslavije pokazuje da postoji ta odlična kombinacija: briljantan um, a i dovoljno prilježnosti.
Ćerka je ove godine upisala MG i ima nekoliko drugarica i drugova u odeljenju koji nisu iz Beograda i toj deci je stvarno teže - odvojenost od roditelja, poznate sredine, direktne podrške porodice...
Tačno, težište trougla, a ja ko slepac merio lenjirom :x
Mouchette svaka čast na uočavanju.
Quote from: Dr00d on 29-10-2011, 13:10:12
Tačno, težište trougla, a ja ko slepac merio lenjirom :-x
Mouchette svaka čast na uočavanju.
Hvala Dr00d :) Po prvi put sam rešavala neki zadatak na ovaj način, nikad nisam videla ništa slično, ali je ispalo da je ideja dobra. I lepa :lol:
Klinci se oduševili. Sin kaže da je pokušao grafički, ali je mislio da zaprega "jezdi".
Našla sam jedan vrlo simpatičan test, samo da ga prenesem.
Evo ga (jedva sam ga iskopala), nije baš matematika, ali je zabavno za ljude koji vole mozgalice:
RB Šifra Odgovor
0 24 S u D 24 sata u danu Tačno
1 30 S u A
2 7 D u N
3 7 S Č
4 12 Z u H
5 52 K u Š (B Dž)
6 4 S na S G
7 18 R na G I
8 5 P na S
9 90 S u P U
10 32 je T u S F na kojoj V S
11 15 I u R T
12 3 T na T
13 11 I u F T
14 12 M u G
15 29 D u F u P G
16 365 D u G
17 52 N u G
18 9 Ž od M
19 60 M u S
20 23 P U u LJ T
21 64 P na Š T
Zaboravila sam da dodam zaglavlje i 22. pitanje:
Moraš shvatiti šta znače slova. Pogledaj primer pod rednim brojem 0.
Prema MENSI ukoliko točno rešiš 19 ili više ti si "genije".
Samo dva člana MENSE uspela su rešiti tačno celi test.
Bodovi: 1 - 5 je prosek, 6 - 11 oko inteligentan, 12 - 18 stvarno inteligentan, 19 + Genije
Evo i poslednjeg:
22 100 G u V
Pored inteligencije moraš biti i dobro informisan da bi sve ovo znao i zbog toga ne verujem da je ovo Mensin test.
Viđala sam ovaj test i ranije, nije u redu da sad glumim neku pamet. Mada, ne mogu da se setim šta bi moglo da bude 5P na S i ono 23 PU u LJT (u ljudskom telu, verovatno)
I meni su ta dva predstavljala i problem. Sećam se da sam probala sa: 5 prstiju na shaci - no nije išlo. Ovo drugo nisam ni rešila, već su mi rekli šta treba da ide (prilično je bezveze, ali ako neko rešava, da mu ne kvarim zadovoljstvo).
Inače, ja sam ovo dobila od kumova kao poruku, sa tim podatkom da je Mensin test. A - da li je - ne znam.
E, da, bio je i onaj: 4S na SG, ali sam se posle setila.
(https://www.znaksagite.com/diskusije/proxy.php?request=http%3A%2F%2Fboingboing.net%2Fwp-content%2Fuploads%2F2011%2F10%2Fpizza.jpg&hash=9969e577901201b6d1185b0f3d676a09a7c1dbed)
0 24 S u D 24 sata u danu Tačno
1 30 S u A 30 slova u azbuci
2 7 D u N / dana u nedelji
3 7 S Č 7 svetskih čuda
4 12 Z u H 12 znakova u horoskopu
5 52 K u Š (B Dž) 52 karte u špilu (bez džokera)
6 4 S na S G 4 S na srpskom grbu
7 18 R na G I 18 rupa na golf igralištu
8 5 P na S 5 prstiju na šaci
9 90 S u P U 90 stepeni u pravom uglu
10 32 je T u S F na kojoj V S 32 je temperatura u stepenima farenhajta na kojoj voda smrzava
11 15 I u R T 15 igrača u ragbi timu
12 3 T na T 3 točka na trotinetu
13 11 I u F T 11 igrača u fudbalskom timu
14 12 M u G 12 meseci u godini
15 29 D u F u P G 29 dana u februaru u prestupnoj godini
16 365 D u G 365 dana u godini
17 52 N u G 52 nedelje u godini
18 9 Ž od M 9 života od mačke
19 60 M u S 60 minuta u satu
20 23 P U u LJ T 23 procenta ugljenika u ljudskom telu
21 64 P na Š T 64 polja na šahovskoj tabli
Quote3 T na T 3 točka na trotinetu
18 9 Ž od M 9 života od mačke
20 23 P U u LJ T 23 procenta ugljenika u ljudskom telu
3NG u GT - tri najveće gluposti u glupavom testu.
Ovo ne može da bude nikakav Mensin test jer prevuše kulturološki uslovljen.
Ma i star je kao biblija, napamet znam odgovore već.
Ta Mensa me mnogo nervira.
Šta god da je, test inteligencije svakako nije; eventualno može visoko da korelira sa nekim testovima inteligencije, ali i to je pitanje...
Ovo nije mensin test, naravno, ali je zgodna zajebancija.
Evo još nekih slatkih:
66 P ima B
1 V ima K
1 S ima Z
13 Z u K
8. Netačno.
Nemam pojma da li je Mensin test. Kuma je član Mense, verujem da je računala da ne može da joj odmogne, pa je otišla i prošla. (Od kumova sam i dobila pre nekoliko godina). Znam da mi je bilo zabavno, mada su i mene nervirali "životi od mačke").
Boban ima pogrešan odgovor na 8. pitanju, jer nije šaka u pitanju, no - stopalo.
A o testovima inteligencije uopšte, imam stav da su prilagođeni zapadnjačkom vrednovanju pameti. Negde sam pročitala da su Indijanci po tim merilima vidno gluplji, a ja se, naravno, uopšte ne slažem sa tim.
Padaju mi na pamet samo dva: (1 vime ima krava; 13 znakova u kartama).
Ćerka dodaje: 1 Sunce ima Zemlja, ali nemamo asocijaciju na broj 66.
66 poglavlja ima Biblija?
I - da li je Sunce ili satelit?
A ne verujem ni da je rešenje sa kravom valjano.
Moram kratko i brzo da šaljem, u više navrata, jer računar ima katatonični običaj da se zamrzne.
Quote from: Midoto on 01-11-2011, 21:43:49
Moram kratko i brzo da šaljem, u više navrata, jer računar ima katatonični običaj da se zamrzne.
Reinstaliraj operativni sistem, molim te. Ti upravljaš mašinom, ne ona tobom.
1 S ima Z - 1 sekiru ima Zagor!!1
evo jedan vrlo zanimljiv logicki problem...i vrlo tezak: http://www.varljiv.org/a/uzmi-ili-ostavi (http://www.varljiv.org/a/uzmi-ili-ostavi)
Da, želim da zamenim.
a je l znas zasto to zelis? :mrgreen:
Naravno. Zamisli da te je neko pitao, pre voditeljevog otvaranja prazne kutije - želiš li svoju kutiju da zameniš za one dve. Praktično se svodi na to.
Wow, vrlo elegantno i jasno objašnjenje. Upravo si mi preuzela vođstvo u sažetosti, dato od Jevtropijevićke na radionici.
Hvala. Ali i dalje mislim da bi na radionici mogao malo da se razmašeš u davanju komentara.
verovatnoća je čudo.
Ima onaj čuveni zadatak: koliko ljudi je potrebno na gomili da bi dvoje po zakonu verovatnoće imali rođendan istog dana?
Quote from: Boban on 23-12-2011, 16:18:09
verovatnoća je čudo.
Ima onaj čuveni zadatak: koliko ljudi je potrebno na gomili da bi dvoje po zakonu verovatnoće imali rođendan istog dana?
.....a da nisu blizanci ,pa koliko?
Quote from: Boban on 23-12-2011, 16:18:09
verovatnoća je čudo.
Ima onaj čuveni zadatak: koliko ljudi je potrebno na gomili da bi dvoje po zakonu verovatnoće imali rođendan istog dana?
znam da smo ovaj zadatak radili na faksu iz matematike u prvoj godini i asistent se opkladio s nekim u pice da ce da pogodi da je u grupi dovoljno ljudi sa istim datumom. Znaci nije neki preveliki broj...
probajte ovo:
Marija je prvi utorak provela u Beogradu, a prvi utorak posle prvog ponedeljka u mesecu provela je u Novom Sadu. Sledeceg meseca Marija je prvu sredu provela u Nisu, a prvu sredu posle prvog utorka u mesecu provela je na Zlatiboru. Gde je Marija te godine provela 8. mart?
Prvi mesec je počeo u utorak, a sledeći u sredu. To je moguće samo ako je prvi mesec prestupni februar. To znači da je prvog februara bio utorak, prvog marta bila je sreda, kao što je bila i osmog marta. 8. mart je proveden na Zlatiboru.
Ja imam drugi rezultat, jer nisam baratao sa brojevima. Prvi utorak ne mora da bude prvog u mesecu.
Quote from: mac on 13-04-2012, 20:23:00
Prvi mesec je počeo u utorak, a sledeći u sredu. To je moguće samo ako je prvi mesec prestupni februar. To znači da je prvog februara bio utorak, prvog marta bila je sreda, kao što je bila i osmog marta. 8. mart je proveden na Zlatiboru.
:|
Quote from: scallop on 13-04-2012, 20:32:39
Ja imam drugi rezultat, jer nisam baratao sa brojevima. Prvi utorak ne mora da bude prvog u mesecu.
prvi utorak posle prvog ponedeljka je nastupio nedelju dana nakon prvog utorka, a prvom utorku nije prethodio prvi ponedeljak, što znači da je utorak zaista prvi dan u mesecu.
Izvinjavam se. I ja sam dobio Zlatibor, ali mi nije bilo potrebno da otkrivam prestupni mesec. Beograd, Novi Sad i Niš ne mogu da budu osmog u mesecu pod zadatim uslovima. Zlatibor jedini može.
evo jedna zadaća koja spada u domen lateralnog razmišljanja: There are six eggs in the basket. Six people each take one of the eggs. How can it be that one egg is left in the basket?
Poslednja osoba je uzela jaje zajedno sa košarom.
Manite se tih problemčića za osnovce, evo nešto ozbiljnije:
(https://www.znaksagite.com/diskusije/proxy.php?request=http%3A%2F%2Fi44.tinypic.com%2F2emeozn.jpg&hash=32893a1ed128b9f6114876e237ba3ba71ae9fa60)
Jel' ova linija iznad broja beše znači da je to ispod linije jedan broj?
Quote from: mac on 18-04-2012, 16:05:32
Jel' ova linija iznad broja beše znači da je to ispod linije jedan broj?
Da. Dvadesetocifren broj čije neke cifre nisu poznate.
Treba valjda iskoristiti da taj broj mora biti djeljiv sa 23, 3 i 37 (111=3*37), pa na osnovu toga odrediti zbir nepoznatih cifara.
E sad, ne mogu da skontam kako. Nema nekih zgodnih kriterijuma za određivanje da li je broj djeljiv sa 23 ili 37. Pogotovo nema za 23. Za 37 bi se možda moglo iskoristiti da je 1000 = 1 (mod 37), pa ako mu cifre izdjelimo na grupe od po tri, počev od cifre jedinica, dobijemo da je broj djeljiv sa 37 akko je zbir tih trocifrenih brojeva iz grupa djeljiv sa 37. Npr., broj 12345678 daje isti ostatak pri dijeljenju sa 37 kao i broj 12+345+678.
A da rastaviš broj u vidu a*10^19 + a*10^18 + b*10^17... pa onda da grupišeš ovo sa a, b, c, d i da se preračunaju i saberu ove poznate cifre, pa se dobija u vidu zbira brojeva zavisnih od a,b,c,d i jedan poznat broj i onda dalje da se mulja sa deljivošću 3, 23, 27...
Ja sam počeo nešto ovako, al me smorilo da rastavljam na 20 činilaca pa sam batalio :)
Poslednja cifra b može biti 3, 9, 7 ili 1 (zbog množenja 23 i 111). Takođe zbir cifara ovog broja mora biti deljiv sa 3, a on iznosi 50+2(a+b+c+d)+b, pa bi možda trebalo iskoristiti ovo i ono što kaže Ygg :idea:
Quote from: Ygg on 18-04-2012, 14:00:20
Manite se tih problemčića za osnovce, evo nešto ozbiljnije:
Ne zanimaju me matematički problemi s formulama.
sada videh da je ovo zadatak za prvi razred srednje škole :cry:
Quote from: Mouchette on 18-04-2012, 20:56:33
Poslednja cifra b može biti 3, 9, 7 ili 1 (zbog množenja 23 i 111). Takođe zbir cifara ovog broja mora biti deljiv sa 3, a on iznosi 50+2(a+b+c+d)+b, pa bi možda trebalo iskoristiti ovo i ono što kaže Ygg :idea:
50 il 48?
Ako je za prvi srednje onda stvarno sramota :)
:lol: 48! :oops: xrofl
Da, zadatak je za prvi srednje. Državno takmičenje u Srbiji. Zato mislim da rješenje ne bi trebalo biti previše komplikovano.
Inače,
23^11 * 111^2 = 11739569027257494567,
i to je valjda jedina mogućnost, pa je
a=1
b=7
c=5
d=9
što znači da je a + b + c + d = 22.
To je rješenje koje se treba dobiti. Ali pretpostavljam da nisu očekivali da učenici rješe zadatak isprobavanjem raznih kombinacija i računanjem sa ovim ogromnim brojevima. :)
Otkud znaš da su ga na taj način rešili, mislim isprobavanjem?
mislim da je Ygg došao do rešenja tako, zar ne?
Da, ja sam tako došao do rješenja koristeći kalkulator. Ali to je varanje. :mrgreen:
Koristeći kongruencije i ovo što je napisala Mouchette, dobijem:
a, b, c, d su cifre (0-9), s tim da je a različito od nule;
b je 1, 3, 7 ili 9;
a + c + d = 0 (mod3);
7a + 15b + 3c + 11 = 0 (mod23) ili, ekvivalentno tome 7a - 8b +3c +11 = 0 (mod23);
11a + 28b + 36c + 11d +32 = 0 (mod 37) ili 11a - 9b - c + 11d - 5 = 0 (mod37).
Kako rješiti ovaj sistem kongruencija? Nadam se da se ne koristi kineska teorema o ostacima ili tako neka teška artiljerija, jer sam to zaboravio.
Quote from: Ygg on 18-04-2012, 23:18:17
Da, ja sam tako došao do rješenja koristeći kalkulator. Ali to je varanje. :mrgreen:
to se zove potpuno pretraživanje :lol:
Quote from: Ygg on 18-04-2012, 22:33:41
To je rješenje koje se treba dobiti. Ali pretpostavljam da nisu očekivali da učenici rješe zadatak isprobavanjem raznih kombinacija i računanjem sa ovim ogromnim brojevima. :)
Iz ovoga sam skontao da imaš samo rešenje, ali ne i postupak, kao i da imaš informaciju da su klinci rešavali samo ovim putem. Pa sam zato pitao odakle ti te informacije.
A sumnjam da treba uz pomoć mod da se rešava, ja ne mogu da se setim da sam to radio u srednjoj.
Ygg, nemam baš mnogo vremena (gledam poslednje epizode prve sezone serije Akta Manniskor - odlična je!), ali mislim da si na dobrom tragu što se tiče deljivosti sa 37. Ako dati broj izdelimo na po 3 cifre onda je zbir tako dobijenih brojeva deljiv sa 37, a vidi se da je taj zbir u stvari 1290 +102 b + 11(a+d) +10c (ako se opet nisam obrukala u sabiranju xrotaeye ). Ako sad iskoristimo da b može biti 3,9, 7 ili 1 onda se izbor znatno sužava. Npr. za b=3 dobija se 1596+11(a+d)+10c =0 (mod37), pa je 11(a+d)+10c=5 (mod37), što daje ograničen izbor (32, 69,106,143...) pa se mogu odabrati a,d i c, ali tako da je a+c+d=0 (mod3). Ja sam recimo dobila da je dobar samo izbor a+d=2, c=1, ali onda treba proveriti i ostale uslove koje si naveo. E sad to treba uraditi i za ostale tri vrednosti za b. Sve u svemu, relativno zamoran (ali konačan) posao :( Ali Akta Manniskor! Zar samo deset epizoda? :cry:
Radila bih po modulu 11: jer i 23 i 111 daju ostatak 1, pa i ceo broj mora da ima ostatak 1. Kad sabereš cifre na parnim i neparnim mestima, dobiješ direktno da je b sedmica, a time dobiješ i da je stepen nad 23 jednak 4k - 1. Nastaviću zadatak, ali do sada mi pada na pamet nekoliko ideja: dokazati da y ne može da bude 1 - onda je ceo broj deljiv najmanje sa 9; izračunati koliko mora da bude 2(a + c + d) zbog deljivosti s 9 i uzeti u obzir da ne može da zbir te tri cifre ne može biti veći od 27...
Hvala Midoto! To je vjerovatno prava ideja, mada još nisam uspio da riješim zadatak do kraja, tj. da odredim ostale tri cifre. Mučio sam se oko nekih drugih zadataka, pa nisam ni stigao da se bakćem sa ovim. :)
Usput, jedan simpa strip sa Spikedmath-a (http://spikedmath.com/):
(https://www.znaksagite.com/diskusije/proxy.php?request=http%3A%2F%2Fimg.spikedmath.com%2Fcomics%2F508-tired-of-adding-C2.png&hash=55a47c6e53d6dc91d060073e1d92a12df934fab3)
Ја сам својевремено користио ознаке типа Ž, Ж, Ć и слично.
nastaviti niz:
2, 9, 10, 12, 19, 20...?
ko reši ovo skidam kapu...
21
Svi brojevi počinju na slovo D
22 29 30 32 39 40 42 49 50 :-x
@Skalar: 52
Za Ghoula:
0, 1, 1, 2, 3, 5, 8, 13, 21, 34, 55, 89, 144, ...
Ovaj niz ima i naziv...
Quote from: smp on 22-06-2012, 16:31:45
@Skalar: 52
Za Ghoula:
0, 1, 1, 2, 3, 5, 8, 13, 21, 34, 55, 89, 144, ...
Ovaj niz ima i naziv...
ovo je prelako, daj nešto drugo.
Quote from: Milosh on 22-06-2012, 15:41:11
21
Svi brojevi počinju na slovo D
Tačno, je l ti poznata fora ili si se sam setio?
Quote from: Truman on 22-06-2012, 15:37:39
nastaviti niz:
2, 9, 10, 12, 19, 20...?
ko reši ovo skidam kapu...
Мој Маре је дао сасвим валидно другачије решење:
2, 9, 10, 12, 19, 20, 22, 29, 30, 32, 39, 40... итд
:|
Проблем са тим задацима типа "наставите низ" је што сваки има бесконачно много рјешења.
Мислиш, може да се тера унедоглед или постоје алтернативне варијанте? Ако је ово друго, мислим да низови који се настављају по неком строго математичком принципу не могу имати алтернативна решења, рецимо горе споменути Фибоначијев. Добро, можда зависи од тога колико елемената низа даш у поставци. Али углавном алтернативна решења постоје у зезалицама као што је ова са бројевима који почињу на слово Д.
Quote from: Джон Рейнольдс on 22-06-2012, 22:46:40
Quote from: Truman on 22-06-2012, 15:37:39
nastaviti niz:
2, 9, 10, 12, 19, 20...?
ko reši ovo skidam kapu...
Мој Маре је дао сасвим валидно другачије решење:
2, 9, 10, 12, 19, 20, 22, 29, 30, 32, 39, 40... итд
:|
Твој Маре и ја смо на исти начин решили, с тим што сам ја гурао до 50 и стао, јер је већ било очито на који начин се низ наставља. И то је потпуно валидно решење! Разлика између Марета и мене је, верујем, у 40-ак година. Клинац ти је веома интелигентан. Добро за њега, а родитељима честитке.
Ових пет низова је Маре осмислио за своје другаре, а за потребе верске наставе. :lol: Односно, вероучитељ дозвољава интелектуална надметања, смишљање задатака, итд. Очигледно је да су низови све тежи и тежи, све је Маре узео у обзир. 8)
(https://www.znaksagite.com/diskusije/proxy.php?request=http%3A%2F%2Fimg23.imageshack.us%2Fimg23%2F3011%2F20120622221336.jpg&hash=c5c299065ebd3d299229298f69d1e697ca5495bb) (http://imageshack.us/photo/my-images/23/20120622221336.jpg/)
Каже да су клинци имали највише проблема са четвртим задатком.
едит: Тражи се даљих елемената колико има цртица. Логично. Избегавају се алтернативна и крња решења.
I meni je četvrti zadao najviše muke. :lol: Mislim, smislio sam jedno rešenje, ali nešto mi govori da to ne mora biti ono na oje je Mare mislio...
je l četvrti 3330?
Quote from: Джон Рейнольдс on 22-06-2012, 22:46:40
Quote from: Truman on 22-06-2012, 15:37:39
nastaviti niz:
2, 9, 10, 12, 19, 20...?
ko reši ovo skidam kapu...
Мој Маре је дао сасвим валидно другачије решење:
2, 9, 10, 12, 19, 20, 22, 29, 30, 32, 39, 40... итд
:|
da li se uopšte može govoriti o nizu u kojem svi elementi počinju na slovo D? mislim, jel to niz ili skup elemenata koji imaju nešto uajedničko?
Quote from: tomat on 23-06-2012, 01:30:04
je l četvrti 3330?
I ja pomislih isto, ali nula ne bi smela da bude na kraju.
Quote from: tomat on 23-06-2012, 01:30:04
je l četvrti 3330?
Јесте.
Него, занимљиво питање то за низ са бројевима на слово Д. Рецимо, сад сам смислио овај низ:
2, 6, 1, 4, 50, 12, 11...
Да видим следећи елемент. :lol: Можда сам се негде и зезнуо, али идеја је ионако бесмислена.
едит: Да, нула јесте на крају. Нула је "граничник". После би било, рецимо, 44440.
Sa nulom počinje, ne može da se završava sa nulom. Nije simetrično. Svaka nula ima pripadajuće cifre (1, 22, 333). Da je nula graničnik, niz bi glasio ovako: 10220333044440 itd.
Ако почиње с нулом, не може да се заврши са нулом јер... није СИМЕТРИЧНО?
Која си ти глупача, ајде шетај с теме, леба ти. Моје седмогодишње дете је за тебе већ недостижни ниво. Гледај:
|1|22|333|4444|
Јасније? Симетричније? xrofl
pa niz se ne završava nulom, on može da se nastavi u nedogled. dečko je tražio naredna 4 elementa, a dao je dovoljno podataka da se može izvesti zakonitost za nastavljanje niza.
edit: ulete Džon
Ne. 4 crte zbunjuju i varaju. Nula nije graničnik, i svaki sastavljač ovakvih nizova, recimo za mensine testove (mada je ovo prelak niz), objasnio bi vam da nula ovde nije graničnik, već početak svakog novog mikro niza.
Može da stoji nula, ali tako se ne postavlja zadatak sa nulom kao graničnikom. Dala sam primer kako se piše niz sa nulom graničnikom.
Занима ме може ли неко да реши мој идиотски низ и да ли такве ствари (као уосталом и задатак са бројевима на Д) имају икакву логичку тежину, што је Томат питао... Да ли се уопште ту може говорити о низу?
Негде се на форуму појавила гомила задатака са бројевима и словима типа: "10 М И", па је решење - рецимо - 10 малих индијанаца. По мом мишљењу, потпуна глупост, ал ајде...
Quote from: Джон Рейнольдс on 23-06-2012, 02:01:01
Занима ме може ли неко да реши мој идиотски низ и да ли такве ствари (као уосталом и задатак са бројевима на Д) имају икакву логичку тежину,
Takvi nizovi (početak na D, na primer) ne mere matematičku inteligenciju, već neku sasvim drugu... Legitimni su, mada varaju u smislu da te namerno navlače da računaš i tražiš matematičko rešenje. Ali jedno kada uđeš u štos i shvatiš koje sve "logike" treba da koristiš i u kojim pravcima da misliš, nema nerešivog zadatka.
Онда изволи, реши низ који сам задао.
Quote from: D. on 23-06-2012, 02:04:42
Quote from: Джон Рейнольдс on 23-06-2012, 02:01:01
Занима ме може ли неко да реши мој идиотски низ и да ли такве ствари (као уосталом и задатак са бројевима на Д) имају икакву логичку тежину,
Takvi nizovi (početak na D, na primer) ne mere matematičku inteligenciju, već neku sasvim drugu... Legitimni su, mada varaju u smislu da te namerno navlače da računaš i tražiš matematičko rešenje. Ali jedno kada uđeš u štos i shvatiš koje sve "logike" treba da koristiš i u kojim pravcima da misliš, nema nerešivog zadatka.
to 10 m.i. je igra asocijacija, a u onome sa D se traži da nastaviš niz, i rešenje koje su dale Mare i Skalar je zaista niz (ima ovde matematičara na forumu, može li neko da pomogne sa definicijom niza), a brojevi koji počinju sa D ne čine nikakav niz. mada, možda grešim.
Quote from: tomat on 23-06-2012, 02:09:40
D ne čine nikakav niz. mada, možda grešim.
Pa u tome i jeste štos. Navlače te da misliš da je niz, a ono - nije, već ima neku sasvim drugu logiku.
Quote from: Джон Рейнольдс on 23-06-2012, 02:09:31
Онда изволи, реши низ који сам задао.
Nisam ni videla da si zadao. Šta ja znam koje šizoidno rešenje se vrti po tvojoj glavi. 8-)
Quote from: D. on 23-06-2012, 02:14:07
Pa u tome i jeste štos. Navlače te da misliš da je niz, a ono - nije, već ima neku sasvim drugu logiku.
onda je to potpuna nebuloza, obzirom da je eksplicitan zahtev bio na se
nastavi niz.
Slažem se. Prevara.
Quote from: D. on 23-06-2012, 01:57:00recimo za mensine testove
recimo
xrofl
Quote from: Джон Рейнольдс on 23-06-2012, 01:48:16
2, 6, 1, 4, 50, 12, 11...
Evo ti rešenje: svaki broj se povećava za jedan u slovima (kada se napiše). DVA (3 slova), šest (4 slova), jedan (5 slova), četiri (6 slova), pedeset (7 slova), dvanaest (8 slova), jedanaest (9 slova). Sledeći broj u nizu je onaj koji se piše sa 10 slova (prvi sledeći sa 10 slova, mrzi me da ga tražim).
Ali, na primer: 1006
Ajde ti sada reši ovaj (lako je): 29, 18, 16, 11, 7, 21, 15, 1, 20, _, _. :lol:
Quote from: D. on 23-06-2012, 02:30:36
Quote from: Джон Рейнольдс on 23-06-2012, 01:48:16
2, 6, 1, 4, 50, 12, 11...
Evo ti rešenje: svaki broj se povećava za jedan u slovima (kada se napiše). DVA (3 slova), šest (4 slova), jedan (5 slova), četiri (6 slova), pedeset (7 slova), dvanaest (8 slova), jedanaest (9 slova). Sledeći broj u nizu je onaj koji se piše sa 10 slova (prvi sledeći sa 10 slova, mrzi me da ga tražim).
Ali, na primer: 1006
Браво! Видиш да није шизоидно. :lol: Следећи у низу је 17, пошто су увек најнижи бројеви с датим бројем слова у речи.
A možda samo oboje mislimo šizoidno. Pre će biti.
Ne znam koliko je šizoidno ali nije matematički niz. Offtopic! 8)
Заиста ме занима оно што је Томат питао, ево размишљам нешто... На овим примерима низова базираним на словима речи бројева (аух...) види се да се реч "низ" може употребити само у смислу - иду неки бројеви у низу, као што су бемлига, неке перле нанизане на огрлици. Јасно, то математички низови нису, не функционишу ван српског говорног подручја... Како их дефинисати? Ставити их у ранг главоломки, ребуса, итд?
To su logički zadaci. Nije u pitanju matematika, ali jeste logika. I jeste niz, samo ne matematički (u strogom smislu).
Ovaj Džonov zadatak može da se podvede pod matematički i ne mora. On meša dva nivoa rešavanja zadatka: jezički i matematički.
Quote from: D. on 23-06-2012, 02:37:32
Ajde ti sada reši ovaj (lako je): 29, 18, 16, 11, 7, 21, 15, 1, 20, _, _. :lol:
29, 18, 16, 11, 7, 21, 15, 1, 20, 1, 28 :mrgreen:
A kad smo kod matematičko-jezičkih glavolomki, evo još jedne za osnovce:
QuoteSledeće dve rečenice očigledno su tačna tvrđenja:
Ova rečenica sadrži pet reči.
Ova rečenica sadrži trideset tri slova.
Zadatak. Na prazna mesta upiši slovima dva broja tako da se dobije tačno tvrđenje:
U ovoj rečenici ima ________________ reči i ______________ slova.
Quote from: Ygg on 23-06-2012, 14:46:43
Quote from: D. on 23-06-2012, 02:37:32
Ajde ti sada reši ovaj (lako je): 29, 18, 16, 11, 7, 21, 15, 1, 20, _, _. :lol:
29, 18, 16, 11, 7, 21, 15, 1, 20, 1, 28 :mrgreen:
Bravo, ali nemoj da me odaš Džonu. :evil:
Quote from: Ygg on 23-06-2012, 14:46:43
Zadatak. Na prazna mesta upiši slovima dva broja tako da se dobije tačno tvrđenje:
U ovoj rečenici ima ________________ reči i ______________ slova.
Devet i četrdeset.
4 13 24 19 18 21 22 10 !!! :roll:
Eh, sad. Ygg ti je rešio zadatak, džabe ti sad.
Мислиш, јавио ми приватном поруком кључ за решавање основношколске шифре за љубавна писма? xrofl
Pa nije težina zadatka bila cilj, nego poruka. Ako hoćeš, smisliću ti pravi zadatak.
Ne znam o čemu ti to kada pričaš o privatnim porukama. Prosvetli javnost, da ne pomisli svašta. 8-)
Quote from: D. on 23-06-2012, 11:10:39
To su logički zadaci. Nije u pitanju matematika, ali jeste logika.
nije ni logika
a ni naslov topika ne spominje logiku
Začudna je tvoja potreba da na ovom topiku ostaviš reč-dve, a da nijedna nema veze sa rešenjem zadataka koji se ovde postavljaju. Ali ne čudi što uvek tvrdiš da nešto nije, a pojma nemaš šta jeste.
Ај полако, бре. Недељно је јутро, прво кафица и једна домаћа љута, па јак доручак, потом нос напоље на сунце, да се удахне врелине, па припреме за брчкање папака у мору, реци, језеру, базену, локви, лавору, шта већ ко има у близини, а тек увече, кад залади, ајмо почети гађати пикадо стрелицама.
xcheers
Quote from: D. on 24-06-2012, 10:00:18
Ali ne čudi što uvek tvrdiš da nešto nije, a pojma nemaš šta jeste.
a ovo nije ni ono što jeste, već ono što ima. Nemoj da se dižeš previsoko, ipak je pitanje onoga što jeste preteško za tebe.
Ko nema u glavi, ima u trtljanju.
drago mi je što si se sama svrstala u trtljanje
Ko nema svoje fore, otima tuđe.
drago mi je da si napokon priznala da mi kradeš fore već godinama
Ko nema u duhovitosti, ima u smaranju.
drago mi je da si napokon priznala da si smaračica
Ko nema u imaginaciji, nema ni u penisu.
drago mi je da si napokon priznala da nemaš penis
Nema problema. Moram da ti pružim podršku.
The Simpsons - All Syrup Super Squishy Bender (http://www.youtube.com/watch?v=61kHpmenkT8#)
mom penisu nije potrebna tvoja podrška
Nema penisa, nema podrške.
jel to spada u trtljanje, duhovitost, ontopičnost ili u prosto laprdanje?
Pošto ostaje i dalje fakat da onaj niz nije ni matematički ni logički, kao i da si ti prilično glupa.
Stvarno se nadam da penis ima neke veze sa tim, onda ti je dijagnoza bar jasna i precizna.
Zar se ovde ne testira Ghoulovo znanje matematike? :roll:
Da si ovde duže, ne bi postavljao besmislena pitanja.
Nadam se da razumeš da si upravo napala čoveka. Možda će sad on poželeti da se brani. Možda ćeš onda ti poželeti da mu objasniš gde mu je mesto. Bah, već viđeno.
Quote from: mac on 25-06-2012, 12:25:03
Nadam se da razumeš da si upravo napala čoveka.
nadamo se, svi se nadamo xrofl
Nadam se da znaš da si opet bio subjektivan. Nadam se da znaš da je on prvi poželeo nekome da objasni gde mu je mesto. I da znaš da sam ja poželela da se branim. Bah, već viđeno.
pa da, samo što se postavlja pitanje zašto si se baš ti, cvijet bez lažnog morala, morala braniti, zašto ne npr. ja? Moj je post iznad njegovog, mogao sam i ja tako da se branim, ali čekaj malo, pa ja nisam pacijent, u tome je štos :idea:
xrofl
Znam da ti je teško što ispadaš budala svaki dan, ali valjda ćeš se navići vremenom.
xrofl
Quote from: D. on 25-06-2012, 12:27:33
Nadam se da znaš da si opet bio subjektivan. Nadam se da znaš da je on prvi poželeo nekome da objasni gde mu je mesto. I da znaš da sam ja poželela da se branim. Bah, već viđeno.
Jbg, ono što ja vidim je da čovek želi na matematičkoj temi da vidi matematiku. Nikoga nije napao, nije išao ad hominem, nego je iskazao svoju frustriranost. Neka je i hteo da objasni čemu je mesto na matematičkoj temi, bar je govorio o mestu temi, a ne mestu ljudima.
Čini ti se samo. :) Od toliko topika, gde ljudi toliko troluju, svađaju se, skreću sa teme, ovaj je našao nas ovde da uči o čemu je tema.
(https://fbcdn-sphotos-a.akamaihd.net/hphotos-ak-ash4/282386_454712771213460_201321498_n.jpg)
Nisam imao nameru da delim lekcije bilo kome ovde.
Tema topika mi je bila zanimljiva, kao i neki postovi.
Napišem li priču za radionicu, udrite... xtwak
Najefikasniji način da vratiš temu u njene okvire je da postaviš zadatak, a ne pitanje.
U deset kovčega bilo je u svakom po deset zlatnika.
U svim kovčezima bili su zlatnici od po 10 grama, osim u jednom gde su svi zlatnici bili od po 9 grama.
Imate vagu i treba samo jednim jedinim merenjem da ustanovite u kom su kovčegu lakši zlatnici.
I još jedan:
U malom avionu koji je putovao od Čikaga do Dalasa našla su se tri čoveka: Džejson, Kermit i Luis.
Dvojica su članovi Mense, dvojica nose brade, dvojica gaje orhideje, a dvojica su iz Čikaga.
Nikoga ne karakterišu više od tri pomenute osobine.
Ako je Džejson član Mense, on je iz Čikaga.
Ako kermit nosi bradu, on gaji orhideje. Isto se može reći i za Luisa.
Ako je Džejson iz Čikaga, on gaji orhideje. Isto se može reći i za Luisa.
Pitanje: ko je od njih rođen u Dalasu?
Dobro došao
smp! Ne obraćaj pažnju na
D. i
Бату, oni su dežurni trolovi na ovom forumu. :lol:
Quote from: smp on 25-06-2012, 15:12:07
U deset kovčega bilo je u svakom po deset zlatnika.
U svim kovčezima bili su zlatnici od po 10 grama, osim u jednom gde su svi zlatnici bili od po 9 grama.
Imate vagu i treba samo jednim jedinim merenjem da ustanovite u kom su kovčegu lakši zlatnici.
Ovo je standardan matematički problem, za dodatnu u osnovnoj školi. Znam rješenje, ali neću odmah odgovoriti, neka ostali malo razmišljaju.
Quote from: smp on 25-06-2012, 15:33:37
U malom avionu koji je putovao od Čikaga do Dalasa našla su se tri čoveka: Džejson, Kermit i Luis.
Dvojica su članovi Mense, dvojica nose brade, dvojica gaje orhideje, a dvojica su iz Čikaga.
Nikoga ne karakterišu više od tri pomenute osobine.
Ako je Džejson član Mense, on je iz Čikaga.
Ako kermit nosi bradu, on gaji orhideje. Isto se može reći i za Luisa.
Ako je Džejson iz Čikaga, on gaji orhideje. Isto se može reći i za Luisa.
Pitanje: ko je od njih rođen u Dalasu?
Luis je rođen u Dalasu! Mada, pitanje nije baš precizno. Zaključi se da su Džejson i Kermit iz Čikaga, pa dakle Luis nije iz Čikaga, što ne mora značiti da on nije iz nekog trećeg mjesta. A i pitanje je ko je
rođen u Dalasu. Ako su ova dvojica iz Čikaga, ne mora značiti da nisu Rođeni u Dalasu a da sad žive u Čikagu. xnerd
Ah, jeste, a on nije trol? Tek na moje uputstvo šta treba da radi da ne bi bio trol, postavio je zadatke.
Vidiš, ja dobijam rešenje da je Kermit iz Dalasa.
Džejson: nosi bradu, iz Čikaga, gaji orhideje, član je Mense
Kermit: član je Mense, iz Dalasa
Luis: nosi bradu, gaji orhideje, iz Čikaga
Ovako postavljene osobine ispunjavaju sva tri uslova. S tim što i Luis ladno može da bude član Mense, ako to Kermit nije.
...
A ovo za zlatnike: nije dozvoljeno da dodajemo zlatnik po zlatnik na vagu, ne skidajući prethodne? Ja tu uopšte ne bih koristila vagu. :-|
Mora se napomenuti da vaga nije ona sa dva tasa, za koju imaš na raspolaganju samo šta je zadatkom dato, nego ona što meri tačnu težinu.
Ovaj prvi znam, drugi ne uspevam da uklopim (D, tebi Džejson ima 4 osobine, a ne sme više od 3).
Ygg, na osnovu čega si ti zaključio da je Kermit iz Čikaga? A i rekao bih da kod Amera ''from Chicago'' znači ''born in Chicago''. Ali rekao bih da nešto nedostaje u samom pitanju, jer ovako je nekako prelako.
Ovo sa mjerenjem kovčega zlatnika jedan jedini put mi je besmisleno, osim ako ne misle pod jednim mjerenjem da ih nasađuješ jedan na drugi pa da vaga pokazuje 100, 200, 300 i eventualno npr 590.
Eh, da, nisam računala mesto odakle su kao osobinu, ali sada kada ponovo pročitam zadatak, tako je. Ne mora svako da ima tri, nego do tri osobine.
Onda je rešenje sledeće:
QuoteAko je Džejson član Mense, on je iz Čikaga.
Ako kermit nosi bradu, on gaji orhideje. Isto se može reći i za Luisa.
Ako je Džejson iz Čikaga, on gaji orhideje. Isto se može reći i za Luisa.
Ali opet mi ne ispada Luis.
Džejson: nosi bradu, iz Dalasa,
Kermit: član je Mense, iz Čikaga
Luis: nosi bradu, gaji orhideje, iz Čikaga
Quote from: Бата Животиња on 25-06-2012, 20:21:11
Ygg, na osnovu čega si ti zaključio da je Kermit iz Čikaga?
Ne Kermit, nego Luis. Ygg tvrdi da je Luis iz Dalasa.
Dvojica gaje orhideje, fali ti jedan.
Bato: jedan novčić iz prvog kovčega, iz drugog dva, iz trećeg tri... Ako fali 1 gram u prvom kovčegu su lakši, ako fale 2 u drugom, itd.
To je rešenje. Zadatak ne valja. :lol:
angel, niko nije rekao da može da se vadi nešto iz kovčega, inače je ok rješenje
Quote from: D. on 25-06-2012, 20:31:41
Quote from: Бата Животиња on 25-06-2012, 20:21:11
Ygg, na osnovu čega si ti zaključio da je Kermit iz Čikaga?
Ne Kermit, nego Luis. Ygg tvrdi da je Luis iz Dalasa.
moje pitanje je jasno da jasnije ne može biti.
Pa na isti način na koji je zaključio i da je Luis iz Dalasa.
dobro, ajd sad prošetaj
Quote
Ako je Džejson član Mense, on je iz Čikaga.
Ako kermit nosi bradu, on gaji orhideje. Isto se može reći i za Luisa.
Ako je Džejson iz Čikaga, on gaji orhideje. Isto se može reći i za Luisa.
Ako je Luis iz Dalasa, Džejson jeste iz Čikaga, pa je i Kermit iz Čikaga.
To znači da Džejson gaji orhideje, kao i Luis. I Luis nosi bradu jer gaji orhideje.
Džejson je član Mense, jer jeste iz Čikaga.
Kermit ne nosi bradu jer ne gaji orhideje, to znači da Džejson nosi bradu, jer dvojica moraju da je nose. To su već četiri osobine za Džejsona.
Dakle, nije ni Luis.
Zadatak ima manu.
Quote from: Бата Животиња on 25-06-2012, 20:21:11
Ygg, na osnovu čega si ti zaključio da je Kermit iz Čikaga?
Komplikovano je za objasniti. Trebalo bi puno pisati a mrzi me da pišem. :mrgreen:
Ja dobijem ovako:
Mensa brada orhideje Čikago
Džejson - + + +
Kermit + - - +
Luis + + + -
Propustio si ovo, Ygg:
QuoteAko je Džejson član Mense, on je iz Čikaga.
Džejson je član Mense.
Zadatak ima manu, tačnije - jedan od ove trojice ima jednu umesto dve od nekih osobina. Samo tako može da se reši.
Poenta je u ''Ako'', a ja ne vidim zašto prvo ''Ako'' nije tačno, djeluje mi kao bačeno ofrlje + bih volio objašnjenje po kojem svi imaju po tri osobine
Quote from: D. on 25-06-2012, 20:56:21
Propustio si ovo, Ygg: QuoteAko je Džejson član Mense, on je iz Čikaga.
Džejson je član Mense.
Ako je Džejson član Mense, on je iz Čikaga. Obrnuto ne mora da vrijedi. Tj. može se desiti da Džejson nije član Mense a da jeste iz Čikaga. Matematički rečeno, u pitanju je implikacija a ne ekvivalencija. xnerd
Ne pada mi na pamet nikakav zgodan kontraprimjer iz života; jedino matematički. :) Npr. rečenica "Ako se broj n završava nulom, onda je on djeljiv sa 5." je tačna, jer svaki broj koji se završava nulom djeljiv je sa 10, pa samim tim i sa 5. Međutim, obrnuto očigledno ne mora da vrijedi: broj 25 je djeljiv sa 5 a ne završava se nulom.
Eh, da, da. Onda možemo zadatak i ovako da čitamo: "isto se može reći i za...". Može, ali ne mora, jel. Pa onda ima rešenja koliko hoćeš. :roll:
nešto mora biti netačno jer ne mogu svi da gaje orhideje. Mene samo zanima zašto je Kemit taj koji ne gaji i zašto je iz Čikaga
Quote from: Бата Животиња on 25-06-2012, 21:08:22
nešto mora biti netačno jer ne mogu svi da gaje orhideje. Mene samo zanima zašto je Kemit taj koji ne gaji i zašto je iz Čikaga
Ja sam nacrtao tabelu, kao ovu iznad što sam okačio i popunjavao polako plusevima i minusima. Ispočetka ima više mogućnosti, ali se one jedna po jedna eliminišu i na kraju ostaje jedna jedina kombinacija, ova koju sam okačio.
Pokušaću malo kasnije večeras da iskucam kako sam došao do rješenja, sad sam u nekoj frci.
Jasno je i ovako, sa tabelom. Mene je zeznulo to što me je mrzelo da crtam. :lol:
Zadatak koji zavisi od intepretacije uslova, nije zadatak.
Quote from: Ygg on 25-06-2012, 21:12:32
Ispočetka ima više mogućnosti, ali se one jedna po jedna eliminišu i na kraju ostaje jedna jedina kombinacija, ova koju sam okačio.
to mi onda ne liči na logički zadatak, to ti je kao da lupamo šta treba da stoji u određenom polju sudokua pa ako ne ispadne kako treba onda idemo sve iz početka. Mora da postoji neki ključ koji je siguran.
Zez sa ovakvim zadacima je što po automatizmu podrazumevamo neke stvari koje uopšte nisu navedene kao uslov.
Quote from: Ygg on 25-06-2012, 16:33:31
Dobro došao smp! Ne obraćaj pažnju na D. i Бату, oni su dežurni trolovi na ovom forumu. :lol:
Quote from: smp on 25-06-2012, 15:12:07
U deset kovčega bilo je u svakom po deset zlatnika.
U svim kovčezima bili su zlatnici od po 10 grama, osim u jednom gde su svi zlatnici bili od po 9 grama.
Imate vagu i treba samo jednim jedinim merenjem da ustanovite u kom su kovčegu lakši zlatnici.
Ovo je standardan matematički problem, za dodatnu u osnovnoj školi. Znam rješenje, ali neću odmah odgovoriti, neka ostali malo razmišljaju.
Standardan je matematički problem jer postoji vekovima, ne sumnjam da ga zadaju u osnovnoj školi, može se on zadati i predškolskoj deci, međutim reč je o vrlo teškom problemu, na koga skoro niko ko ga čuje prvi put ne nađe odgovor, a čije je rešenje genijalno.
Quote from: Бата Животиња on 25-06-2012, 21:35:20
to ti je kao da lupamo šta treba da stoji u određenom polju sudokua pa ako ne ispadne kako treba onda idemo sve iz početka
Pa komplikovaniji logički problemi se obično tako i rješavaju. I sudoku na najtežem nivou je takav. U jednom momentu ne možeš da odrediš ni za jedno prazno polje koji broj treba da bude upisan u njega, već posmatraš više mogućnosti, pa tjeraš svaku od njih dok ne dođeš do kontradikcije u svim slučajevima, osim u jednom; to znači da je taj slučaj pravi.
No, da pokušam da ispišem kako sam došao do rješenja.
Ako pretpostavimo da je Džejson član Mense, onda je on iz Čikaga, a to znači da on gaji orhideje. Odatle slijedi da on nema bradu (jer nikoga ne karakterišu više od tri navedene osobine). Odatle zaključujemo da preostala dvojica imaju bradu (jer svaka osobina vrijedi za dvojicu od njih). Ako Kermit nosi bradu, on gaji orhideje, a ako Luis nosi bradu i on gaji orhideje takođe, pa dobismo da sva trojica gaje orhideje, što ne smije biti. Dakle, početna pretpostavka ne valja, tj. Džejson nije član Mense. To dalje znači da ostala dvojica jesu članovi Mense i time je prva kolona popunjena.
Ajmo sad na drugu kolonu. Pretpostavimo da Džejson nema bradu. Pošto onda Džejson ima već dva minusa, preostala dva znaka u njegovom redu su plusevi, tj. Džejson gaji orhideje i iz Čikaga je. (Moramo u svakom redu imati po barem dva plusa, jer imamo ukupno četiri osobine puta po dvije osobe, to je osam pluseva, a ni u jednom redu ne smiju biti više od tri plusa. Znači u jednom redu će biti dva plusa, a u preostala dva reda po tri plusa.) E sad, slično prošlom razmatranju, zaključujemo da Kermit i Luis imaju bradu, pa da i jedan i drugi gaje orhideje, pa opet dobijamo da sva trojica gaje orhideje. Kontradikcija. Dakle, Džejson mora imati bradu.
Za sad imamo popunjenu prvu kolonu i prvi znak u drugoj koloni. Sad možemo lako zaključiti da Džejson gaji orhideje, jer ako ne gaji orhideje, onda nije iz Čikaga (jer kad bi bio iz Čikaga onda bi morao da gaji orhideje xrotaeye ), pa bi u njegovom redu opet imali tri minusa.
Ako sad pretpostavimo da Kermit ima bradu, onda on gaji orhideje. Onda Luis nema bradu i ne gaji orhideje (jer druga dvojica imaju brade i gaje orhideje), pa zaključujemo da je on iz Čikaga (jer ne može imati tri minusa u svom redu). I eto opet kontradikcije, jer imamo da je Luis iz Čikaga, a da ne gaji orhideje. Dakle, Kermit nema bradu, a Luis ima, pa su nam sad popunjene prve dvije kolone i prvi znak u trećoj.
Dalje je lako. Pošto Luis ima bradu, on gaji orhideje, pa Kermit ne gaji orhideje (jer druga dvojica gaje). Popunjena i treća kolona. Sad vidimo da Luis nije iz Čikaga, jer ne može imati sva četiri plusa u svom redu, a onda druga dvojica jesu iz Čikaga. The end! :)
Бато, nadam se da ćeš ovo da iščitaš, da nisam džaba kucao. 8)
Quote from: Alex on 25-06-2012, 22:20:33
Standardan je matematički problem jer postoji vekovima, ne sumnjam da ga zadaju u osnovnoj školi, može se on zadati i predškolskoj deci, međutim reč je o vrlo teškom problemu, na koga skoro niko ko ga čuje prvi put ne nađe odgovor, a čije je rešenje genijalno.
U pravu si. Na to sam i mislio kad sam rekao da je standardan. Rješenje zadatka nije nimalo trivijalano.
Evo vam za sudoku :)
17 and Sudoku Clues - Numberphile (http://www.youtube.com/watch?v=MlyTq-xVkQE#ws)
Nije im los kanal (http://www.youtube.com/user/numberphile)
Evo još jedne logičke mozgalice. Zadatak je sa opštinskog takmičenja iz matematike, za prvi razred srednjih škola.
U vestima je data sledeća vremenska prognoza za sutra:
1. biće oblačno ili će padati sneg ili će duvati vetar;
2. ako bude oblačno sa snegom, duvaće vetar;
3. ako ne bude vetrovito, biće oblačno bez snega.
Da li se odatle može zaključiti da će, ako bude padao sneg, duvati vetar?
Quote from: smp on 25-06-2012, 15:12:07
U deset kovčega bilo je u svakom po deset zlatnika.
U svim kovčezima bili su zlatnici od po 10 grama, osim u jednom gde su svi zlatnici bili od po 9 grama.
Imate vagu i treba samo jednim jedinim merenjem da ustanovite u kom su kovčegu lakši zlatnici.
Iz svakog kovčega uzmemo drugi broj zlatnika (1, 2, 3...) i kad ih sve zajedno izmerimo gledamo koliko fali do okrugle cifre - ako je jedan gram onda je iz kovčega 1, dva grama iz kovčega 2 itd.
Ygg, shvatam ja to, samo sistemi eliminacije po meni nisu pravi logički problemi. Jednostavno, ne možeš logički problem da rješavaš po principu hajde da vidimo dokle će ovo da dogura pa ako dođemo u ćosokak probaćemo drugačiji pristup. To po meni nije logički problem, koji na svakom nivou zaključivanja ne može da pogriješi pa da se vraća nazad, kad je nivo pređen onda je pređen, tako se riješava logički problem.
Quote from: Бата Животиња on 25-06-2012, 23:32:28
Ygg, shvatam ja to, samo sistemi eliminacije po meni nisu pravi logički problemi.
xrofl
Drugim rečima, još ga nisi rešio :) Ovo rešavaju programeri, a ne filozofi :)
Odgovor je da.
EDIT: A Bata se raspravlja sa Yggom od ranije. Ja sam mislio na meteorološki problem.
Quote from: mac on 25-06-2012, 23:36:50
Ovo rešavaju programeri, a ne filozofi :)
EDIT: A Bata se raspravlja sa Yggom od ranije. Ja sam mislio na meteorološki problem.
mada si ovo i pogodio, ovo jeste neka vrsta programskog jezika, razbijanje šifrarnika sistemom eliminacije, ali logički problem (a logika je ipak filozofska disciplina) gotovo i da nije.
Quote from: angel011 on 25-06-2012, 20:32:35
Dvojica gaje orhideje, fali ti jedan.
Bato: jedan novčić iz prvog kovčega, iz drugog dva, iz trećeg tri... Ako fali 1 gram u prvom kovčegu su lakši, ako fale 2 u drugom, itd.
:-x Tako mi i treba kad preskačem.
Quote from: Бата Животиња on 25-06-2012, 23:32:28
Ygg, shvatam ja to, samo sistemi eliminacije po meni nisu pravi logički problemi. Jednostavno, ne možeš logički problem da rješavaš po principu hajde da vidimo dokle će ovo da dogura pa ako dođemo u ćosokak probaćemo drugačiji pristup. To po meni nije logički problem, koji na svakom nivou zaključivanja ne može da pogriješi pa da se vraća nazad, kad je nivo pređen onda je pređen, tako se riješava logički problem.
Zapravo baš tako se rešava. Ovaj problem sa Dalasom se principski rešava - ako je jedna pretpostavka pogrešna, a ostale tačne, ako su dve pogrešne itd itd - prave se razne kombinacije i analiziraju - svaka koja izađe iz okvira tj postavljenog uslova se odbacuje dok ne ostane ona koja zadovoljava.
Dug, dosadan i zamoran (ali intelektualno) način rešavanja - u startu sam ga odbacio kao gnjavažu.
Logika je matematička disciplina.
džaba se raspravljamo, to što vi nazivate logikom za mene je od logike udaljeno svjetlosnim miljama
ja stvarno ne znam šta ste vi učili iz logike u srednjoj školi, ja sam učio aristotelovske kategorije, i tu ne postoji mogućnost netačne premise ili zaključka. Ako je nešto od toga netačno to nije logički problem, a ovdje je upravo to slučaj, da ja raspravljam koja premisa nije tačna
Quote from: Ygg on 25-06-2012, 22:47:19
....
U vestima je data sledeća vremenska prognoza za sutra:
1. biće oblačno ili će padati sneg ili će duvati vetar;
2. ako bude oblačno sa snegom, duvaće vetar;
3. ako ne bude vetrovito, biće oblačno bez snega.
Da li se odatle može zaključiti da će, ako bude padao sneg, duvati vetar?
Ne može da se zaključi: 'ako bude padao sneg onda će duvati vetar'.
Iz (3) imamo ako nije vetar onda je oblačno ali bez snega. Ovo može i da se obrne, ako nije oblačno ili ima snega onda ima i vetra. Što će reći imamo dva zaključka. Jedan je ako nije oblačno onda ima vetra, a drugi je ako ima snega onda ima i vetra.
Malo opuštanja:
:mrgreen: :mrgreen:
Quote from: Ygg on 25-06-2012, 22:47:19
Evo još jedne logičke mozgalice. Zadatak je sa opštinskog takmičenja iz matematike, za prvi razred srednjih škola.
U vestima je data sledeća vremenska prognoza za sutra:
1. biće oblačno ili će padati sneg ili će duvati vetar;
2. ako bude oblačno sa snegom, duvaće vetar;
3. ako ne bude vetrovito, biće oblačno bez snega.
Da li se odatle može zaključiti da će, ako bude padao sneg, duvati vetar?
Najlakše ga je rešiti iskaznim formulama.
A evo jednog zadatka za 8. razred ili 1. gimnazije:
Koliko najviše konja može da se postavi na tablu 5 X 5, a da svaki konj napada
tačno dva druga? Traži se i dokaz - zašto ne može više od toga.
@smp: :-|
Quote from: smp on 27-06-2012, 10:46:37
Malo opuštanja:
Je l 0! stvarno 1? Nisam to znao...dajte pojašnjenje...
Quote from: Truman on 30-06-2012, 21:12:42
Je l 0! stvarno 1? Nisam to znao...dajte pojašnjenje...
Hm. Nema nekog objašnjenja. Uzima se
po definiciji da je
0!=1, kao što se po definiciji uzima da je
a^0=1 (za
a različito od nule). Definiše se tako da bi se faktorijel mogao definisati rekurzivno (
n!=n*(n-1)! ), a drugi razlog je zbog gama funkcije, koja je uopštenje faktorijela na skup realnih brojeva (za nju vrijedi da je
Г(n)=(n-1)!, a
Г(1)=1, pa se onda taman uklapa da je
0!=1).
Usput, postoji greška u ovoj
smp-ovoj jednakosti, (
1 + 1/z)^z teži nuli kad
z teži beskonačno, a u jednakosti u eksponentu umjesto
z piše
2. xnerd
Quote(1 + 1/z)^z teži nuli kad z teži beskonačno
Колко си ти литара ракије попио вечерас?
Pardon, e a ne 0. Udarila mi je ova vrućina u glavu. Ali greška u jednakosti i dalje stoji. :)
На коју грешку мислиш? Јасно је да у експоненту умјесто 2 треба да буде з и кад се направи та преправка, онда је у реду. Лимес је једнак е, а ел ен од е је 1.
Quote from: Ygg on 01-07-2012, 00:43:34
Quote from: Truman on 30-06-2012, 21:12:42
Je l 0! stvarno 1? Nisam to znao...dajte pojašnjenje...
Hm. Nema nekog objašnjenja. Uzima se po definiciji da je 0!=1, kao što se po definiciji uzima da je a^0=1 (za a različito od nule). Definiše se tako da bi se faktorijel mogao definisati rekurzivno ( n!=n*(n-1)! ), a drugi razlog je zbog gama funkcije, koja je uopštenje faktorijela na skup realnih brojeva (za nju vrijedi da je Г(n)=(n-1)!, a Г(1)=1, pa se onda taman uklapa da je 0!=1).
Usput, postoji greška u ovoj smp-ovoj jednakosti, (1 + 1/z)^z teži nuli kad z teži beskonačno, a u jednakosti u eksponentu umjesto z piše 2. xnerd
Mrzi me da se udubljujem u jednačine, a pitam zato što na onim iq testovima za posao često daju pitanje da se dobije neki broj od nekih cifara i onda treba da se koristi faktorijal a ovo nisam imao pojma...
Један од разлога зашто је 0!=1 лежи и у комбинаторици. n! је број начина на које n различитих елемената може да се распореди на n мјеста. Ако имамо нула елемената, ти елементи могу на нула мјеста да се распореде тачно на један начин.
Quote from: Прометеј on 02-07-2012, 01:17:47
Један од разлога зашто је 0!=1 лежи и у комбинаторици. n! је број начина на које n различитих елемената може да се распореди на n мјеста. Ако имамо нула елемената, ти елементи могу на нула мјеста да се распореде тачно на један начин.
Jel? Meni bi bilo logičnije da ne postoji nikakav način da rasporedim ništa po mestima (koja uz sve i ne postoje). A ako bi se i našao jedan način, sledeći odgovor bi mi bio - da je 0! = beskonačno.
Quote from: Бата Животиња on 26-06-2012, 00:26:48
džaba se raspravljamo, to što vi nazivate logikom za mene je od logike udaljeno svjetlosnim miljama
ja stvarno ne znam šta ste vi učili iz logike u srednjoj školi, ja sam učio aristotelovske kategorije, i tu ne postoji mogućnost netačne premise ili zaključka. Ako je nešto od toga netačno to nije logički problem, a ovdje je upravo to slučaj, da ja raspravljam koja premisa nije tačna
Aristotelovska logika je jedina koju ste učili u srednjoj školi?
Sad kad spomenuste predmet Logiku iz srednje. To je bre bilo obično bubačenje, kakva crna logika. :mrgreen:
Quote from: Midoto on 02-07-2012, 20:16:04
Quote from: Прометеј on 02-07-2012, 01:17:47
Један од разлога зашто је 0!=1 лежи и у комбинаторици. n! је број начина на које n различитих елемената може да се распореди на n мјеста. Ако имамо нула елемената, ти елементи могу на нула мјеста да се распореде тачно на један начин.
Jel? Meni bi bilo logičnije da ne postoji nikakav način da rasporedim ništa po mestima (koja uz sve i ne postoje). A ako bi se i našao jedan način, sledeći odgovor bi mi bio - da je 0! = beskonačno.
Па није тако. Очигледан (и једини) начин за распоређивање је да ниједан елемент не ставиш ни на једно мјесто :lol:
Ekipa Srbije na otvaranju Međunarodne matematičke olimpijade u Argentini!!! :)
(https://www.znaksagite.com/diskusije/proxy.php?request=http%3A%2F%2Fa3.sphotos.ak.fbcdn.net%2Fhphotos-ak-ash4%2F376404_245424615560979_1295630650_n.jpg&hash=009b199d57c6d5ed842507f68cfcc46b7102d974)
A evo i Bosanaca...
(https://www.znaksagite.com/diskusije/proxy.php?request=http%3A%2F%2Fa5.sphotos.ak.fbcdn.net%2Fhphotos-ak-ash3%2Fs720x720%2F534764_245148662255241_813633369_n.jpg&hash=23efc03cd43bda3d25dc7f3ea5b5ec449fbc784c)
Juče je bio prvi dan takmičenja, a danas je drugi. Evo jučerašnjih zadataka:
(https://www.znaksagite.com/diskusije/proxy.php?request=http%3A%2F%2Fi48.tinypic.com%2F2eaifxx.jpg&hash=c9c7394b0ad1efed8f64fac421e6ed579a209b54)
Današnji još nisu "procurili". A moguće da još uvijek traje takmičenje, mrzi me da preračunavam koliko je sad sati u Argentini.
Evo svih zadataka u kompletu, i jučerašnjih i današnjih. (http://www.artofproblemsolving.com/Forum/resources/files/international_competitions/International_Competitions-IMO-2012-16)
Već dva sata pokušavam da pronađem na netu ovu prokletu knjigu:
http://www.amazon.com/gp/product/images/0444000712/ref=dp_image_0?ie=UTF8&n=283155&s=books (http://www.amazon.com/gp/product/images/0444000712/ref=dp_image_0?ie=UTF8&n=283155&s=books)
Besplatnu, naravno. U pdf formatu ili tako nešto. Knjiga je definitivno postojala nekad online, ali svi linkovi na koje nailazim su mrtvi.
Ako neka dobra duša naleti na knjigu, nek mi pošalje link na PM!
А какве то везе има са Гхоулом и његовим (не)знањем математике?
Quote from: Harvester on 14-07-2012, 03:10:26
А какве то везе има са Гхоулом и његовим (не)знањем математике?
Nikakve. :lol: Čak sam nekad razmišljao da molim Bobana ili Zakka da promjene naziv teme, ali je ipak Ghoul bio inspiracija za ovaj topik, pa je red da njegovo ime ostane u naslovu.
Quote from: Ygg on 14-07-2012, 03:08:29
Već dva sata pokušavam da pronađem na netu ovu prokletu knjigu:
http://www.amazon.com/gp/product/images/0444000712/ref=dp_image_0?ie=UTF8&n=283155&s=books (http://www.amazon.com/gp/product/images/0444000712/ref=dp_image_0?ie=UTF8&n=283155&s=books)
Besplatnu, naravno. U pdf formatu ili tako nešto. Knjiga je definitivno postojala nekad online, ali svi linkovi na koje nailazim su mrtvi.
Ako neka dobra duša naleti na knjigu, nek mi pošalje link na PM!
To bi moglo biti ovo?
http://en.bookfi.org/book/695431
Quote from: Irena Adler on 14-07-2012, 09:44:44
To bi moglo biti ovo?
http://en.bookfi.org/book/695431
Jej, hvala ti puno!!! To je to! :-| :-| :-|
Ovo je još uvijek nezvanično, ali xcheers !!!
QuoteTeodor von Burg from Serbia will have another gold medal this year.
He will become the greatest contestant in IMO history, with 1 bronze, 1 silver and 4 gold medals!
What an incredible achievement!!
Congratulations to him and his country!
http://www.artofproblemsolving.com/Forum/viewtopic.php?f=833&t=488927 (http://www.artofproblemsolving.com/Forum/viewtopic.php?f=833&t=488927)
Ako se dobro sećam, zemlju mogu i da izostave koliko se otvorila za dečka :(
Власт увек може да цитира Кенедија... :(
Quote from: Jevtropijevićka on 14-07-2012, 22:18:25
Ako se dobro sećam, zemlju mogu i da izostave koliko se otvorila za dečka :(
Da. U jednom momentu bilo je upitno i da li će ići na olimpijadu, jer država nije mogla da im obezbjedi pare za avionsku kartu do Argentine. :-x
http://www.blic.rs/Vesti/Beograd/320242/Mladi-matematicari-ne-mogu-na-Medjunarodnu-olimpijadu-u-Argentini (http://www.blic.rs/Vesti/Beograd/320242/Mladi-matematicari-ne-mogu-na-Medjunarodnu-olimpijadu-u-Argentini)
http://www.blic.rs/Vesti/Beograd/320402/Milion-dinara-talentovanim-ucenicima-za-odlazak-na-Matematicku-olimpijadu (http://www.blic.rs/Vesti/Beograd/320402/Milion-dinara-talentovanim-ucenicima-za-odlazak-na-Matematicku-olimpijadu)
Ygga slabo ima na forumu u poslednje vreme, ali evo, ako svrati, neka pogleda ove Venove dijagrame:
http://cartesianproduct.wordpress.com/2012/08/12/venn-diagrams-for-11-sets/ (http://cartesianproduct.wordpress.com/2012/08/12/venn-diagrams-for-11-sets/)
Hvala Meho, tek sad vidjeh ovo!
Usput, evo jednog finog jutjub kanala o matematici: http://www.youtube.com/user/numberphile/videos (http://www.youtube.com/user/numberphile/videos)
Ima mnogo zanimljivih stvari. Na primjer:
http://youtu.be/M-yAgyrzGdo (http://youtu.be/M-yAgyrzGdo)
(https://www.znaksagite.com/diskusije/proxy.php?request=http%3A%2F%2Fa8.sphotos.ak.fbcdn.net%2Fhphotos-ak-snc6%2F226064_463995516954798_1184412518_n.jpg&hash=f250c69a69d8b6bc8051ff4e3c106cae3e1a4d59)
Ne znam da li ovo ide ovde, ali je svakako kuriozitet.
Jutros sam kupovao neke kobasice:
Požarevačke kobasice po 929,80 d/kg;
Stiške kobasice po 362,20 d/kg.
Kolika je bila verovatnoća da cena svakog paketa bude 249,19 dinara?
Verovatnoća - 0
Prejebao te je mesar
Verovatnoća zavisi od ješnosti. U mom slučaju takođe 0, jer su mi kobasice prejake za želudac... :(
Ja imam fotografiju sa oba paketa i obe cene. Ako treba dokaz postovaću ga, ali sam se nadao da mi se može verovati na reč. Imam utisak da je verovatnoća izuzetno mala, ali me mrzi da se sad podsećam kako se to izračunava.
Mićo, ponesi u četvrtak kameru da se slikamo. I to je neponovljivo.
za izracunavanje su potrebni visoki fakulteti. xnerd
Clever Bird Goes Fishing (http://www.youtube.com/watch?v=uBuPiC3ArL8#)
Nema dovoljno podataka za računanje verovatnoće. Potrebni podaci su cene po kilogramu C1 i C2 i statistička raspodela željene mase M1(m) i M2(m), koji su funkcije, recimo u Gausovoj krivi. Kad se pomnože cena i funkcija dobijamo raspodelu cena po parama C(p). Tu analognu funkciju onda treba diskretizovati na nivo jedne pare, tako da za svaku cenu u parama imamo konačnu i realnu verovatnoću da je mesar baš toliko para isekao. Verovatnoća da se pogode cene u parama je onda
(https://www.znaksagite.com/diskusije/proxy.php?request=http%3A%2F%2Flatex.codecogs.com%2Fgif.latex%3F%255Csum_%257Bp%3D0%257D%255E%257B%255Cinfty%257DC_%257B1%257D%28p%29C_%257B2%257D%28p%29&hash=db11f16630f2673da560c70415de0eec1627708c)
Mac, dao sam cene po kilogramu za obe kobasice i cenu za kupljenu količinu - ista 249,19 dinara. To je sigurno dovoljno podataka, a tražio sam samo verovatnoću.
Problem je u mesaru. Ti tražiš X grama a on ti odreže Y, gde je Y blizu ali ne baš jednako X.
Odustajem. Tebi ni fotografija ne bi pomogla. Ej, prepisao sam sa etikete na pakovanju. Tu nema mesara, on je svoje već otsekao. Pitao sam koja je verovatnoća da mi dva paketa različite cene po kilogramu proizvoda i različite količine sadržaja završe sa cenom istom u stotom delu dinara. Možda je to Yggu bliže kao zadatak.
Velika verovatnoća ako tražiš određenu i tačnu cenu. xwink2
vjerovatnoca uopce nije posebno mala. radi se o inkrementima od jednog grama. ako uzmemo raspon 200 do 800 g, sto bi bilo prikladno kupovini obicnog potrosaca, vjerovatnoca da se ovo desi je 1 : 600
slikovito: skalop kupi komad kobasice tezine 200 do 800 g i ja idem pogoditi tezinu iz jednog pokusaja odsijecanjem vrijednostnog ekvivalenta druge kobasice.
sta zbunjuje je tocan iznos na stotinku, sto ne znaci da je inkrement stotinka dinara, vec je u slucaju skuplje kobasice skoro dinar (ekvivalent jednom gramu).
ovo vrijedi i u slucaju da osnovna cijena kobasica nije djeljiva jedna kroz drugu bez ostatka. zasto? zato sto i u tom slucaju postoji jedan iznos gdje se funkcije krizaju, iako najcesce u praksi nije prikaziv. znaci ili je vjerovatnoca 1 : 600 u slucaju kad skalop izabere iznos krizanja (ukoliko ga je moguce izabrati) ili je 0 na bilo koji drugi iznos.
znaci problem moramo podijeliti na pravilo (= 0) i izuzetak (= 1 : 600)
Ja i dalje mislim da te je prejebao mesar
Dakle, pretpostavimo da se to dogodilo ovako:
Verovatno se radi o digitalnoj vagi koja izbacuje one lepljive papiriće na kojima je odštancana cena
Odmerio je prvo skuplje kobasice, dva puta štancnuo papirić, zatim je izfingirao merenje jeftinijih, gde je gurnuo istu količinu, odokativno, a zatim na to pakovanje nalepio onaj, već pripremljeni, i ranije odštampani papirić. I, tako ti je maznuo izvesnu količinu jeftinijih kobaja. Mislim, ne bi bio jedini koga su tako zajebali, to se radi...
Dakle, verovatnoća da pogodi u paru je otprilike tolika kao kada bi ono vozilo što se švrćka po Marsu zavijatilo kamen prema Zemlji i razbilo ti ogledalo u kupatilu...
Hobite, ja i računam sa nekom takvom marsovskom verovatnoćom.
Ali, ženska mi je izmerila četiri Stiške kobasice 0,688 kg. X 362,20 d/kg = 249,19 d.,
a onda dve Požarevačke kobasice 0,268 kg X 929,80 d/kg = 249,19 d.
Ja kod kuće premerio i - tačno. Hej, interesuje me koliko je to moguće.
Ygg, gde si?
Quote from: scallop on 28-08-2012, 15:40:25
Hobite, ja i računam sa nekom takvom marsovskom verovatnoćom.
Ali, ženska mi je izmerila četiri Stiške kobasice 0,688 kg. X 362,20 d/kg = 249,19 d.,
a onda dve Požarevačke kobasice 0,268 kg X 929,80 d/kg = 249,19 d.
Ja kod kuće premerio i - tačno. Hej, interesuje me koliko je to moguće.
Ygg, gde si?
:shock:
Ygg otišao da kupi 0,688 kg Stiških (koje, uzgred, ne volim) i 0,268 kg Požarevačkih - pa će da vidi šta će da ga pogodi...
:lol:
Скалопе, вјероватноћа је 100%.
И што је најгоре, уопште се не зезам.
Onda da preformulišemo pitanje koje sada glasi "kolika je verovatnoća da četiri slučajno odabrane stiške kobasice koštaju isto koliko i dve slučajno odabrane požarevačke". Da bismo mogli da utvrdimo tu verovatnoću moramo da znamo raspodelu cena za oba tipa kobasica. Ta raspodela je opet neka diskretna Gausova kriva, sa vrhom negde oko 62.5 dinara za stišku i 125 dinara za požarevačku. Dakle, verovatnoća da slučajno odabrana stiška kobasica košta X dinara je Y, data na primer sledećoom tabelom:
┌─┬────┬──────┬──────┬──────┬──────┬──────┬──────┬───┐
│X│... │62.48 │62.49 │62.50 │62.51 │62.52 │62.53 │...│
├─┼────┼──────┼──────┼──────┼──────┼──────┼──────┼───┤
│Y│... │0.0018│0.0019│0.0020│0.0020│0.0019│0.0019│...│
└─┴────┴──────┴──────┴──────┴──────┴──────┴──────┴───┘
Sledeći korak je izračunati tabelu verovatnoća za četiri takve kobasice, što je opet diskretna Gausova kriva, ali komplikovano se računa. Cena od C dinara može da se dobije velikim brojem kombinacija sa četiri kobasice, pa je bolje taj proces prepustiti računaru, koji bi odvrteo četiri povezane petlje i sagradio tu novu tabelu. Kad imaš obe tabele verovatnoća onda treba iskoristi onu jednačinu iz mog prethodnog posta.
mace, u prakticnom svijetu (jedne) stiske kobasice postoji samo 62,30 +- 0,36n, pozarevacke 124,60 +- 0,92n
uz to da je i n uzasno ogranicen
u svijetu vise kobasica postaju znacajni i manji dijelovi koji u svijetu jedne ne dolaze do izrazaja.
Eto, to su podaci koji nedostaju. A i to +/- treba bolje da se definiše da bi se problem rešio. To je ta diskretna raspodela o kojoj pričam.
E, ljudi...
Zajebala ga mesarka
Izvin'te što sam pitao. :(
Evo mog pokušaja da (možda) pomognem scallopu, a da se (sigurno) obrukam :(
Mislim da scallop očekuje neko 'jednostavnije' rešenje, nešto nalik ovakvom razmišljanju:
PRIMER
- kobasice se prave mašinskim putem, pa je zanemarljiva verovatnoća da se u prodaji jave one od 25g (ili 525g), dakle ne pravimo veliku grešku ako ne posmatramo normalnu raspodelu težina već samo vrednosti od, recimo 120-160g (ovo samo kao primer!) koje se, heh, javljaju sa sličnim verovatnoćama (znam, znam, ali želimo jednostavno=približno rešenje)
-4 kobasice prve vrste mogu, dakle, težiti 480-640g, pa uz činjenicu da vaga meri grame dobijamo samo 160 (tačnije 161, ali nema veze) različitih mogućih novčanih iznosa
- 2 kobasice druge vrste teže 240-320g, što daje 80 različitih novčanih iznosa
- ukupno imamo 160x80 mogućih parova, tj. svih mogućih ishoda pri ovakvoj kupovini
-na kraju, treba još utvrditi koliko od ovih ishoda čine povoljni parovi, tj. oni sa istim novčanim iznosom, a očigledno da postoji bar jedan takav par (249,19; 249,19)
- verovatnoća će biti približno jednaka količniku broja povoljnih i broja svih mogućih ishoda
Klasično. Ali razumljivo, nadam se. A možda i dovoljno dobro približno rešenje (zavisno od toga kakva odstupanja pravi mašina pri izradi kobasica), koje bi potvrdilo ono što smo i bez računa mogli da znamo: verovatnoća je zaista mala
ove nema jedno pola godine i odmah o kobasicama 8-)
Heh, nisam razmišljao o tome da se prvo meri težina, pa tek onda određuje cena. Ako je cena veća od jedne pare po gramu, to jest 10 dinara po kilogramu onda neke cene zaista neće ni postojati.
:lol: Jednom Bata, uvek Životinja 8)
Da, Mac, uz neko poznato odstupanje u težini kobasica +- 20g (na primer) ispada da njihova cena ne može imati mnogo vrednosti, ali je interesantno da se uopšte javio par (249,19; 249,19) sa istim vrednostima, što daje interesantan zaključak o cenama kobasica 362,20 i 929,80, tj. njihovom sadržaocu 24919.
Inače, ispada da je to jedini slučaj, jer je ukupan iznos linearna funkcija količine (cena x količina), pa ako uopšte te dve funkcije imaju presek na posmatranom (očekivanom) intervalu, onda je takav presek jedinstven.
Ovo je već bliže mojoj pameti.
kad zensko nespretno ponovi vec receno? :lol:
Uzo pop da krsti jariće.
Jedno jare košta 300 dinara žive vage.
Drugo 289 dinara žive vage.
Koje će biti skuplje kršteno?
Quote from: scallop on 28-08-2012, 15:40:25
Ygg, gde si?
Okasnih, ali objasnili su drugi. To je to, uglavnom! :)
Quote from: Lord Kufer on 28-08-2012, 23:23:57
Uzo pop da krsti jariće.
Jedno jare košta 300 dinara žive vage.
Drugo 289 dinara žive vage.
Koje će biti skuplje kršteno?
To je ono: Dokon pop i jariće krsti. :lol:
A skalope, kako pripremaš te kobasice? :lol:
Bez oftopika, moliću lijepo! Nemojte da moram brisati komentare. :mrgreen:
Boban je obećao da će me postaviti za moderatora ovog topika.
za moderatore prihvacamo jedino strojarce, matematicari i elektrotehnika ne dolaze u obzir. :)
:cry:
Quote from: Lord Kufer on 28-08-2012, 23:23:57
Uzo pop da krsti jariće.
Jedno jare košta 300 dinara žive vage.
Drugo 289 dinara žive vage.
Koje će biti skuplje kršteno?
Dragi Lorde, nije nam jasan tvoj zadatak xrofl Po težini razumevanja podseća na ovaj:
Konj vranac se potkiva 5min, dorat 4min, šarac 2min. Žaba diže nogu 0,12m brzinom 0,0003m/min.
Pitanje:
Koliko (u godinama) žaba pati što nad popom nema pop?
To treba pitati Muja dok KUJE konja NA MJESECU :idea:
http://www.oblakznanja.com/2012/10/13/jedanaest-zanimljivih-nacina-koristenja-wolfram-alpha-trazilice/ (http://www.oblakznanja.com/2012/10/13/jedanaest-zanimljivih-nacina-koristenja-wolfram-alpha-trazilice/)
Ima li šanse da se ova knjiga nađe negdje na netu?
http://www.amazon.com/Japanese-Temple-Geometry-Problems-Sangaku/dp/0919611214 (http://www.amazon.com/Japanese-Temple-Geometry-Problems-Sangaku/dp/0919611214)
(https://www.znaksagite.com/diskusije/proxy.php?request=http%3A%2F%2Fecx.images-amazon.com%2Fimages%2FI%2F419mndqL21L._SL500_.jpg&hash=4d19ea88be1f4451c24eca9ffe2854ae9ccb3dfb)
Nema je na bookfi-ju, a nemam pojma gdje drugo da tražim. Ako neko pronađe, nek okači link ovdje ili mi pošalje na PP. Hvala unaprijed! :)
otidi na libgen.info (libgen.org trenutno zajebava) i kucaj "japanese temple geometry" u srču, i izaćiće izdanje iz 2002.
Quote from: tomat on 22-10-2012, 00:57:42
otidi na libgen.info (libgen.org trenutno zajebava) i kucaj "japanese temple geometry" u srču, i izaćiće izdanje iz 2002.
xjap xjap xjap
Ajd da ne otvaram novi topik zbog ovoga...
Da li bi mi neka dobra duša skenirala i poslala na PM strane 180 i 181 Znaka Sagite 2?
U mom primjerku fale te dvije stranice, usred Martinove priče Nightflyers. Žao mi je da ih preskočim, jer se baš zakuvala radnja u tom dijelu priče. :lol:
Evo, dok ne dobijes pristojan sken
(https://www.znaksagite.com/diskusije/proxy.php?request=http%3A%2F%2Fi542.photobucket.com%2Falbums%2Fgg429%2FBojan_B%2Fonline%2Fhdr_00036_light-1.jpg&hash=8e204bff50ac40ea57d828ba6c687c0dd6137d77)
(https://www.znaksagite.com/diskusije/proxy.php?request=http%3A%2F%2Fi542.photobucket.com%2Falbums%2Fgg429%2FBojan_B%2Fonline%2Fhdr_00037_normal-1.jpg&hash=dc101970ea374ea9a1b1fccfbea6479435afd20b)
Hvala Melkore, ovo je dovoljno dobro! xjap
Ako ovo Gula ne privoli matematici, ništa neće:
At last, science explains the physics in "Call of Cthulhu" (http://updates.io9.com/post/34658260553/at-last-science-explains-the-physics-in-call-of)
Quote
By Annalee Newitz
Benjamin K. Tippett has a theory. The University of New Brunswick mathematician believes that he's figured out what, exactly, those insane sailors saw that night in 1928 when they encountered Cthulhu on a lost island in the Pacific. And so Tippett has written a hilariously deadpan paper explaining "non-Euclidean geometry" once and for all. I cannot tell you how much joy and dread I am experiencing right now. Here's what he has to say.
In 1928, the late Francis Wayland Thurston published a scandalous manuscript in purport of warning the world of a global conspiracy of occultists. Among the documents he gathered to support his thesis was the personal account of a sailor by the name of Gustaf Johansen, describing an encounter with an extraordinary island. Johansen's descriptions of his adventures upon the island are fantastic, and are often considered the most enigmatic (and therefore the highlight) of Thurston's collection of documents.
We contend that all of the credible phenomena which Johansen described may be explained as being the observable consequences of a localized bubble of spacetime curvature. Many of his most incomprehensible statements (involving the geometry of the architecture, and variability of the location of the horizon) can therefore be said to have a unified underlying cause.
We propose a simplified example of such a geometry, and show using numerical computation that Johansen's descriptions were, for the most part, not simply the ravings of a lunatic. Rather, they are the nontechnical observations of an intelligent man who did not understand how to describe what he was seeing. Conversely, it seems to us improbable that Johansen should have unwittingly given such a precise description of the consequences of spacetime curvature, if the details of this story were merely the dregs of some half remembered fever dream.
We calculate the type of matter which would be required to generate such exotic spacetime curvature. Unfortunately, we determine that the required matter is quite unphysical, and possess a nature which is entirely alien to all of the experiences of human science. Indeed, any civilization with mastery over such matter would be able to construct warp drives, cloaking devices, and other exotic geometries required to conveniently travel through the cosmos.
Read the entire amazing scientific paper:
http://titaniumphysicists.brachiolopemedia.com/wp-content/uploads/2012/10/Rlyeh.pdf (http://titaniumphysicists.brachiolopemedia.com/wp-content/uploads/2012/10/Rlyeh.pdf)
(https://www.znaksagite.com/diskusije/proxy.php?request=http%3A%2F%2Fwww.smbc-comics.com%2Fcomics%2F20121116.gif&hash=cd8a6a3c485a03f9e32d24b11faf099e5a385191)
(https://www.znaksagite.com/diskusije/proxy.php?request=http%3A%2F%2Fi542.photobucket.com%2Falbums%2Fgg429%2FBojan_B%2FRazno%2F30061112.jpg&hash=08c743b0dc08ed6aa4c7887c73d69bf995fe6c82)
(https://www.znaksagite.com/diskusije/proxy.php?request=http%3A%2F%2Fi542.photobucket.com%2Falbums%2Fgg429%2FBojan_B%2FRazno%2F25301012.png&hash=947d339da2c53b8e888414d1d83bbb6bc16955c3)
(https://www.znaksagite.com/diskusije/proxy.php?request=http%3A%2F%2F25.media.tumblr.com%2Ftumblr_lsffqy9XH41qafrz9o1_500.png&hash=185d4aa7f6f3c8b6b51822fbb02716521b83e070)
(https://www.znaksagite.com/diskusije/proxy.php?request=http%3A%2F%2Fsphotos-h.ak.fbcdn.net%2Fhphotos-ak-ash3%2F579222_501493323205017_815560851_n.jpg&hash=8d6cb3e788d5ea5dc17624aa3bf0ce30ac037885)
Jedan zanimljiv zadačić. :)
QuoteNeku osnovnu školu pohađa 40 učenika. Ako je poznato da svaka dva učenika imaju zajedničkog djeda, dokazati da postoji djed koji u toj školi ima barem 21 unuče.
Dirihle? :)
Jedan učenik - napr. Laza, može imati dedu A ili dedu B. Ostali učenici po uslovu zadatka moraju imati ili A ili B dedu, da bi imali zajedničkog dedu s Lazom. Neka njih 19 ima A-dedu i drugih 19 B-dedu. Onaj preostali, četrdeseti, mora imati ili A ili B dedu, a to je, s Lazom, dvadeset jedno unuče.
Quote from: Midoto on 07-01-2013, 02:20:51
Dirihle? :)
Jedan učenik - napr. Laza, može imati dedu A ili dedu B. Ostali učenici po uslovu zadatka moraju imati ili A ili B dedu, da bi imali zajedničkog dedu s Lazom. Neka njih 19 ima A-dedu i drugih 19 B-dedu. Onaj preostali, četrdeseti, mora imati ili A ili B dedu, a to je, s Lazom, dvadeset jedno unuče.
Naravski! :lol:
Bravo, Midoto!
Bravo! Kasno sam videla zadatak :( Jeste, to je baš lepa primena one priče o tri zeca i dva kaveza.
A evo jedne lake pitalice od mene: šta mislite, da li je pre hiljadu godina na Zemlji živeo čovek koji je do danas stalno imao muške potomke?
U međuvremenu - vic :)
Na svetu postoji 10 vrsta ljudi: oni koji znaju binarni sistem i oni koji ne znaju.
I još jedan :lol:
Na svetu postoji 10 vrsta ljudi: oni koji znaju binarni sistem, oni koji ne znaju i oni koji ne očekuju vic sa osnovom 3.
Postojao. :)
Evo zadatka za Mušet (rekla bih da joj je geometrija jača strana):
Dat je konveksni četvorougao ABCD. Iz temena A povući pravu koja dati četvorougao deli na dve jednake površine.
Dobri vicevi - oba 10. :)
Quote from: Midoto on 09-01-2013, 00:15:59
Dobri vicevi - oba 10. :)
Evo onda jedan i od mene:
Zasto plavusa konvergira?
Zato sto je monotona i ogranicena. :)
I do mene jedan, vrlo stari i bajkovit:
Bili jednom brat i sestra. Brat je bio normalan, a sestra je radila matematiku.
Quote from: Boris on 09-01-2013, 00:20:32
Zasto plavusa konvergira?
Zato sto je monotona i ogranicena. :)
Odličan! Moglo bi tu ponešto da se kaže i o plavuši sa dva policajca :lol:
Quote from: Mouchette on 09-01-2013, 01:45:56
Quote from: Midoto on 09-01-2013, 00:15:59
Postojao. :)
Da, ali kako dokazati? :lol:
Ne možemo sa sigurnošću da kažemo - koji je, ako je bilo veselica u toj lozi, ali je pitanje - da li je postojao. :) Svaki današnji dečak je morao da ima tatu, pa je i ovaj imao tatu...
Imam vrlo težak i isto toliko lep zadatak sa Turnira gradova. Valjala bi mi pomoć, samo još da ga lepo sročim (prevod je malo nejasan).
xjap Baš tako, Midoto!
Inače, čini mi se da imam rešenje za onaj tvoj problem podele četvorougla (tačnije: imam rešenje, a čini mi se da je dobro).
Evo, sad sam stigla kući i nacrtala na brzinu:
(https://www.znaksagite.com/diskusije/proxy.php?request=http%3A%2F%2Fi50.tinypic.com%2F6tnt5u.jpg&hash=aef7be8d7750304d8d201681bbed0dd298bfd891)
S je sredina dijagonale BD, pa četvorouglovi ABCS i ASCD imaju jednake površine.
Ako povučemo SW paralelno sa AC kao na slici, dobijamo jednake šrafirane površine, pa je AW tražena prava. Je l' tačno? :!:
Uvek su ti elegantna rešenja. :)
Sjajno rješenje, Mouchette! xjap
Quote from: Midoto on 09-01-2013, 10:09:07
Imam vrlo težak i isto toliko lep zadatak sa Turnira gradova. Valjala bi mi pomoć, samo još da ga lepo sročim (prevod je malo nejasan).
Kad će taj zadatak? Onaj prošli sa Turnira gradova je bio super zanimljiv! :)
Ja se trenutno zabavljam ovim (http://www.imocompendium.com/srb/dodatne/trigonometrija_ap.pdf) zadacima vezanim za primjenu trigonometrije u geometriji. Prvi sam bezuspješno pokušavao riješiti još ljetos, pa sam onda pomoć dobio na nekom matematičkom forumu. :) Drugi i treći sam uspio samostalno da riješim, kao i sedmi pod a. Ako uspijete riješiti bilo koji od preostalih zadataka, osjećajte se slobodnim da rješenja okačite ovdje. :lol:
Ko zna ruski, evo mu u originalu:
Банк обслуживает миллион клиентов, список которых известен Остапу Бендеру. У каждого
клиента есть свой PIN-код из шести цифр, у разных клиентов коды разные. Остап Бендер
за один ход может выбрать любого клиента, которого он еще не выбирал, и подсмотреть
у него цифры кода на любых N позициях (у разных клиентов он может выбирать разные
позиции). Остап хочет узнать код миллионера Корейко. При каком наименьшем N он
гарантированно сможет это сделать?
Evo o čemu se radi:
Ostap Bender hoće da provali Korejkovu šestocifrenu šifru u banci. Banka ima milion klijenata i svi imaju različite šifre. Ostap sme da otvori nekoliko cifara u kodu svakog klijenta, ali ne sme više da se vraća istom klijentu ako ga je već "obradio". Ako otvara svih 6 cifara - očigledno će pronaći lako Korejkov kod. Isto tako, ako otvara po 5 cifara. Pitanje je: koliko najmanje cifara može da otvara na kodu svakog klijenta, da bude siguran kako će otkriti i Korejkovu šifru?
Eto, malo je zafrknut u formulaciji, ali mislim da je jasno. (Ne mora uvek da bude isti redosled. Ako je jednom klijentu otvarao prvu, drugu, treću, šestu, sledećem može prvu drugu treću, četvrtu... Da ovo ne bude navlaka - ne znači da je najmanji broj potrebnih otvaranja 4.)
Dodatak: Bender zna koji je Korejkov račun, samo ne zna šifru. Jasno, može i kod njega da otvori isto toliko cifara kao kod drugih klijenata.
Zanimljiv zadatak!
Sa tri cifre Bender može lako da dođe do Korejkove šifre. Dovoljno je da u šiframa svih ostalih klijenata otvori prve tri cifre pa da vidi koji mu trocifreni broj fali (svaki trocifreni broj će se ponavljati 1000 puta, osim onog kojim počinje Korejkova šifra; taj će se ponavljti 999 puta), a onda u Korejkovoj šifri otvori tri posljednje cifre.
No, javlja mi se da je moguće doći do šifre i sa otvaranjem po dvije cifre. A i ako nije, treba dokazati da ne može.
Quote from: Ygg on 10-01-2013, 00:31:59
No, javlja mi se da je moguće doći do šifre i sa otvaranjem po dvije cifre. A i ako nije, treba dokazati da ne može.
To! Zato i rekoh da bi mi valjala pomoć.
A ovaj zadatak me je ponukao da ponovo potražim Iljfa i Petrofa. :)
Da li cifre mogu da se otvaraju postepeno, prvo otvoriš jednu cifru, pa onda odlučiš koju ćeš sledeću?
Trebalo bi. Ako imaš rešenje sa dva otvaranja uz taj uslov, odlično!
Lep zadatak :!: Probaću i ja tokom dana da razmislim o tom dokazu da ne može 2 (što mi izgleda mnogo verovatnije od nalaženja algoritma sa 2 otvaranja).
Hm... taj dokaz da ne može manje od 3 možda bi mogao da sadrži i ovo:
- ma kakav da je algoritam za odabir šifara koje ćemo otvorati, mi uvek u stvari otvaramo neku poziciju i kada nađemo grupu od 100000 istih cifara odbacujemo je (tu cifru)
- ako sa M označimo broj neotvaranih šifara koje su preostale posle otkrivanja grupe od po 100000, onda je M <= 899999 (jednu koju tražimo smo ostavili po strani)
a najmanje je Mmin=999999-(10x99999 +1)= 8
Tih 8 neotvaranih je bitno za određivanje min broja N.
E sad, pošto moramo ovo isto da ponovimo još 5 puta, za ostalih 5 cifara (jer su cifre nezavisne u odnosu na svoju poziciju, pa sve ide opet isto), onda... razmisliću šta dalje (spremam mafine, izgoreće mi u rerni - eto zašto veliki matematičari nisu bili matematičarke :lol: )
Nešto sam zaboravila da pitam Midoto. Da li onaj zadatak koji sam rešila (o podeli četvorougla) ima i neko drugačije rešenje (pošto si pomenuola da je ovo moje rešenje 'elegantno')? Tj. da li se na neki drugi način može doći do ove iste prave?
Quote from: Midoto on 10-01-2013, 00:55:30
Quote from: Ygg on 10-01-2013, 00:31:59
No, javlja mi se da je moguće doći do šifre i sa otvaranjem po dvije cifre. A i ako nije, treba dokazati da ne može.
To! Zato i rekoh da bi mi valjala pomoć.
A ovaj zadatak me je ponukao da ponovo potražim Iljfa i Petrofa. :)
И Мела Брукса!
E ovo sa Melom Bruksom mi nije jasno.
Mušet, mene buni što nisu sve cifre nezavisne. U redu, ima 100 000 napr. dvojki na prvom mestu. Međutim, ima samo 10 000 parova 23 na prvom i drugom mestu. Ako pretpostavimo da otkrivam u 900 000 raačuna prvo i drugo mesto, a onda na 90 000 računa prvo i treće mesto (itd.) da li bih nekim eliminacijama došla do odgovarajućih cifara?
Ima i drugačije rešenje onog zadatka: prvo pretvorim ABCD u trapez iste površine, a onda taj trapez u trougao čija je osnovica na pravoj DC, a preostalo teme je A. Konačno sam trougao podelim trivijalno na dve jednake površine iz temena A. Ako nisam jasna, naoružaj se strpljenjem, pa ću ispisati korak po korak, jer sam tek savladala kačenje fotografija, a skeniranje pa kačenje je van mojih trenutnih mogućnosti. :)
Za prethodni zadatak - stvarno ne može manje od 3 (sem ako mi je nešto neverovatno promaklo). Imamo milion otvaranja (sem jednog, Korejkovog) za još 2 000 000 nezavisnih dvocifrenih brojeva...
Evo jednog vrlo simpatičnog:
U zatvoru je 6 zatvorenika. Stražar ima 60 flašica: u 59 je pravi sok, u jednoj je tečnost koja po svemu (mirisu, ukusu, boji...) liči na sok, ali je nekakva gadna supstanca - dovoljno je uzeti jednu kap i sutradan se probuditi trešten pijan.
Stražar mora (i može) tokom jedne noći da pronađe koja je to flašica. Kakva mu je strategija?
20 kapi iz 20 boca će pomešati i dati jednom zatvoreniku.
Ako se napije, dakle, gadna boca je u tih 20.
Onda će drugom dati 7 kapi iz sedam boca od tih 20.
Ako se napije onda će trećem dati 3 kapi iz tri boce od tih 7.
Ako se napije onda će dvojici dati po kap iz dve boce, pa ako neki bude pojan tajj je, ako ne bude, onda je u preostaloj boci.
A ko drugi nije bio pijan onda treba novih 7 kapi iz 14 mogućih boca dati jednom....
I tako dalje, ima gomila varijacija na temu, ali to je siste,
Suština je da dođe do prave boce pre nego što napije svu šestoricu.
Ovom zadatku nedostaje ipak neko ograničenje, jer on može krenuti redom i dati po jednu kap iz različitih boca svakome, pa ako niko ne padne pijan, onda ponoviti sa drugih šest boca i tako redom dok jedan zatvorenik ne padne. Zato mislim da treba ograničiti broj pokušaja.
Imaš ograničenje - sutradan se budi trešten. Stražar može tek nakon buđenja da odredi koja je boca neispravna na osnovu pijanstva zatvorenika. Dakle - tokom noći im daje piće, sutra ujutru podnosi izveštaj.
E, to je već ozbiljan zadatak.
Quote from: Midoto on 10-01-2013, 17:30:25
E ovo sa Melom Bruksom mi nije jasno.
Мел Брукс је урадио одличну адаптацију књиге Дванаест столица. Франк Лангелла је Остап Бендер.
I ja bih da ti postavim jedan zadatak, ali da podneseš izveštaj meni. :evil:
Za zatvorenike koristiti binarni sistem. Svakoj bočici dodeliti broj od 1 do 60. Svaki broj se predstavi kao 6 cifara u binarnom obliku, pa je na primer 42 dekadno = 101010 binarno. Svaka cifra kontroliše po jednog zatvorenika, i zatvorenik mora da popije po kap iz svake bočice koja ima cifru jedan dodeljenu za tog zatvorenika. Sutradan konstruišemo binarni broj tako što upisujemo 1 za svakog pijanog, i 0 za treznog, i to upisujemo tačno na ono mesto u broju koje odgovara tom zatvoreniku. Dati binarni broj određuje redni broj bočice.
Rešenje zadatka bi bio očiglednije da je dato 64 bočica.
Quote from: Harvester on 24-01-2013, 11:02:20
Quote from: Midoto on 10-01-2013, 17:30:25
E ovo sa Melom Bruksom mi nije jasno.
Мел Брукс је урадио одличну адаптацију књиге Дванаест столица. Франк Лангелла је Остап Бендер.
Naći ću, mada sam ubeđena da sam davno odgledala sve njegove filmove.
@Mac, zato je i stavljeno 60 - da ne bi odmah vuklo na binarni sistem. Zapravo je u originalu 1000 bočica, a 10 zatvorenika, ali kada se smanji stepen dvojke, dostupnije je i klincima koji intuitivno dele na pola.
Zanimljivo, imam utisak da bih se dosetio rešenja za sekundu brže, da je bilo dato 1000/10. U mom svetu 1024 je "okruglija" brojka od 64 :)
Da, to je onaj vic o dva programera koja se klade u 1000 dinara, a jedan kaže: "Neka bude 1024, da bude okruglo."
Tragam za zadacima za koje ne treba previše znanja, ali razvijaju logičko razmišljanje (po meni je ovaj prethodni jedan od takvih) i često mogu da se rešavaju u glavi. Ima li ko da prijavi neki takav?
Quote from: Midoto on 24-01-2013, 10:08:08
U zatvoru je 6 zatvorenika. Stražar ima 60 flašica: u 59 je pravi sok, u jednoj je tečnost koja po svemu (mirisu, ukusu, boji...) liči na sok, ali je nekakva gadna supstanca - dovoljno je uzeti jednu kap i sutradan se probuditi trešten pijan.
Stražar mora (i može) tokom jedne noći da pronađe koja je to flašica. Kakva mu je strategija?
Lijep zadatak! I odlično
mac-ovo rješenje! Iskoristiću ga na dodatnoj. (Zadatak, a ne
mac-a. :) )
Quote from: Midoto on 24-01-2013, 12:37:35
Tragam za zadacima za koje ne treba previše znanja, ali razvijaju logičko razmišljanje (po meni je ovaj prethodni jedan od takvih) i često mogu da se rešavaju u glavi. Ima li ko da prijavi neki takav?
Evo jednog iz kombinatorne geometrije koji spada u takve:
Imamo u ravni 3 vodoravne i 7 uspravnih pravih. Njihovi presjeci određuju 21 tačku. Svaka tačka je obojena crvenom ili plavom bojom. Dokazati da je, bez obzira na bojenje, uvijek moguće pronaći pravougaonik čija su sva četiri tjemena obojena istom bojom.
Napomena: Ako imamo 3 vodoravne i 6 uspravnih pravih, onda je moguće obojiti tačke tako da nemamo istobojnih pravougaonika. Evo na slici jednog takvog bojenja.
(https://www.znaksagite.com/diskusije/proxy.php?request=http%3A%2F%2Fi47.tinypic.com%2F352oy9e.jpg&hash=e013e7cfbe0b4d3faa649009f1768c69071ae602)
Мрзим те задатке гдје мора да се мисли.
I ovo mi miriše na nešto binarno. Ovih šest kolona predstavljaju binarne brojeve od 1 do 6. Istovremeno, tih šest kolona su i sve moguće kombinacije od jednog elementa jedne boje i dva elementa druge boje. Ali ne znam kako to da iskoristim.
Zbog Maca ću plave nazvati 1, a crvene 0 (mada bih radije baratala slovima P i C). :)
U ovom primeru je raspored: 110, 001, 011, 010 100, 101, tj. ne smeju da se ponove dve jedinice ili dve nule na istim mestima (prvo i drugo, prvo i treće...). Ako sedmi put ponovimo neku od ovih kombinacija - očigledno je da imamo praougaonik. Ukoliko ne želimo da ponavljamo, ostaje nam samo 000 ili 111, a to će stvoriti tri pravougaonika.
Ygg, vole li tvoji učenici ovakve mozgalice?
Čula sam pre izvesnog vremena na seminaru desetak lakih zadataka, za čije rešavanje nije potrebno nikakvo matematičko predznanje (možda će baš zbog toga biti interesantni i drugima). Za početak:
1. Šta je sledeće?
A H ∀
2. Koje slovo treba dodati?
A B C D _
Mouchette, hajde razjasni mi ovaj drugi, jer dumam šta bi moglo da bude sem E, a ne nalazim logiku.
Evo vam jedan "orah". Opet stražar i opet zatvorenici (i opet sam ga preradila u manje brojeve, jer mislim da je odličan za dodatnu nastavu :) )
Stražar ujutru okupi deset zatvorenika i na leđa im zalepi jedan od brojeva iz skupa od 1 do 10. Neki zatvorenici mogu imati i isti broj na leđima, jer stražar ima po 10 komada od svakog broja, pa nasumično stavlja nalepnicu, a mogu imati i svi različite. Tek - svaki zatvorenik ima na leđima jedu nalepnicu i svaki može da vidi ostalih devet, pa da (bez ikakvog domunđavanja, znakova ili sličnih prevarica) zapiše broj od 1 do 10, za koji misli da je na njegovim leđima. Ako bar jedan zatvorenik pogodi broj - svi bivaju nagrađeni. Ako svi promaše - svi će biti kažnjeni. Elem, zatvorenici naprave strategiju (matematičku) zahvaljujući kojoj svakodnevno dobijaju nagrade. Kakva im je strategija?
Ha, znam odgovor na drugo: F, jer već imamo donju crtu da na kraju dobijemo E.
:)
Baš me je oraspoložio.
Jednostavan odgovor na prvo može biti H, jer kad H okreneš za 180 stepeni opet dobijaš H, međutim moglo bi biti i —. Posmatrajmo kretanje dve kose linije koje čine slovo A. Ako razdvojimo ove dve linije, tako što ih okrenemo oko spojnica sa trećom horizontalnom linijom, onda dobijamo H. Ako nastavimo da okrećemo ove dve linije onda dobijamo ∀, a ako nastavimo da ih okrećemo još više onda dobijamo prosto jednu dugu vodoravnu liniju, to jest —
1. Da, Mac je u pravu, F je traženo slovo.
2. Da, Mac je u pravu, — je sledeći simbol.
Obično svi kažu da je to H, ali tada nemamo pravilo i vezu između susednih članova, a ako ćemo da posmatramo pravilo za dva člana (A H ∀ H...) onda bismo mogli da imamo i za tri člana (A H ∀ A H ∀...), što mislim da nije namera onog ko je sastavljao zadatak.
A kad ste ovako brzi, evo još jednog zadatka sa istog tog skupa.
Sudija se obraća optuženom: 'Vi ste drugooptuženi. U toku istražnog postupka nedvosmisleno je dokazano da ste prisustvovali svim ubistvima, ali da ni jedno niste vi izvršili. Čak je dokazano da ste pokušali da sprečite da do ubistava dođe. Zbog svega toga u potpunosti vas oslobađam optužbe.'
Zatim se obraća drugom optuženom: ''Vi ste prvooptuženi. U toku istražnog postupka nedvosmisleno je dokazano da ste baš vi počinili sva ubistva i zbog toga bih morao da vam, po zakonu, izreknem smrtnu kaznu, ali ja to neću, ipak, učinitii.'
Zašto?
(ako neko zna ovaj zadatak odranije ne mora odmah da napiše rešenje, možda će ga rešiti neko ko ga čuje po prvi put :) )
Obožavam ovakve pitalice a dosetila sam se i rešenja, samo ne bih odmah da ga pišem, da ne kvarim zabavu. :)
Quote from: Dacko on 29-01-2013, 18:17:45
Obožavam ovakve pitalice a dosetila sam se i rešenja, samo ne bih odmah da ga pišem, da ne kvarim zabavu. :)
Hajde, pošalji mi ga na PM, da ne budeš u neizvesnosti.
Ubica je Đems Bond, on ima dozvolu za ubijanje.
Ako ima dozvolu onda mu po zakonu ne mogu ništa. To "po zakonu" je najveći deo problema.
Ubistvo je protiv zakona. Ne sudi se za ubistvo nego za "protiv zakona".
Ne, ne, to nije jedna od onih priča o tome kako je žena napustila muža a njega su optužili da ju je ubio. On onda odleži 20 godina u zatvoru, zatim je nađe, ubije i biva oslobođen, jer je već robijao. Ovde se radi o višestrukom ubistvu, žrtve su poznate, a ubica nije imao nikakvu dozvolu, dozvolu za ubijanje.
Da kažem i javno: Dacko je rešila! xjap
Možda su u pitanju sijamski blizanci, pa ne možeš da kazniš krivca, jer bi tako kaznio i nevinog.
I meni je palo na pamet - ili sijamski blizanci ili shizofrena ličnost (mada ne bi bilo baš po zakonu, s obzirom na bolest). :)
Koji je pravi odgovor?
I opet: da, mac je u pravu! Radi se o sijamskim blizancima.
P.S. A kad je Midoto već tu, jedno pitanje vezano za njen zadatak: da li zapisivanje brojeva ide tako da zapisuju jedan po jedan, a ostali vide šta je on zapisao?
Hoću i ja pohvalu!
Nisam pročitala diskusiju pre nego što sam dala šansu traženju rešenja :)
Dodatak: tek maločas sam otkrila pitalicu.
Dodatak: pohvala za Midoto :lol:
Hvala. :lol:
Ala ste se raspisali!
Žao mi je što sam propustio priliku da odgovaram na pitalice. Možda ne bi bilo loše da odgovore pišete tako da se ne vide dok se mišem ne skroluje preko teksta, pa da i mi koji kasnije vidimo zadatke razmišljamo malo o njima prije nego što pogledamo odgovore. Teško je ne vidjeti odgovor, kada se nalazi odmah ispod zadatka.
I samo nastavite sa zanimljivim zadacima i pitalicama, jer materijala za dodatnu nikad dosta! :lol:
Quote from: Midoto on 28-01-2013, 22:08:02
Ygg, vole li tvoji učenici ovakve mozgalice?
Imam samo dvoje učenika koji redovno dolaze na dodatnu. Još nekoliko njih navrati ponekad, ali ne redovno. Ovih dvoje vole, i uglavnom oni meni postavljaju zadatke, a ne ja njima. Ova starija je danas zadatak sa pravama i tačkama riješila za nekih 20 sekundi. A za dokaz neke uslovne nejednakosti za koji je meni trebalo dobrih 45 minuta (zadatak sam rješavao za ovog mlađeg učenika), ona je potrošila možda dva minuta i njeno rješenje je elegantnije od moga. :cry:
Quote from: Ygg on 30-01-2013, 00:07:48
Ovih dvoje vole, i uglavnom oni meni postavljaju zadatke, a ne ja njima. Ova starija je danas zadatak sa pravama i tačkama riješila za nekih 20 sekundi. A za dokaz neke uslovne nejednakosti za koji je meni trebalo dobrih 45 minuta (zadatak sam rješavao za ovog mlađeg učenika), ona je potrošila možda dva minuta i njeno rješenje je elegantnije od moga. :cry:
Blago tebi! :) Nema optimističnije stvari nego kad vidiš da mladi mozgovi tako dobro rade.
Zar se nikome ne dopada zadatak sa zatvorenicima i brojevima, pa nema ni naznake rešavanja?
Meni nije baš najjasnije kako se obavlja zapisivanje brojeva (verovatno nisi videla da sam gore isto to pitala). Da li zapisivanje brojeva ide tako da zapisuju jedan po jedan i da li ostali vide šta je ko zapisao?
Uf, izvini, stvarno nisam videla (morala sam da se cenjkam oko pohvale). Evo ovako: napravili su plan; svi su videli brojeve na leđima preostalih zatvorenika; svi zapišu broj za koji misle da imaju na svojim leđima; na osnovu čiste matematičke strategije - ispune uslov za nagradu.
Znači svi zapisuju istovremeno? Ne može jedan zatvorenik da čeka da drugi završi svoje zapisivanje?
Da, i mene interesuje baš ovo što pita mac (onda se mogu dogovoriti da zapisuju, recimo, najmanji broj koji vide, ili tome slično, pa da svaki naredni ima tu informaciju).
Zadatak nema smisla ako vide ko šta zapisauje, jer bi onda strategija bila jednostavna: dogovore se da prvi zapiše broj koji vidi na leđima npr. desetog, i onda deseti zna koji je njegov broj.
Tačno - nema varanja.
Mali hint: na osnovu njihove strategije, tačno jedan će biti u pravu.
Hoću da kažem: mogu svi istovremeno da napišu svoj broj, ako jednakom brzinom računaju. :)
Midoto je odgovorila, pa sad treba porazmisliti, ali ja bih samo da napomenem da sam pitao samo za redosled zapisivanja, ali ne i da je potonji zatvorenik video šta je prethodni zapisao.
Uzgred, meni se ovaj zadatak "javlja" iz neke Galaksije, ali to mi ništa ne pomaže u vezi sa rešenjem.
U originalnom zadatku je 100 zatvorenika. Istini za volju, meni je sin (1. gimnazije) objasnio kako je rešio. Pomoglo mi je tek kada sam problem svela na 3 zatvorenika, a pod istim uslovima: imaju nalepnice sa nekim od brojeva iz skupa od 1 do 3.
Quote from: Midoto on 29-01-2013, 10:02:25
Stražar ujutru okupi deset zatvorenika i na leđa im zalepi jedan od brojeva iz skupa od 1 do 10. Neki zatvorenici mogu imati i isti broj na leđima, jer stražar ima po 10 komada od svakog broja, pa nasumično stavlja nalepnicu, a mogu imati i svi različite. Tek - svaki zatvorenik ima na leđima jedu nalepnicu i svaki može da vidi ostalih devet, pa da (bez ikakvog domunđavanja, znakova ili sličnih prevarica) zapiše broj od 1 do 10, za koji misli da je na njegovim leđima. Ako bar jedan zatvorenik pogodi broj - svi bivaju nagrađeni. Ako svi promaše - svi će biti kažnjeni. Elem, zatvorenici naprave strategiju (matematičku) zahvaljujući kojoj svakodnevno dobijaju nagrade. Kakva im je strategija?
Ja ne uspjevam da riješim ovaj zadatak! :cry:
Zatvorenici dobijaju brojeve koji su nezavisni jedni od drugih, jer stražar ima po 10 naljepnica sa svakim brojem i stavlja ih nasumično, tako da ne vidim kakvu strategiju oni mogu da imaju. Eventualno bi mogli da koriste vjerovatnoću, npr. ako jedan zatvorenik vidi na ostalim zatvorenicima 3 šestice i 5 dvojki, onda može da zaključi da je veća vjerovatnoća da on na leđima ima šesticu nego da ima dvojku, a još veća vjerovatnoća je da na leđima ima broj koji niko od preostalih zatvorenika nema. Ali teško da tako mogu da naprave strategiju pomoću koje jedan od njih sigurno pogađa svoj broj.
Usput, evo najtežeg logičkog problema na svijetu! :)
(https://www.znaksagite.com/diskusije/proxy.php?request=http%3A%2F%2Fimg.spikedmath.com%2Fcomics%2F536-lolgic-puzzle.png&hash=5547621c471f5037533952c5b1b8f2864049af9c)
@Ygg: Obrazlažem na primeru 3 zatvorenika. Neka njih trojica imaju na sebi brojeve a, b i c (ne nužno međusobno različite). Zbir ta 3 broja može biti, po modulu 3 (deca mogu da ukapiraju i preko ostataka pri deljenju sa 3): 0, 1 ili 2. Zatvorenici se dogovore za redosled: prvi se ponaša kao da ukupan zbir treba da bude 0 po modulu 3; drugi kao da je 1; treći da je 2 po modulu 3. Jedan od njih mora biti u pravu, a ostali greše. Dakle, ako prvi vidi brojeve b i c, on zapiše broj x koji zadovoljava uslov x + a + b = 3k. Drugi vidi a i c, pa zapisuje broj y koji zadovoljava uslov y + a + c = 3k +1. Za trećeg - po istom principu. Dakle, tačno jedan od njih je u pravu. Kada je 10 ili 100 zatvorenika - isto to, samo po modulu 10 ili 100.
Lep, jel da? :)
Sad ću da razmatram ovaj tvoj problem, baš mi se dopao.
Ubi me zadatak, uporno mi fali četvrto pitanje, zbog one loše mace koja nasumično govori istinu i laž. Ima li neko rešenje (nemojte odmah da stavite ako imate, dajte još dan fore)?
Quote from: Midoto on 02-02-2013, 18:21:01
@Ygg: Obrazlažem na primeru 3 zatvorenika. Neka njih trojica imaju na sebi brojeve a, b i c (ne nužno međusobno različite). Zbir ta 3 broja može biti, po modulu 3 (deca mogu da ukapiraju i preko ostataka pri deljenju sa 3): 0, 1 ili 2. Zatvorenici se dogovore za redosled: prvi se ponaša kao da ukupan zbir treba da bude 0 po modulu 3; drugi kao da je 1; treći da je 2 po modulu 3. Jedan od njih mora biti u pravu, a ostali greše. Dakle, ako prvi vidi brojeve b i c, on zapiše broj x koji zadovoljava uslov x + a + b = 3k. Drugi vidi a i c, pa zapisuje broj y koji zadovoljava uslov y + a + c = 3k +1. Za trećeg - po istom principu. Dakle, tačno jedan od njih je u pravu. Kada je 10 ili 100 zatvorenika - isto to, samo po modulu 10 ili 100.
Lep, jel da? :)
Dovraga! Čak mi je u jednom momentu palo na pamet da gledam zbirove po modulu. Razmišljao sam da, ako imamo 10 zatvorenika, zbirovi njihovih brojeva mogu uzimati vrijednosti od 10 do 100, pa sam onda "zaključio" da zbir po modulu 10 ne određuje jedinstveno rješenje. :)
Quote from: Midoto on 03-02-2013, 22:30:40
Ubi me zadatak, uporno mi fali četvrto pitanje, zbog one loše mace koja nasumično govori istinu i laž. Ima li neko rešenje (nemojte odmah da stavite ako imate, dajte još dan fore)?
Ovaj zadatak je dovoljno poznat da ima i svoj članak na Vikipediji. U originalu nisu mace, već bogovi:
QuoteThree gods A, B, and C are called, in no particular order, True, False, and Random. True always speaks truly, False always speaks falsely, but whether Random speaks truly or falsely is a completely random matter. Your task is to determine the identities of A, B, and C by asking three yes-no questions; each question must be put to exactly one god. The gods understand English, but will answer all questions in their own language, in which the words for yes and no are da and ja, in some order. You do not know which word means which.
A autor je dao i dodatna pojašnjenja:
Quote
- It could be that some god gets asked more than one question (and hence that some god is not asked any question at all).
- What the second question is, and to which god it is put, may depend on the answer to the first question. (And of course similarly for the third question.)
- Whether Random speaks truly or not should be thought of as depending on the flip of a coin hidden in his brain: if the coin comes down heads, he speaks truly; if tails, falsely.
Ma ja posebno volim ovakve zadatke, ali ako ga danas ne rešim, moraću da pročitam rešenje. Trpe ostale moje obaveze - juče mi je zagoreo spanać dok sam raspisivala jedan slučaj s različitim odgovorima. O pisanju da ne govorim. Inače - ovo je upravo zadatak koji ne traži matematičko znanje, već samo razmišljanje. Izuzetno je lep. :)
Ovaj zadatak koji je Ygg postavio znam odranije (ima i nekoliko sličnih varijacija ovog problema). Međutim, rešenje zadatka sa zatvorenicima koje je Midoto dala je super! Pomislila sam i ja u jednom trenutku na zbirove po modulu, ali sam ih isto kao i Ygg odbacila. Tek sada vidim da oni, iako idu od 10 do 100, imaju u stvari relativan 'raspon' 10 (u odnosu na ono što zatvorenici vide pojedinačno). Lep zadatak. Red je da se sad Midoto pomuči sa LOLgičkom zavrzlamom.
U to ime evo jednog zanimljivog rasuđivanja:
limc→0 (sec x /c2 ) = lim c→0 (sex/c) = beskonačan seks :lol:
Ovo je suviše apstraktno. :)
Quote from: Midoto on 05-02-2013, 11:06:36
Ovo je suviše apstraktno. :)
Aha! I još moraš biti glup da bi do njega došao :cry:
I uz skraćivanje.
I uz skraćivanje na pogrešnom mestu.
Nema veze, neka drugi misle da se OVO može nazvati beskonačnim seksom. Mi znamo šta je tačno 8)
Pa uz skraćivanja na pravim mestima nikada ne bi došao do takvog rezultata.
Zaključak: mora se biti glup, (u)skraćen na pogrešnom mestu, sklon apstraktnom razmišljanju... Nemoguća kombinacija. :)
Da, nemoguća kombinacija :)
Sad će neko pametan da primeti kako grešimo što uopšte potežemo matematiku. Zadovoljstvo i matematika su za mnoge disjunktni.
Quote from: Midoto on 04-02-2013, 10:14:42
Ma ja posebno volim ovakve zadatke, ali ako ga danas ne rešim, moraću da pročitam rešenje. Trpe ostale moje obaveze - juče mi je zagoreo spanać dok sam raspisivala jedan slučaj s različitim odgovorima. O pisanju da ne govorim. Inače - ovo je upravo zadatak koji ne traži matematičko znanje, već samo razmišljanje. Izuzetno je lep. :)
Sad ću imati grižu savjest kad god okačim neki problem na ovaj topik! :)
Elem, evo detaljno objašnjenog rješenja na Vikipediji: http://en.wikipedia.org/wiki/The_Hardest_Logic_Puzzle_Ever (http://en.wikipedia.org/wiki/The_Hardest_Logic_Puzzle_Ever)
Ukratko, pitanja su sljedeća:
Quote
Q1: Ask god B, "If I asked you 'Is A Random?', would you say ja?". If B answers ja, either B is Random (and is answering randomly), or B is not Random and the answer indicates that A is indeed Random. Either way, C is not Random. If B answers da, either B is Random (and is answering randomly), or B is not Random and the answer indicates that A is not Random. Either way, you know the identity of a god who is not Random.
Q2: Go to the god who was identified as not being Random by the previous question (either A or C), and ask him: "If I asked you 'Are you False?', would you say ja?". Since he is not Random, an answer of da indicates that he is True and an answer of ja indicates that he is False.
Q3: Ask the same god the question: "If I asked you 'Is B Random?', would you say ja?". If the answer is ja, B is Random; if the answer is da, the god you have not yet spoken to is Random. The remaining god can be identified by elimination.
Hvala, ja sam morala da pogledam rešenje ranije. Čak sam i koristila tu caku: Kada bih te pitala - šta bi mi ti odgovorila - ali nisam izgurala do kraja.
I samo daj još takvih zadataka! Dok ne rešim zadatak, stvarno pravim razne brljotine, ali kad ga rešim (ili pročitam rešenje, što pruža nešto manje radosti) kao da mi se malo pročisti i sredi mozak, pa sve što radim bude za nijansu bolje od uobičajenog. Mislim da matematika dovodi misli na pravo mesto: u tome je štos. :)
Ево вам један задачић и од мене. Мене је прилично напатио, али видјевши какве заврзламе овде рјешавате, ово ће вам бити пиеце оф цаке.
Имате пет гусара који треба да подијеле плијен од 100 златника. Гусари су стриктно хијерархијски поредани - означимо их рецимо са А, Б, Ц, Д, Е, гдје је А капетан, Б рецимо први официр, Ц други официр итд.
Подјела функционише на сљедећи начин: Капетан предлаже како да се подијели лова и онда се гласа о приједлогу (гласају сви, укључујући и оног ко предлаже). Ако буде 50% или више гласова за, приједлог се усваја и лова се распоређује како је предложено. Ако више од пола гусара гласа против, капетан се шаље на ход по дасци, а лова се распоређује између осталих гусара (Б, Ц, Д и Е), при чему сљедећи приједлог даје први официр (Б). Процедура се понавља - дакле, приједлог, гласање, ако има пола гласова за, усваја се, ако не, опет ход по дасци и расподјела међу преостала три гусара итд.
Гусари се строго придржавају сљедећих правила:
1) Први приоритет је да преживљавање (дакле подразумијева се да нико неће дати такав приједлог расподјеле који би други због својих интереса одбили)
2) Други приоритет је да се добије што је могуће више новца (дакле, примјера ради, ако онај ко предлаже расподјелу може да прође и са, рецимо, 56 и са 57 златника, предложиће расподјелу у којој добија 57. Слично важи и за гласање - сваки гусар ће да одбије приједлог расподјеле ако би смрт предлагача и смањивање броја гусара довели до тога да он добије више новца)
3) Ако је гусару свеједно да ли приједлог пропада или пролази (тј. ако у оба случаја добија исту количину новца), гласаће тако да предлагача пошаље у смрт.
Питање гласи - колико новца ће да припадне капетану (дакле гусару А)?
Imam utisak da prva četvorica ne dobijaju ništa, ali bar zadržavaju svoje živote. Ali verovatno nije to rešenje.
Није, одговор је заправо прилично неочекиван (да не кажем суманут).
Peti gusar će uvek da glasa protiv, s ciljem da dobije što više, koliko god da je to. Četvrti gusar ne sme da dozvoli da preostanu samo dva (jer će poginuti šta god da predloži), pa će da glasa za bilo šta što predloži treći gusar. Međutim to nije dovoljno, jer i dalje ima 50% glasova protiv, što znači da ni treći gusar ne sme da dozvoli da on predlaže podelu, pa će da glasa za bilo šta što predloži drugi gusar. Drugi gusar ne mora da se plaši za svoj život jer ga "čuvaju" treći i četvrti, i zato može da glasa protiv bilo čega što prvi gusar predloži. Pošto svi to znaju i pošto svi znaju da poslednja tri gusara neće dobiti ni cvonjka bilo šta što prvi gusar predloži biće jednoglasno odbijeno. Prvi gusar ne dobija ništa, i pritom gubi život, a drugi gusar dobija sve.
Znači, drugi gusar je predložio ovakvu raspodelu, a to bi i trebalo da bude pitanje: koji od njih petorice smišlja ovakav zadatak i nameće ga drugima :twisted:
A ne, čekaj, sve mi je pogrešno. Mislio sam da onaj ko predlaže ne glasa, ali glasa i on. Hm, nazad u laboratoriju.
Srednji gusar ma šte da predloži, poslednji mora da usvoji, jer ako ih ostanu dvojica - nema više od pola glasova protiv. Dakle, ako bi treći predložio raspodelu: 99, 0, 1 - poslednji bi se složio, jer je to više nego što bi mu sledovalo na kraju. Po istom kriterijumu, ako drugi gusar predloži: 97, 0, 1, 2 - svi bi trebalo da prihvate (sem trećeg), dakle, kapetan treba da odobrovolji poslednju dvojicu, da drugog i trećeg eliminiše, pa da predloži:
A: 95
B: 0
C: 0
D: 2
E: 3.
Je li to rešenje?
Eh, ja ne razumem :oops: Šta ako kapetan za sebe predloži 48, za dvojicu po 26, a za dvojicu 0? Onda će njih trojica da prihvate predlog, podeliće 100 zlatnika i... šta onda?
eh, da, ulete Midoto. Traži se, dakle, koliko najviše može da uzme A?
Mislim da kapetan gleda što više da isposluje za sebe.
Ja ne vidim "rupu" u ovom rešenju. Kako vama izgleda?
Vidim - mora da bude više od pola protiv. Znači, četvrti može da predloži: 98, 0, 0, 2 i da već ima dva glasa. Onda bi kapetan mogao da da ovakvu raspodelu:
96, 0, 0, 1, 3 i da dobije 3 glasa, jer su poslednja dvojica "profitirala".
Okej, ako preostane dvojica onda peti uvek glasa protiv, a četvrti umire. Ako preostane trojica onda četvrti uvek glasa za i bar zadržava život, a treći dobija sve. Ako preostane četvorica onda drugi mora da obezbedi još dva glasa da bi preživeo, a to su četvrti i peti, jer treći uvek glasa protiv da bi dobio sve. To znači da ako preostanu četvorica drugi ima 98 zlatnika, četvrti i peti imaju po jedan zlatnik, a treći ne dobija ništa.
Prvi za većinu mora obezbedi isto dva glasa, i ta dva glasa mora da kupi dajući više nego što bi odgovarajući ljudi dobili u slučaju da preostane četvorica. Treći je najjeftiniji, i košta 1 zlatnik, što je više od nula. Dva zlatnika treba dati ili četvrtom, ili petom (svejedno je), što znači da prvi dobija 97 zlatnika sa 60% glasova.
Ovo je ko razgovor gluvih telefona, stalno primetim nešto tek kada sam poslala. :)
Mac, mora biti više od 50% protiv da bi se predlog odbacio. Znači - kod poslednje dvojice odlučuje pretposlednji.
Ufff...
Da, kad ostanu dvojica, D uvek može da računa na svih 100 zlatnika, pa će on uvek glasati protiv svakog drugačijeg perdloga, zato D još u početku ne mora dobiti ništa (rekoh D a ne D.)
A onda C može biti zadovoljan sa podelom 99+0+1 (ako ostanu trojica).
Mrzi me više da pišem... Ako preostane četvorica onda drugi potkupljuje četvrtog jednim zlatnikom, jer ako odbije onda ne dobija ništa, jer će treći potkupiti petog s jednim zlatnikom. Treći i peti u tom slučaju dakle ne dobijaju ništa. Prvi može da ih kupi sa po jednim zlatnikom, što znači da zadržava 98 zlatnika.
Dalje, B može proći sa podelom 98+0+0+2
A onda A ne može proći ni na koji način.
@mac:Zaboravio si treći uslov:
3) Ако је гусару свеједно да ли приједлог пропада или пролази (тј. ако у оба случаја добија исту количину новца), гласаће тако да предлагача пошаље у смрт.
@mouchette: :) Čini mi se da je onda dobro ono rešenje sa 96 za kapetana. Pretpostavljam da si u žaru rešavanja preskočila onaj moj post sa istom logikom.
Meni se ipak čini da mi je logika dobra. 98 zlatnika za kapetana, harg!
Za Midoto: Ali, B želi da dobije svojih 98, C svojih 99, D svojih 100, pa neće glasati da A ostane, znači A gine kako god predloži.
A E u zadnjem krugu ne dobija ništa, u predzadnjem, 1, itd, pa mu u prvom krugu mora ponuditi bar 3. Žao mi je kapetana, ali...
Quote from: mac on 08-02-2013, 17:20:53
Meni se ipak čini da mi je logika dobra. 98 zlatnika za kapetana, harg!
Na dasku!
Ako krenu od kraja, moraju da snize očekivanja. Pretposlednji shvata da - ako dođe do poslednja tri - on neće dobiti baš ništa...
Quote from: Слободан Павле on 08-02-2013, 15:57:39
Није, одговор је заправо прилично неочекиван (да не кажем суманут).
Još da se javi Harvi i potvrdi da je ovo moje rešenje dovoljno sumanuto za njegov ukus :lol:
Mouchette: ako je već pretposlednji odustao od svojih 100, zar ne bi prihvatio kapetanov predlog ako mu ovaj poveća dobitak za 1 zlatnik?
Za drugog logika nije 98+0+0+2 nego 99+0+1+0. Kako god da četvrti glasa sada, kad preostane trojica neće dobiti ništa. Njemu je cilj da dobije više od ništa i da ne preostanu trojica. 1 je više od ništa, i zato će četvrti glasati za predlog drugog.
Ista logika se onda primenjuje na prvog. Ako se odbije predlog prvog onda treći i peti ne dobijaju ništa, i zato će glasati za bilo koji predlog koji im daje više od ništa: 98+0+1+0+1.
Mac, tek sada vidim da si ti u pravu. Raspored bi stvarno trebalo da bude: 98, 1, 1, 0, 0.
Eh, da...
Честитам, народе! Рјешење је заиста 98 златника за капетана (за ове остале мислим да је Мац написао добру расподјелу). А ко би то могао да претпостави на први поглед, прије детаљне анализе?
Иначе, ово је задатак са једног онлајн курса из теорије игара, у склопу такозване backward индукције (то је овај метод да се крене од краја, тј. да се анализира шта ако су преостала само два гусара, па одатле да се иде према почетку). Ја сам приликом рјешавања добро почео са анализом, дошао сам до 99-0-1, али онда сам у сљедећем кораку забрљао и мислио да треба 98-0-0-2, а превидио сам 98-0-1-0.
Lep. :)
Quote from: Midoto on 08-02-2013, 18:02:39
Lep. :)
Хвала! А да ме тек видиш кад се почешљам! :evil:
:cry:
(ovo nije zbog frizure, već zbog onog 98-0-0-2 kod kog sam i ja zapela)
Ima li još tako lepih?
(zadataka)
Quote from: Слободан Павле on 08-02-2013, 18:05:12
Quote from: Midoto on 08-02-2013, 18:02:39
Lep. :)
Хвала! А да ме тек видиш кад се почешљам! :evil:
Ma baš zato što si razbarušen ko Jesenjin. :)
Mušet, evo jedan sa tvog omiljenog terena:
U pitanju je igra. Dva lika odlučuju da stavljaju čaše na sto - običan, pravougaoni; ne znamo dimenzije stola, niti dimenzije čaša. Onaj koji poslednji može da stavi čašu na sto, a da ona ne padne - pobeđuje. Ko ima pobedničku strategiju i kakvu.
'Ajde, mac, da te vidim!
Sećam se ovog od ranije, a i tada sam ga rešio :)
Pa, možda onaj koji stavlja drugi može da postavlja čaše simetrično u odnosu na prvog (centralno simetrično), pa ako ovaj prvi postavi, onda će to uspeti i on, a ako prvi ne može, onda drugi pobeđuje.
A evo, zauzvrat, jedan pravi pacerski :)
Na trećem spratu jedne zgrade živi vozač A. Njegov stan ima 3 prozora i 2 vrata. Na četvrtom spratu žive vozačeva braća: B, C i D. Stan B ima onoliko vrata koliko C prozora, a prozora onoliko koliko D ima vrata. Sva braća imaju jednak broj prozora i vrata. Da li tašta vozača A živi u stanu sa zetom?
I ovog se sećam 8-)
Mac je rešio, tek da se zna!
A ono moje rešenje? Čini mi se da je dobro, da se traži neko takvo rešenje.
Quote from: Mouchette on 08-02-2013, 20:17:15
Pa, možda onaj koji stavlja drugi može da postavlja čaše simetrično u odnosu na prvog (centralno simetrično), pa ako ovaj prvi postavi, onda će to uspeti i on, a ako prvi ne može, onda drugi pobeđuje.
Али шта ако овај први постави тачно на осу симетрије?
Пардон, промакло ми је ово "централно симетрично".
Дакле, шта ако стави тачно у центар?
Da, treba onda biti prvi i staviti na centar simetrije. Hvala, Павле.
Ал можда је то побједничка стратегија за првог?
Едит: Предухитрила си ме :-)
Bila sam na nekoj priredbi povodom kineske Nove godine, pa nisam stigla da napišem:
mac je odmah imao rešenje. (I to počinje da me sekira. Moraću da mu nađem neku žešću zavrzlamu.)
I ja znam ovaj sa taštom od ranije, pa ostaje onima koji su ga prvi put videli.
Da, čak i da ga nisi znala od ranije ipak bi ga rešila lako :)
Evo malo jednačina:
Jedan čovek je 1986. imao toliko godina koliko je iznosio zbir cifara njegove godine rođenja. Kada je rođen?
Midoto zna rešenje (tačnije - rešila je zadatak, da ne ispadne da se nas dve sećamo kad je čovek rođen :( ).
Quote from: Mouchette on 08-02-2013, 22:31:47
Da, čak i da ga nisi znala od ranije ipak bi ga rešila lako :)
Evo malo jednačina:
Jedan čovek je 1986. imao toliko godina koliko je iznosio zbir cifara njegove godine rođenja. Kada je rođen?
Човјек је очигледно рођен 19xy године. Његов број година 1986 износи 1986-19xy=86-10x-y. Из услова задатка долазимо до једначине
86-10x-y=1+9+x+y, односно
11x+2y=76
Једина комбинација једноцифрених бројева која задовољава горњу једначину је x=6 y=5. 1965
Quote from: Mouchette on 08-02-2013, 22:58:59
Midoto zna rešenje (tačnije - rešila je zadatak, da ne ispadne da se nas dve sećamo kad je čovek rođen :( ).
Šta - seća se? Bila s njim u obdaništu :)
Quote from: Midoto on 09-02-2013, 00:23:31
Šta - seća se? Bila s njim u obdaništu :)
Očekivala sam da će ga se bar neko setiti. Ali, čudi me da vozačevu taštu niko ne poznaje (osim Midoto, verovatno joj je žena u komšiluku :roll: ).
ispravka: mac je rešio zadatak sa taštom
Još jedan lak (nadam se da Alisu ne poznajete lično :roll: ).
U zemlji čuda na jednom stablu rastu banane i jabuke. Ima 200 jabuka i 99 banana. Alisa svakog dana ubere dva ploda i svaki put umesto ta dva ploda izraste jedan. Ako su ubrana dva ploda iste vrste, onda izraste jabuka a ako su ubrani plodovi različiti, izraste banana. Očigledno se svakodnevno smanjuje broj plodova i na kraju će ostati samo jedan. Da li će to biti banana ili jabuka?
Koliko se ja sećam onog razgovora sa Alisom, ona reče: banana. Nešto kao - invarijanta po pitanju parnosti? Ali davno to beše. :)
Pa da, šta god da se desi broj banana ostaje neparan, sve dok ne ostane samo jedna voćka - banana.
Znam, samo se šalim s Mouchette.
Dodatak: Mac, jesi li ti uvek tako veoma ozbiljan?
Nisam uvek ozbiljan, niti sam uvek neozbiljan, ali sam zato uvek Mac :)
Quote from: mac on 09-02-2013, 02:09:29
... ali sam zato uvek Mac :)
Zaista je ponekad neozbiljan.
A što se tiče priče o Alisi, da, na kraju će joj ostati banana, iz razloga koji su vam poznati.
Quote from: Mouchette on 09-02-2013, 02:22:54
Quote from: mac on 09-02-2013, 02:09:29
... ali sam zato uvek Mac :)
Zaista je ponekad neozbiljan.
Bije ga glas da je šeret - to mi reče ona vozačeva tašta koja mi nije komšinica, ali sa njom idem na aerobik. :)
Istina je da me majke više gotive od ćerki...
Verujem ti da tašta vozača A nije jedina. :)
Moram da dodam za one koji su rešavali zadatak (valjda je dovoljno vremena prošlo): tašta vozača A ne postoji.
Ala ste se raspisali... :)
Odličan je ovaj sa gusarima! Jedva čekam da ga postavim đacima na dodatnoj. :!:
Za ostale zadatke sam već čuo. Uzgred, ovaj sa taštom je prvi zadatak u prvom postu na prvoj strani ovog topika.
Evo od mene dva zadatka. To jest, ne od mene, već Od Luisa Kerola:
1. Guverner Kgovjnija želi da priredi vrlo malu večeru, i poziva očevog šuraka, bratovog tasta, tastovog brata i zetovog oca. Nađi broj gostiju.
2. L isplete 5 šalova, dok M isplete 2; Z isplete 4, dok L isplete 3. Pet šalova koje isplete Z teže koliko jedan koji isplete L; 5 koje isplete M teže koliko 3 koja isplete Z. Jedan koji isplete M topal je koliko 4 koja isplete Z; a jedan koji isplete L koliko 3 koja isplete M. Koja je pletilja najbolja, ako u rezultatu jednaku težinu imaju hitrina rada, lakoća i toplina?
Quote from: Ygg on 09-02-2013, 16:02:13
Koja je pletilja najbolja, ako u rezultatu jednaku težinu imaju hitrina rada, lakoća i toplina?
Z, ubedljivo (ako sam dobro razumela, ceni se veća brzina, veća težina i veća toplina).
Quote from: Mouchette on 09-02-2013, 16:42:15
Z, ubedljivo (ako sam dobro razumela, ceni se veća brzina, veća težina i veća toplina).
Ne. Cijeni se lakoća, a ne težina šalova. Pletilja koja plete lakše šalove je bolja. ("... Zuzuino pletivo tako je vilinsko da njenih 5 ne teže više od jednog koji isplete Lolo. Samo što Mimi ima čak i lakšu ruku - 5 njenih staju na tas spram 3 koja isplete Zuzu...")
E, onda ću da pogledam, ali još ovo: šurak je ženin brat? Ako je tako, onda je pozvao dvojicu.
Sa Vikipedije (http://sr.wikipedia.org/wiki/%D0%A1%D1%80%D0%BF%D1%81%D0%BA%D0%B8_%D1%81%D1%80%D0%BE%D0%B4%D0%BD%D0%B8%D1%87%D0%BA%D0%B8_%D0%BE%D0%B4%D0%BD%D0%BE%D1%81%D0%B8):
Шурак (понегде и шуријак) је женин брат.
Таст или пунац је женини отац.
Зет је ћеркин или сестрин муж.
Da, onda je šurak guvernerov ujak, u srodstvu je sa njim, a svi ovi ostali mogu biti ista osoba (koja ima brata, sina i ćerku).
Ygg, meni ovo sa lakoćom ne menja krajnji rezultat, M je najlošija, L bolja, a najbolja Z (3.2 puta bolja od L).
Quote from: Mouchette on 09-02-2013, 17:11:53
Da, onda je šurak guvernerov ujak, u srodstvu je sa njim, a svi ovi ostali mogu biti ista osoba (koja ima brata, sina i ćerku).
A da li je moguće da je gost samo jedan?
Quote from: Mouchette on 09-02-2013, 17:11:53
Ygg, meni ovo sa lakoćom ne menja krajnji rezultat, M je najlošija, L bolja, a najbolja Z (3.2 puta bolja od L).
Ne znam kako si to dobila, ali nije tačno.
Ako bi to bio samo jedan, onda bi npr. guvernerov brat bio oženjen ćerkom svog ujaka, dakle svojom bliskom rođakom, zato bih rekla da to ne valja.
A što se tiče onog drugog, L:M:Z = 15:6:20 (brzina), 25:3:5 (lakoća), 1:3:12 (toplina)...
Sunce vam žarko, čovek ne bude prisutan jedno popodne, a ovde čudo!
Meni je težina različita:
L : M : Z = 3 : 25 : 15
dok su mi ostale srazmere iste.
Pri tome je "pozitivno" što brže plesti i što više šalova staviti da bi nosio istu težinu, ali delim sa brojem šalova koji treba da te zagreju (tu mi ne valja porast, pa množim recipročno).
Na kraju za sve dobijem ovakav odnos:
L : M : Z = 45 : 50 : 25 = 9 : 10 : 5
Sad, meni se često desi da grešim u računu, ali u prvoj kontroli nisam našla grešku. Po meni je najbolja M, a najgora Z.
Uzgred, čini mi se da ovde "živne" nekako istovremeno sa zahuktavanjem takmičenja iz matematike. :) Navika je druga priroda...
Quote from: Mouchette on 08-02-2013, 22:31:47
Evo malo jednačina:
Jedan čovek je 1986. imao toliko godina koliko je iznosio zbir cifara njegove godine rođenja. Kada je rođen?
Морам да се пувам мало. Мој Маре решио ово за петнаестак минута, али напамет, методом елиминације. Претпоставио је да је човек рођен између 1960. и 1970. јер су иначе збирови постајали премали или превелики. Насмејао ме је кад сам му дао Васино решење: "Моје је брже, ово је компликовање."
(https://www.znaksagite.com/diskusije/proxy.php?request=http%3A%2F%2Fimg836.imageshack.us%2Fimg836%2F7075%2F20130210125057.jpg&hash=13ca9c9955205666cb0f2d15b7e514c28104124d)
Али са гусарима се већ данима мучи.
Zato što pokušava da reši aritmetički, a ne zelenaško-logički.
Evo još jednog zadatka za Mareta, sasvim je moguće da će ga rešiti pre nekog ko rešava algebarski.
Dva brata imaju zajedno 35 godina. Stariji brat ima dva puta više godina nego što je imao mlađi kad je stariji imao toliko godina koliko mlađi ima sada. Koliko je star svaki od braće?
E, kada bi zbir bio neki petocifren broj, onda bi se tek videla prednost algebre. Inače, ja lično uopšte nisam protiv 'prostijih' načina rešavanja u situacijama kada su od koristi. Čak mislim da je u svakodnevnom životu važno umeti optimalnim načinima rešavati tekuće probleme, u skladu sa njihovom složenošću, ne cimati se previše zbog nečega (nekoga) što se može nas lak način zaobići. Pritom ne kažem da su svi matematičari rigidni i komplikovani. Takvi su samo loši :)
Каже да је пошао од претпоставке да је број година старијег паран, а млађег непаран. Уз то, да су вероватно обојица стари између 15 и 20 година, што испаде и решење. Разлика не би могла да буде велика јер је збир свега 35.
(https://www.znaksagite.com/diskusije/proxy.php?request=http%3A%2F%2Fimg705.imageshack.us%2Fimg705%2F3775%2F20130210175646.jpg&hash=b247455a9288239d83027638eaeca9ac5efc6280)
Bravo, Mare! Svaka čast.
Dobra mu je i pretpostavka, tako da je praktično proveravao samo za 18, 20...34, što je za njegov uzrast odličan rezon. Stariji bi ga verovatno rešavali drugačije, ali rešenje je isto xjap
A1=2B2
A2=B1
B1=B2+X
A1+B1=35
A1-B1=A2-B2
2B2-B2-X=B2+X-B2
B2-X=X
B2=2X
A1=4X
B1=3X
7X=35
X=5 8)
Bato, super si se snašao sa tolikim nepoznatim. Moglo je i bez njih, na primer ako stariji ima x, mlađi ima 35-x. Ranije je stariji imao 35-x, pa je, dakle, to bilo pre 2x-35 godina (jer on sada ima x). Onda rešavamo samo ovu jednačinu
x = 2((35-x)-(2x-35))
Svejedno, na isto se svodi xjap
E, Mare je super. Koliko ima godina?
Evo još jednog slatkog koji može da rešava i Mare:
Imamo 4 različite tačke u ravni. Ako merimo rastojanje između ma koje dve, uvek dobijemo ili veličinu a ili veličinu b. Na koliko različitih načina možemo rasporediti te 4 tačke, a da uslov bude zadovoljen? Vrednosti a i b se razlikuju od slučaja do slučaja, ali je bitno da se ne pojavi i treća vrednost.
Ускоро ће осам и по и једва чека математичка такмичења за која се већ спрема, али то постоји тек од трећег разреда. Те задачиће већ кида, па су очекивања велика. Зато нам ова тема добро дође јер проберемо неке задатке с којима може да се избори, мада му јесу тешки. Однеће сутра ова два у школу да се пува и он ко тата на форуму. 8-)
Quote from: Джон Рейнольдс on 11-02-2013, 02:06:22
Ускоро ће осам и по и једва чека математичка такмичења...
(https://www.znaksagite.com/diskusije/proxy.php?request=http%3A%2F%2Fi50.tinypic.com%2Feg7l1.jpg&hash=fdf4c23527eb7765988acd15fe858103f0491f74)
Kako je "multiple choice" kad je pitanje samo sročeno tako da ima samo jedan odgovor?
Multiple choice je termin koji objašnjava da na pitanje na ispitu/ testu ne moraš sam da formulišeš odgovor već samo da izabereš jedan od ponuđenih.
Dobar je vic, mada mislim da poslednjih godina to više ne važi.
Vidim da nikome nije zabavan onaj zadatak sa četiri tačke (jedno od rešenja je kvadrat: jedna razdaljina je dužina stranice, druga je dijagonala). Evo vam onda nešto zabavnije (čula sam ga prošle godine od klinaca):
Biolozi vole da misle za sebe da su i hemičari; hemičari misle da su i fizičari. Fizičari misle da su bogovi, a Bog misli da je matematičar.
Midoto, lep je taj zadatak sa tačkama. Treba naći ćetvorouglove sa npr. 4 jednake stranice i dve jednake dijagonale (kvadrat), 4 stranice+jednom dijagonalom, a druga je različita (romb sa 60o) itd. Ima tu raznih mogućnosti, treba pobrojati sve (čak i u prostoru).
Za malog Mareta je ovo ipak teško.
Извињавам се, заборавио сам на задатак с тачкама (притисо посо, јбг). Позвао сам Марета и скоро као из топа је рекао - квадрат. Рекао сам му да покуша да нацрта Мушетин ромб и - гле стварно! :lol: Нисам сигуран ни да бих се ја тога сетио без мало жврљања по папиру. Али добро, и квадрат се рачуна.
Nešto mi se javlja da će Mare vrlo brzo početi da postavlja zadatke nama :lol:
Ево Маретовог задатка, извињавам се ако је овде био.
Како торту у три праволинијска реза исећи на осам делова?
Quote from: Джон Рейнольдс on 15-02-2013, 14:18:43
Ево Маретовог задатка, извињавам се ако је овде био.
Како торту у три праволинијска реза исећи на осам делова?
isečeš tortu na pola jednim rezom, zatim na četvrtine drugim, a onda je horizontalno prepoloviš trećim rezom?
Тачно.
едит: Плус Мидото на ПП. Мораћемо да смислимо нешто теже. :lol:
dobro, ovo je više "out of the box" razmišljanje nego matematika, ali nije loše.
evo ti za Mareta jedan sličan zadatak:
treba sa četiri prave linije da spoji sve tačke (stavići x umesto tačke, zbog vidljivosti)
x x x
x x x
x x x
ne znam kako će ispasti, ali spoljašnji x-evi čine kvadrat, i svaki x sa svoja susedna tri x čini manji kvadrat.
Mare je majstor.
@Mouchette: vidiš da je rešavao zadatak s tačkama :) Ti daroviti klinci umeju da iznenade. I ovaj poslednji zadatak je lep.
Evo vam težak (Mare ima iksiće, neće da se ljuti):
Imamo kompoziciju od deset vagona i svaki vagon prenosi zlatnike. U nekim vagonima su lažnjaci, ali ne znamo u kojima, niti kolika je masa lažnog zlatnika. Znamo da su svi pravi zlatnici međusobno iste mase, da su svi lažni takođe međusobno iste mase, ali različite od mase pravog zlatnika. Ako imamo digitalnu vagu i možemo iz bilo kog vagona da uzmemo koliko nam je potrebno zlatnika - u koliko najmanje merenja možemo otkriti masu lažnog i u kojim vagonima se oni nalaze?
Задао сам ово Томатово Марету пре извесног времена, није успео да реши сам. Али решење сада зна. Нека се занимају други, мада је то познат задатак.
Možda je u formulaciju trebalo ubaciti da linije smeju da se seku - verovatno bi rešio sam.
Midoto, taj zadatak sa novčićima sam prvi put čula pre mnogo godina, na fakultetu. Profesor ga je postavio na pauzi predavanja, da nas zabavi (kad sam ga rešila pomislio je da sam rešenje znala odranije). Lep zadatak.
A onaj sa tortom je o.d.l.i.č.a.n. :) Nisam ga znala.
Quote from: Midoto on 15-02-2013, 14:44:24
Imamo kompoziciju od deset vagona i svaki vagon prenosi zlatnike. U nekim vagonima su lažnjaci, ali ne znamo u kojima, niti kolika je masa lažnog zlatnika. Znamo da su svi pravi zlatnici međusobno iste mase, da su svi lažni takođe međusobno iste mase, ali različite od mase pravog zlatnika. Ako imamo digitalnu vagu i možemo iz bilo kog vagona da uzmemo koliko nam je potrebno zlatnika - u koliko najmanje merenja možemo otkriti masu lažnog i u kojim vagonima se oni nalaze?
Ceo vagon lažnjaci ili je mešano?
Svaki vagon prenosi jednu vrstu zlatnika - prave ili lažne. I ne mora da bude samo jedan sa lažnjacima - može da ih bude više. Treba da otkrijemo tačno - koji su sa lažnjacima i kolika je masa neispravnog zlatnika. Zapravo treba dokazati da je to moguće u dva merenja, a jedno nije dovoljno (sem u slučaju da svi vagoni nose ispravne zlatnike).
Izvinjavam se, ne znam jesam li naglasila: masa ispravnog zlatnika nam je poznata.
Ovo je opet nešto binarno...
Zadatak koji ja znam glasi:
U deset kutija imamo po deset zlatnika. U jednoj kutiji su zlatnici lažni. Svi pravi zlatnici su iste težine, lažni su 1g lakši. Možemo da obavimo samo jedno merenje na (digitalnoj) vagi. Treba otkriti u kojoj su kutiji lažni zlatnici. Kako?
Quote from: Mouchette on 15-02-2013, 17:11:58
Zadatak koji ja znam glasi:
U deset kutija imamo po deset zlatnika. U jednoj kutiji su zlatnici lažni. Svi pravi zlatnici su iste težine, lažni su 1g lakši. Možemo da obavimo samo jedno merenje na (digitalnoj) vagi. Treba otkriti u kojoj su kutiji lažni zlatnici. Kako?
Znam taj o kome pričaš. Ovaj što sam vam dala me je mučio dva dana (uslovno, umalo opet da strada ručak). I malo je zafrknut bez papira i olovke.
Mac je delimično u pravu - može i nešto binarno. I ja sam radila tako, jer je najjednostavnije.
Zar taj zadatak nije na jednoj od prethodnih strana topika? Odnosno, sigurna sam da jeste...
Nije, ovo je teško. Znaš samo koliko teži pravi zlatnik, a ne znaš koliko teži lažni, niti u kojim se sve vagonima nalazi. Lupam glavu, ali ne znam kako da u drugom merenju utvrdim težinu lažnog zlatnika.
Quote from: smp on 25-06-2012, 15:12:07
U deset kovčega bilo je u svakom po deset zlatnika.
U svim kovčezima bili su zlatnici od po 10 grama, osim u jednom gde su svi zlatnici bili od po 9 grama.
Imate vagu i treba samo jednim jedinim merenjem da ustanovite u kom su kovčegu lakši zlatnici.
Evo ga, znala sam da ga ima!!! Kasnije kreću ponuđena rešenja :lol: :lol: :lol: :lol:
Nije isto, zato što se zna da je samo u jednom kovčegu lakše zlato, i to tačno se zna težina lakšeg zlatnika. Ovde se ne zna u kojim vagonima je fejk zlato (može nijedan, može svi), niti se zna težina fejk zlata.
Meni pada na pamet ova ideja: da uzimamo iz svakog vagona po 2, 3, 7, 11, 17... zlatnika (tako da brojevi budu prosti i da nisu zbir prethodnih prostih brojeva). Na osnovu razlike očekivane težine i izmerene možemo tačno znati iz kog je vagona i kolika je težina lažnog.
Da, vidim da nije ok.
Quote from: mac on 15-02-2013, 18:24:20
Nije isto, zato što se zna da je samo u jednom kovčegu lakše zlato, i to tačno se zna težina lakšeg zlatnika. Ovde se ne zna u kojim vagonima je fejk zlato (može nijedan, može svi), niti se zna težina fejk zlata.
Ah :( matematika... :cry:
Mouchette, uzmimo da ti je to prvo merenje i dobila si, napr, odstupanje u ukupnoj masi – 5g. Kojim brojem treba da deliš? (Uzgred, uvek me fascinira tvoje netipično rešenje.)
To je slično onome što je Mac predložio – uzimamo redom stepene broja 2 (može i ma kog drugog prirodnog broja): iz prvog 1, iz drugog 2, pa 4,...
Samo, meni je ovo drugo merenje.
@Jevtropijevićka: vrlo čvrsto sam uverena da ljudi koji su i inače logični - samo nisu imali učitelje koji su na dobar način pristupili matematici.
Ma, nije do učitelja već do učenice... ja sam u osnovnoj i srednjoj školi matematiku baš volela i imala iz nje mnogo bolje ocene (4-5) nego iz hemije i fizike, bila mi je mnogo razumljivija i, jelte, logična, ali sam je potpuno i bez ostatka zabatalila čim sam maturirala, i - sad kad god svratim na ovaj topik, osećam se kao da sam zaboravila plivanje. :(
Quote from: Midoto on 15-02-2013, 19:15:35
Mouchette, uzmimo da ti je to prvo merenje i dobila si, napr, odstupanje u ukupnoj masi – 5g. Kojim brojem treba da deliš? (Uzgred, uvek me fascinira tvoje netipično rešenje.)
Ako znamo težine pravog i lažnog, onda nije problem. Tj da je 10/9g onda 5g manjka znači pet zlatnika, dakle iz petog vagona, ako smo uzeli iz prvog 1, iz drugog 2, itd...
Tj da smo uzimali prost broj ili stepen dvojke i razlika u težini bi morala da prati tu progresiju.
Al ako ne znamo težinu lažnog, stvarno, šta onda?
Ja sam u prvom merenju uzela iz svakog vagona po jedan zlatnik ili "zlatnik" i dobila neko odstupanje u težini. U drugom merenju sam uzimala stepene dvojke, da ne bi došlo do zabune (da Mušet nije postavila onaj uslov o prostim brojevima koji nisu zbir prethodnih, možda bi bila varijanta da je neki od tih prostih brojeva jednak zbiru nekih 5 ili 6 prethodnih).
Imamo jednoznačno rešenje.
Na primeru 3 vagona: ako pretpostavimo da samo jedan vagon nosi lažnjake - vidimo da li je razlika "ispravne" i izmerene mase daje odgovarajući rezultat pri deljenju sa 1, sa 2 ili sa 4. Ukoliko to nije slučaj, možda je u prvom i drugom, prvom i trećem ili dugom i trećem... U svakom slučaju, bitno je da ne mogu dobiti dva moguća rešenja, već samo jedno. Sa deset je isto to - samo zamršenije. E sad, kako bih dokazala da je nemoguće iz jednog merenja - nemam pojma.
Zadatak za treći razred osnovne (to valjda odgovara po planu i programu srbijanskom drugom razredu):
Jedno jato lasta
leti iznad hrasta,
drugo iznad smreke.
treće iznad reke.
Najveće je jato treće,
od prvog je dvaput veće,
a drugo je, reče Mira
polovina njihova zbira.
Sva tri jata,
šapnu Nata,
90 ima lasta.
Koliko lasta ima u kom jatu? Kako to riješiti a da tako mali klinci to razumiju? Zadatak se lako riješi preko jednačine x + 3x/2 + 2x = 90, ali em tu ima previše iksova za tako malu dječicu, em imamo čak i razlomak! Ne pada mi na pamet nikakvo elegantno i jednostavno rješenje.
Ako je drugo jato polovina zbira prvog i trećeg to je isto kao da si rekao da je zbir prvog i trećeg dva puta veći od drugog. Sva tri jata u zbiru su velika kao trostruko drugo jato (drugo jato plus dvostruko drugo jato). 90 = 3 puta drugo jato, pa je drugo jato 30 lasta. Postupak se ponavlja sa preostalih 60 lasta i dva jata, jer je opet jedan deo dva puta veći od drugog. U 60 lasta imamo trostuko prvo jato, to jest u prvo jatu ima 20, a u trećem preostalih 40 lasta.
(https://www.znaksagite.com/diskusije/proxy.php?request=http%3A%2F%2Fimg849.imageshack.us%2Fimg849%2F3016%2F20130227213754.jpg&hash=5d35fd6f0cf42206b823e6f451a612802ce5f628)
Da, da, ovo što predlaže mac - da drugo jato bude nepoznato, pa sve onda ide preko celih brojeva.
Ygg, moraš da zadaš nešto teže za Mareta :lol:
Hvala i mac-u i Maretu! Proslijedio sam rješenja. A evo za Mareta nešto dosta teže (autor je Luis Kerol):
Ako jedna čaša limunade, 3 sendviča i 7 piškota staju 1 šiling i 2 penija, a jedna čaša limunade, 4 sendviča i 10 piškota 1 šiling i 5 penija, nađi koliko staju:
(a) jedna čaša limunade, sendvič i piškota;
(b) 2 čaše limunade, 3 sendviča i 5 piškota.
Napomena: 1 (tadašnji) šiling ima 12 penija!
Lep zadatak. Možda će biti pretežak za Mareta, ali je interesantan i za starije.
Evo jedne šaljive pitalice za klince:
Riddle #281 - How Many Mice? (http://www.youtube.com/watch?v=O4ruKhmHVwA#)
Evo još nekih zabavnih zadačića (http://www.plastelina.net/game3.html) za decu svih uzrasta.
Ako se nekom dopadnu mozgalice ovog tipa, ovde je i torent za Plastelina Logic Games (http://thepiratebay.se/torrent/7157188/logic__amp_amp__flash_games)
Quote from: tomat on 15-02-2013, 14:39:34
evo ti za Mareta jedan sličan zadatak:
treba sa četiri prave linije da spoji sve tačke (stavići x umesto tačke, zbog vidljivosti)
x x x
x x x
x x x
ne znam kako će ispasti, ali spoljašnji x-evi čine kvadrat, i svaki x sa svoja susedna tri x čini manji kvadrat.
Quote from: Джон Рейнольдс on 15-02-2013, 14:46:36
Задао сам ово Томатово Марету пре извесног времена, није успео да реши сам.
Али решење сада зна. Нека се занимају други, мада је то познат задатак.
Realno, jeste poznat zadatak. Al ja nisam znao vise od jednog resenja.
(https://www.znaksagite.com/diskusije/proxy.php?request=http%3A%2F%2Fi1.wp.com%2Fwww.theawsc.com%2Fwp-content%2Fuploads%2F2012%2F09%2FNine-Dot_Puzzle_2.jpg&hash=1e5be573f910cea1568b0cff3008c5e7582d7694)
(https://www.znaksagite.com/diskusije/proxy.php?request=http%3A%2F%2Flateralaction.com%2Fbase%2Fmedia%2Fpost-images%2Fnine-dots-sol-1.png&hash=c1c6b0b0ecbc6050759e4b1d6bb2ed262f9da8f6)
A nikad mi nije palo na pamet resenje sa 3 linije.
(https://www.znaksagite.com/diskusije/proxy.php?request=http%3A%2F%2Fwww.jimloy.com%2Fpuzz%2F9dots1.gif&hash=da6120e92d0b277a2abf8f897fc83835ecf9e4eb)
Ако су стварно у питању тачке (а не кругови), ово рјешење са три линије није добро.
ovo je zadatak za ''out of box'' razmišljanje, svojevremeno kad sam prvi put naišao na njega nisam uspeo da ga rešim. :mrgreen: A ovo poslednje rešenje nije dobro. Gornja horizontalna linija je prava, a ne kosa.
Quote from: Слободан Павле on 28-02-2013, 16:32:01
Ако су стварно у питању тачке (а не кругови), ово рјешење са три линије није добро.
Svakako.
Quote from: Truman on 28-02-2013, 16:33:01
ovo je zadatak za ''out of box'' razmišljanje
Da, tako nekako je pisalo u jednom mom udzbeniku. I bili su krugovi.
zato ja i nisam umeo da ga uradim. :x
Quote from: Truman on 28-02-2013, 18:10:01
zato ja i nisam umeo da ga uradim. :x
Izmice mi... zasto?
Quote from: Boris on 28-02-2013, 00:04:50
Realno, jeste poznat zadatak. Al ja nisam znao vise od jednog resenja.
prva dva rešenja su suštinski jednaka, a treće - what Harvi said.
bio si u kutiji?
I dalje sam izgleda. :x
Aj, junaci da vidim kakvo ste znanje poneli sa sobom osmogodišnje škole. Neki novi (genijalni) izdavač misli da je u redu da ovakav zadatak metne u zbirku osmog razreda.
Automobil ide iz tačke A u tačku B, kreće se brzinom 60 km/h. Iz tačke B vraća se u tačku A brzinom 80km/h. Koja je prosečna brzina kretanja automobila?
S=v1*t1=v2*t2
Prosečna brzina je vp=2*S/(t1+t2) = 2*S/(S/v1 + S/v2)=2*v1*v2/(v1+v2)=68,57 km/h
Quote from: mac on 28-02-2013, 21:19:33
S=v1*t1=v2*t2
Prosečna brzina je vp=2*S/(t1+t2) = 2*S/(S/v1 + S/v2)=2*v1*v2/(v1+v2)=68,57 km/h
Штреберу!
Ne laskaj mu. To bi čak i ja rešio da nisam čitao Popular Science...
Imam ionako kompleks od fizike i fizicara, jos mi samo ovo fali...
Nemaš čega da se plašiš - ovo je matematika...
Quote from: tomat on 15-02-2013, 14:39:34
evo ti za Mareta jedan sličan zadatak:
treba sa četiri prave linije da spoji sve tačke (stavići x umesto tačke, zbog vidljivosti)
x x x
x x x
x x x
ne znam kako će ispasti, ali spoljašnji x-evi čine kvadrat, i svaki x sa svoja susedna tri x čini manji kvadrat.
a da li je (https://www.znaksagite.com/diskusije/proxy.php?request=http%3A%2F%2Fi47.tinypic.com%2F2hyc6jr.gif&hash=f84fabdeb5ce1a583e287cc9fe89e364b1296d7d) ok rešenje?
Postoji i uslov da se ne diže olovka sa papira.
Quote from: mac on 28-02-2013, 21:19:33
S=v1*t1=v2*t2
Prosečna brzina je vp=2*S/(t1+t2) = 2*S/(S/v1 + S/v2)=2*v1*v2/(v1+v2)=68,57 km/h
Odličan, pet (5) M.M
Znao da će mac ovo brzo rešiti. Trebalo je da ga zamolim da sačeka malo.
Quote from: mac on 01-03-2013, 09:47:01
Postoji i uslov da se ne diže olovka sa papira.
bilo bi lepo da ga je neko napisao -_-
Quote from: Miljan Markovic on 28-02-2013, 21:11:36
Aj, junaci da vidim kakvo ste znanje poneli sa sobom osmogodišnje škole. Neki novi (genijalni) izdavač misli da je u redu da ovakav zadatak metne u zbirku osmog razreda.
Automobil ide iz tačke A u tačku B, kreće se brzinom 60 km/h. Iz tačke B vraća se u tačku A brzinom 80km/h. Koja je prosečna brzina kretanja automobila?
Ovo je gradivo fizike za 6. razred i čak ne spada u teže zadatke (a taj deo fizike je, zapravo, totalno matematički). Pogledaj na sajtu DFS napr. drugi zadatak sa republičkog takmičenja za šestake 2010. godine. Kada sam proveravala da li je tačno urađen taj zadatak, bila sam uverena da postoji neki skriveni, lakši način, jer su mi se pojavljivale jednačine sa tri brzine i tri dela puta (nema, naravno). Ali neka deca su se odlično iskobeljala iz toga.
Quote from: Ygg on 27-02-2013, 23:18:58
Hvala i mac-u i Maretu! Proslijedio sam rješenja. A evo za Mareta nešto dosta teže (autor je Luis Kerol):
Ako jedna čaša limunade, 3 sendviča i 7 piškota staju 1 šiling i 2 penija, a jedna čaša limunade, 4 sendviča i 10 piškota 1 šiling i 5 penija, nađi koliko staju:
(a) jedna čaša limunade, sendvič i piškota;
(b) 2 čaše limunade, 3 sendviča i 5 piškota.
Napomena: 1 (tadašnji) šiling ima 12 penija!
Ево, потрајало је двадесетак минута. Коментар - глуп задатак, како нешто може да буде бесплатно? :-|
(https://www.znaksagite.com/diskusije/proxy.php?request=http%3A%2F%2Fimg845.imageshack.us%2Fimg845%2F2276%2F20130301214054.jpg&hash=705c424dcc3e28564e9af1ebea53c16e32a3895b)
Ово је неуспели покушај, кад је све имало неку цену, али бројеви нису били цели. Као, заокружили би то трговци. :)
(https://www.znaksagite.com/diskusije/proxy.php?request=http%3A%2F%2Fimg16.imageshack.us%2Fimg16%2F5426%2F20130301214437.jpg&hash=ec80d0314ff5b4480fb99e8e164726bb3da287d2)
Ово је, иначе, редак случај да је узео да решава поново. Обично се тврдоглаво држи свога, поготово ако мисли да је тачно. Међутим, овде је превагнуо здрав разум - како нешто може да кошта 0,3333333333, заокруживали трговци или не. Али одмах је видео, још кад је синоћ читао с екрана, да је 1С+3П=3.
Svaka čast za Mareta! :-|
I mala napomena: ne moraju piškote biti džabe, može npr. da piškota košta pola penija (ukoliko uopšte postoji kovanica od pola penija), sendvič 1,5 i limunada 6 penija.
Quote from: Miljan Markovic on 28-02-2013, 21:11:36
Aj, junaci da vidim kakvo ste znanje poneli sa sobom osmogodišnje škole. Neki novi (genijalni) izdavač misli da je u redu da ovakav zadatak metne u zbirku osmog razreda.
Automobil ide iz tačke A u tačku B, kreće se brzinom 60 km/h. Iz tačke B vraća se u tačku A brzinom 80km/h. Koja je prosečna brzina kretanja automobila?
Kao što Midoto reče, to je standardan zadatak i radi se kao primjer kad za srednju vrijednost nečega ne treba koristiti aritmetičku, već harmonijsku sredinu.
Evo tog tadatka za 6. razred osnovne škole:
2.
Три друга, Милан, Ненад и Петар, крећу истовремено из Великог у Мало место. Милан иде аутомобилом и први стиже у Мало место и одмах се враћа назад. У повратку, на растојању t1 = 25km од Малог места, срео је Ненада на мотоциклу и на растојању t2 = 32km Петра на бициклу. Следећи у Мало место стиже Ненад, одмах се враћа и на t3 = 8km од Малог места среће Петра. Колико је растојање између Великог и Малог места?
Moj utisak je da su deca sve pametnija. :)
Математичари, замолио мој Маре за помоћ. Играјући се са факторијалима, открио је следеће:
3! х 5! = 6!
6! х 7! = 10!
Већ сам писао да су му низови посебна забава и пошавши од претпоставке да "у математици ништа није случајно", покушао је да нађе неку законитост, то јест да следећи члан низа где производ факторијала даје неки трећи факторијал. Али узалуд, рвао се истина с неким километарским бројевима и на крају замолио вас за објашњење и помоћ.
Ја о овоме немам појма, али за рјешавање проблема (тј. налажење других сличних једнакости) може да се упрегне компјутер, који би се једноставним програмчићем натјерао да методом грубе силе проналази сличне кандидате.
Прије тога, ево још неких (очигледних) једнакости изведених по истом принципу као што је прва горе наведена:
4! х 23! = 24!
5! х 119! = 120!
итд.
Принцип је сљедећи: У једнакости a! x b! = c! претпоставимо да је b веће од а, а највеће ће наравно да буде c. Е сад, c! због особина факторијела увијек једнако производу b! и неког цијелог броја. На примјер, у првом примјеру, 6! = 6 х 5!. У другом - 10! = (10 х 9 х 8) х 7!. Једнакости као што су ове које сам горе навео генеришу се врло једноставно тако што узмемо било који цијели број, прогласимо га за а, узмемо његов факторијел, прогласимо га за c и онда за b узмемо c-1. (У суштини, користимо једнакост n! = n x (n-1)!)
Неке интересантније једнакости би могле да се добију тако што кренемо од броја c, узмемо нпр. (лупам) 45 или нешто слично и резонујемо овако:
45! = 45 х 44 х 43 х 42 х ... х 3 х 2 х 1, а ово је једнако 45 х 44!. Ако је 45 једнако факторијелу неког броја, имамо тражену једнакост, тј. ако је то неки број а, онда ће бити 45! = а! х 44!.
Наравно, одмах се види да 45 није једнако факторијелу ниједног броја, па онда му придружимо сљедећи фактор:
45! = (45 х 44) х 43 х 42 х ... х 3 х 2 х 1 = 45 х 44 х 43!. Сад се питамо да ли је 45 х 44 једнако факторијелу неког броја а. Ако јесте, имамо једнакост 45! = а! х 43!, а ако није (а није) онда идемо на 45 х 44 х 43 и тако даље. Ако је уопште могуће наћи два броја таква да им производ факторијела даје 45! факторијел, овај једноставни алгоритам ће их пронаћи. Наравно ако компјутер не експлодира од преоптерећења.
Извињавам се ако сам штогод лупио, касно је, а спремам вјежбе :-)
Е, него, сад ми паде на памет кад сам погледао Маретове једнакости:
6! х 7! = 10!, али 6! = 3! х 5!. Дакле, 10! = 3! х 5! х 7!. Пошто је 1=1!, можемо да напишемо и 1! х 3! х 5! х 7! = 10!. Питам се има ли ово какву генерализацију...
n! x (n!-1)! uvek daje ovo što tražiš
da, već je gore to pomenuto, sad sam videla
Sudeći po ovom topiku (http://mathforum.org/kb/message.jspa?messageID=3400511) i odgovoru na pitanje sa math.stackexchange.com (http://math.stackexchange.com/questions/112670/on-the-factorial-equations-a-b-c-and-abc-d) nema mnogo tih netrivijalnih slučajeva. Prvi spada u trivijalne, drugi se navodi u odgovoru, a preostala dva u odgovoru su 9!=7!3!3!2! i 16!=14!5!2!. To je to, nema ih više.
Glavno ograničenje (i olakšavajuća okolnost za računarsku pretragu) je da svi prosti brojevi u rezultujućem faktorijelu moraju da se nađu i u proizvodu faktorijela, to jest veći od dva faktorijelna činioca mora da sadrži najveći prosti broj koji se sadrži u rezultujućem faktorijelu. Zaključak: ako pretragom koju opisuje Viktor dođeš do prostog broja (kao što je 43) dalje više ne moraš ni da tražiš, jer nema rešenja.
Хвала, људи. Ево јутрос сам му искрено честитао јер је у суштини схватио правило, али га је исказао речима тима - треба наћи факторијал који би био заправо следећи у низу оног другог чиниоца. Међутим, није се трудио да га баци на папир у виду формуле јер га је збунило то нетривијално решење које се није уклапало и онда је кренуо "пешке" да тражи неко друго правило које очигледно не постоји. Занимљиво је да га привлаче велики бројеви. Тако је случано набасао на 6!7!=10! - множио је у школи с другаром не би ли направио таблицу првих десет факторијала (и да му прође време), па пошто му је овај био само сајдкик, задао му је "домаћи" лупивши два броја. Потом да би проверио решење, помножио је и одушевио се - хеј, ово је 10!
Учитељ им даје да се баве разним стварима јер има њих тројица одликаша који су несносни, Банда одличних ученика се зову, и пошто су себи привукли још неколицину сад их је критична маса, много су јаки, прозивају и малтретирају децу, туку јаче од себе (чудновата замена улога), смишљају грубе игре и често им се гледа кроз прсте због успеха, али изгледа да је за неке ствари стрпљење при крају. Организоваће им сад неко такмичење логичких задатака у сарадњи с Архимедесом, "Мислиша" се зове, па ће бити мира недељу дана.
a za čega ti faktorijali uopšte služe?
Služe svačemu, ali najviše služe tome da matematičari imaju šta da rade, kao što im služi i ostatak matematike. Pogledaj članak na Vikipediji, odeljak Applications (http://en.wikipedia.org/wiki/Factorial#Applications)
Quote from: Alexdelarge on 11-03-2013, 10:50:25
a za čega ti faktorijali uopšte služe?
Da, recimo, izračunaš na koliko načina možeš poređati svoje knjige na policu. Ali, kako lepo reče mac, verovatno imaš nešto pametnije da radiš :(
Ja ih uvek tako poređam da posle ne mogu da ih nađem... :(
Eh, Mico, ti tvoji načini su neprebrojivi
Ono što je mac pomenuo, da matematika postoji kako bi matematičari imali šta da rade :lol: , lepo je objašnjeno i u ovom tekstu. Šta se dogodilo sa matematikom u proteklom veku, zašto tradicionalni pristup opstaje u školama, o značaju zakona isključenja trećeg i logike uopšte... o tome kako je matematika lepša od filozofije jer, kako Hilbert kaže, 'mathematics is a game played according to certain rules with meaningless marks on paper"... i o mnogim drugim lepotama iste. (http://www.ams.org/notices/201201/rtx120100031p.pdf)
Pade mi na pamet, Džone, možda si ti negde i spominjao, ali mislim da bi trebalo Mareta da pošalješ, ako to još uvek funckioniše, na časove u Klub mladih matematičara Arhimedes.
Jedan zabavan, ne preterano težak zadatak (u mom slobodnom prevodu):
Pera je uhvaćen u krađi, sudi mu bivša žena. Ona prema njemu i dalje gaji simpatije (rekoh, zadatak je zabavan), ali je pravedna i izvršiće propisanu kaznu: pucaće dva puta u njega iz revolvera koji ima dva metka. Ona mu saopštava da će metke staviti jedan do drugog, a zatim zavrteti burence i pucati jednom, a onda još jednom. Pera je imao sreće i preživeo je prvi pucanj. Žena ga pita da li želi da opet zavrti burence ili da puca odmah, bez okretanja. Šta je bolje da Pera odabere?
Zadatak je matematički, dakle otpadaju odgovori tipa 'nek odabere pancir' ili 'nek brzo trči'...
Znam rešenje, ali neću da kažem :)
Quote from: mac on 16-03-2013, 15:46:11
Znam rešenje, ali neću da kažem :)
:)
Priznaj da ti je žena sudija
I sudija, i porota, i dželat. Još samo da je nađem...
Neoženjen? Nisam nikada ni sumnjala u tvoju pamet.
Quote from: Mouchette on 16-03-2013, 15:00:22
Jedan zabavan, ne preterano težak zadatak (u mom slobodnom prevodu):
Pera je uhvaćen u krađi, sudi mu bivša žena. Ona prema njemu i dalje gaji simpatije (rekoh, zadatak je zabavan), ali je pravedna i izvršiće propisanu kaznu: pucaće dva puta u njega iz revolvera koji ima dva metka. Ona mu saopštava da će metke staviti jedan do drugog, a zatim zavrteti burence i pucati jednom, a onda još jednom. Pera je imao sreće i preživeo je prvi pucanj. Žena ga pita da li želi da opet zavrti burence ili da puca odmah, bez okretanja. Šta je bolje da Pera odabere?
Zadatak je matematički, dakle otpadaju odgovori tipa 'nek odabere pancir' ili 'nek brzo trči'...
ovaj je lak, jer je sansa za pogodak iz prve 2:4 (valjda je u pitanju klasican 6-shooter) a sansa za pogodak posle prvog promasaja 2:3 (sad su ostala samo 3 prazna mesta u burencetu).
Quote from: Berserker on 16-03-2013, 18:20:13
ovaj je lak, jer je sansa za pogodak iz prve 2:4 (valjda je u pitanju klasican 6-shooter) ...
Verovatno si hteo da kažeš da je (sa obrtanjem) verovatnoća da bude pogođen 2:6, tj. da preživi 4:6 (kao i kod prvog pucnja)?
Onda bi tvoja druga verovatnoća (bez obrtanja) bila 2:5, (da preživi 3:5), ali nije!
Ево задатка који је данас био забава. Фали и питање - да ли је уопште могуће решити?
(https://www.znaksagite.com/diskusije/proxy.php?request=http%3A%2F%2Fimg248.imageshack.us%2Fimg248%2F6087%2F20130320183532.jpg&hash=495a56da0915150d6cdfc32f5ae93bc57b7f65a3)
"Дужи" - мисли се на све црте између тачака пресека, наравно, не странице, итд.
Ovde imamo tri površi ograđene neparnim brojem duži. Pošto imaju neparan broj duži onda linija koja preseca te duži mora da započne iz tih površi (ili završi u njima). Pošto takvih površi ih ima više od dve onda ne postoji rešenje sa jednom linijom, jer linija ima samo dva kraja.
Маре има (контровезно, додуше) решење. Са много варијанти.
У старту ми се учинило да је задатак од thinking outside the box феле, па ако се захтев на одређени начин интерпретира, може и ово:
(https://www.znaksagite.com/diskusije/proxy.php?request=http%3A%2F%2Fimg703.imageshack.us%2Fimg703%2F6%2F20130320205109.jpg&hash=4cef9b8e362fe935af3f7b4d1f39271215772a57)
Tja, ovo seče dvaput, a na jednom mestu. Dobra ideja, u svakom slučaju. Možda Mare jednom reši i Kobajaši Maru :)
zaebano, nisam provalio
Проблем је у томе што не знамо одакле је овај задатак, јер га је Марету пренео један другар, а вероватно је извучен из неког енигматског часописа или сл. Деца су то пренела, па не знам како тачно гласи, можда је трик у томе да каже "сече на једном месту" или "у једној тачки", а не "једном". И онда би овакво решење било валидно. А опет, веома је било корисно твоје (едит: mac) објашњење зашто не може ако се посматра стриктно "једном", тако да... хвала несразмерно. :) Велика је свађа испала око тога у школи. :lol:
ja nešto potražio na netu, kažu ne može da se reši.
Answer: Unfortunately, this, one of the most popular classic puzzles, has no solution. At least, one wall always will be left unpassed.
It was easy proved by Martin Gardner. The proof (adopted to our case with the walls and rooms) is as follows: <<A continuous line that enters and leaves one of the rectangular rooms must of necessity cross two walls. Since the three bigger rooms have each an odd number of walls to be crossed, it follows that an end of a line must be inside each if all the 16 walls are crossed. But a continuous line has only two ends, so the puzzle is insoluble.>>
edit: ulete tomat, ali to je to...
Najpre, dugujem rešenje za onaj zadatak sa revolverom:
Quote from: Mouchette on 16-03-2013, 15:00:22
Pera je uhvaćen u krađi, sudi mu bivša žena. Ona prema njemu i dalje gaji simpatije (rekoh, zadatak je zabavan), ali je pravedna i izvršiće propisanu kaznu: pucaće dva puta u njega iz revolvera koji ima dva metka. Ona mu saopštava da će metke staviti jedan do drugog, a zatim zavrteti burence i pucati jednom, a onda još jednom. Pera je imao sreće i preživeo je prvi pucanj. Žena ga pita da li želi da opet zavrti burence ili da puca odmah, bez okretanja. Šta je bolje da Pera odabere?
Ukratko (bez mnogo teorije o uslovnoj verovatnoći i nezavisnosti):
Ako se burence okrene, verovatnoća da bude upucan je 2/6 , tj. oko 33% (jer imamo dva metka na šest mogućih položaja).
Ako je prvi bio promašaj, onda je verovatnoća da u drugom bude upucan bez okretanja 1/4, tj. 25% (jer za dva susedna položaja ima 6 mogućnosti: 11, 10, 00, 00 00, 01, a pod pretpostavkom da je prvi bio 0, imamo samo jednu od 4 mogućnosti da u drugom bude pogođen).
Dobro, to ne znači da će se izvući ako ne izabere okretanje burenceta, ali znači da, ako ponavlja krađu npr. 100 puta, možemo očekivati da će u prvom slučaju biti ubijen 33, a u drugom 25 puta :evil:
A sad nešto zanimljivo za ljubitelje čokolade - kako je jesti u nedogled!
(https://www.znaksagite.com/diskusije/proxy.php?request=http%3A%2F%2Fi.imgur.com%2FKuZrnrr.jpg&hash=e11806f57fe90b793a0873799e802ef1a7fd9871)
Ili:
(https://www.znaksagite.com/diskusije/proxy.php?request=http%3A%2F%2Fi.imgur.com%2FqjjlkGm.gif&hash=c6f146e0410e3f26c2a20d48f58632f29e1e881d)
Ево још мало пувања. Маре међу првима на "Архимедесовом" такмичењу. Тачније, дели прво место (редни број 45, нема преваре, нема лажи) међу 240 деце која су освојила максимални број бодова. Учествовало је 9.186 другака. Сви подаци су на сајту http://www.mislisa.rs/ (http://www.mislisa.rs/) .
Сад, не бих ја био ја да критички не анализирам табеле. Такмичење су локално организовале саме школе, те се десило оно што сам некако и предвидео. Нпр, у Маретовој конкуренцији чак 12 деце освојило је максималан број бодова из неке школе из Каравукова. Е сад, можда је то Каравуково (Где је то? Кара-вуково - вуко-јебина, WTF?!? нашао сам, мало западно од Оџака) заиста расадник младих математичара, а можда им је неко решавао задатке. Исто тако, има још неких школа које су дале много тих с првим местима, док ето Маретов "Панчић", по успеху најбољи на Чукарици, једног на првом и једног на другом месту. А у "Панчићу" није било зезања с преписивањем, помагањем, итд. Наставници су поштено одрадили посао.
Било како било, финале надзиру сами "Архимедесовци", ту ће се видети ко је ко, па чекамо 21. април.
Друга занимљива ствар је драстично опадање броја учесника по разредима. Ево за оне који не би да се зајебавају по сајту, занимљиво је (бар мени јер сам поносни ћале, иначе бих одмахнуо руком јер све је јасно), дакле број учесника од другог разреда основне, па до четвртог средње:
9.186 8.556 7.986 4.273 3.168 2.462 1.771 37.402 1.246 1.083 825 785
Драстичан је пад броја учесника између четвртог и петог разреда. Јасно, клинце све више занимају Карлеуша, "Фарма", дрога, Драгојевићеви филмови, улазак у Европску унију, "Свет" и "Скандал" него тамо нека математика. Али тако ваљда и треба да буде.
Свака част! :!:
Bravo!
(...ne treba potcenjivati značaj specijalizacije. Možda se klinci u starijim razredima okreću fizici, astronomiji... nije Karleuša koren svega zla..)
Quote from: Mme Chauchat on 01-04-2013, 01:02:52
Bravo!
(...ne treba potcenjivati značaj specijalizacije. Možda se klinci u starijim razredima okreću fizici, astronomiji... nije Karleuša koren svega zla..)
Свакако. :lol: Морао сам некако у причу да убацим "улазак у ЕУ", па ето... Тих девет сома је претерано, клинце натерају родитељи или наставници, јер ту неколико деце из окружења учествовало је само због тога и наравно нису урадили ништа.
Quote from: Джон Рейнольдс on 01-04-2013, 00:47:27
Маре међу првима на "Архимедесовом" такмичењу...
xjap
Zanimljivi su zadaci na tom takmičenju, nije tu dovoljno nabubati formule i obrasce i šablonski ih primeniti. Čestitke Maretu!
Quote from: Mouchette on 01-04-2013, 17:54:49
Quote from: Джон Рейнольдс on 01-04-2013, 00:47:27
Маре међу првима на "Архимедесовом" такмичењу...
xjap
Zanimljivi su zadaci na tom takmičenju, nije tu dovoljno nabubati formule i obrasce i šablonski ih primeniti. Čestitke Maretu!
I ja se pridružujem čestitkama i ovoj karakteristici zadataka koje deca rešavaju na "Misliši". Navijam za Mareta u finalu :)
Slažem se i sa Jevtropijevićkom (iliti gospođom Šoša - samo ne mogu da zamislim Jevtropijevićku kako treska vratima dok ulazi u prostoriju, kao što to Klavdija čini :) ) da se manja deca okušavaju u svemu, a vremenom shvate šta ih više zanima ili ne zanima, u čemu su uspešniji...
Quote from: Midoto on 02-04-2013, 10:03:50
Slažem se i sa Jevtropijevićkom (iliti gospođom Šoša - samo ne mogu da zamislim Jevtropijevićku kako treska vratima dok ulazi u prostoriju, kao što to Klavdija čini :) )
A to nam je, uz kose oči, glavna sličnost :mrgreen:
Zdravo svima. Evo jednog (zanimljivog) zadatka, koji je mnogo lakši nego što izgleda na prvi pogled. Formulisan sinoć u zadimljenoj kafani u kojoj diplomirani PMF-ovci (a sada nezreli ljudi u zrelim godinama), kukaju kako se ne bave strukom, a nekad su znali sve što se moglo znati iz topologije i diferencijalne geometrije (besramna laž).
Elem, na zidu je prozor. Gledamo njegovo staklo (pravougaonik). Tjemena su mu A (donje lijevo), B (donje desno), C (gornje desno), D (gornje lijevo), a dužine stranica a(=AB=DC) i b(=AD=BC). Fore radi, neka bude da je b>a (čisto da liči na pravi prozor). Neka iz tjemena C kreće zrak svjetlosti. Taj zrak zaklapa sa stranicama DC i CB ugao od 45 stepeni (tj. simetrala je ugla DCB). Nakon toga, zrak udari u stranicu AD. Od te stranice se zrak odbije pod pravim uglom unutar pravougaonika (tj. ulazni i izlazni zrak zaklapaju ugao od 90 stepeni). Onda udari u stranicu CB. Od nje se opet odbije pod pravim uglom, i nakon odbitka udari u stranicu AD. I tako dalje, zrak se od stranica stalno odbija pod pravim uglom. U jednom trenutku, zrak će se odbiti i udariti u stranicu AB (moglo je to da bude već i u drugom koraku, ali ja sam stavio da se odbio par puta čisto da se shvati zadatak). No prob - i od stranice AB se odbija pod pravim uglom na isti način na koji je opisano gore. I tako se zrak stalno odbija unutar stranica pravougaonika... ali do kada?
Da li će, u opštem slučaju i za proizvoljno a i b, zrak ikad završiti u bilo kom od tjemena pravougaonika nakon konačno mnogo odbijanja ili će se tako odbijati od zidova pravougaonika do kraja vremena? Ako hoće - zašto, a ako neće - u kojim specijalnim slučajevima hoće?
Mislim da je ovo nivo srednje škole, pri čemu je rješenje elegantno, a zadatak fin. Pozdrav.
P.S. I da se pridružim čestitkama :)
Au, dokonog li sveta. Šta da očekuješ u kafani, do ovakvih, elegantnih problema.
Bolje da si ovo postavio kao problem sa bilijarem. Jer, brat bratu, ništa te nisam razumeo.
Ali, taj kafić nema bilijarski sto! Kvariš radnju fabule... :)
Ne znam kako da dokažem, ali mislim da količnik a/b treba da bude prikazan kao binarni broj sa konačnim brojem cifara (decimala u binarnom sistemu). U pitanju je binarni broj jer je zrak poslat pod uglom od 45 stepeni. Ako količnik ima konačan broj cifara onda će zrak jednom završiti u nekom uglu.
Baš lep zadatak. Meni, opet, vuče na simetriju, ali moraću da ga rešavam u prevozu.
@Jevtropijevićka: I isturene jagodice i zarazan smeh, ali lupanje vratima - jok.
Moje rešenje nije tačno. Nema potrebe za binarnim sistemom. Ako je a/b racionalni broj onda će svetlost jednom završiti u uglu.
Dok se vratih - Mac već napisao ono što sam i ja nameravala.
Ja sam rešavala (u početku) preko niza osnih simetrija, dok nisam dokonala da mogu prosto da napravim mrežu pravougaonika.
Mislim da ovaj zadatak mogu da rade i stariji osnovci (čim odrade racionalne brojeve).
Quote from: Джон Рейнольдс on 01-04-2013, 00:47:27
Нпр, у Маретовој конкуренцији чак 12 деце освојило је максималан број бодова из неке школе из Каравукова. Е сад, можда је то Каравуково (Где је то? Кара-вуково - вуко-јебина, WTF?!? нашао сам, мало западно од Оџака) заиста расадник младих математичара, а можда им је неко решавао задатке.
... danas potpuno, ali potpuno slučajno naiđem na ovaj blog i na jedan određen unos:
http://tatjanakovacev.wordpress.com/2012/12/24/%D1%83%D1%87%D0%B8%D1%82%D0%B0%D1%98-%D0%BA%D1%9A%D0%B8%D0%B3%D1%83/ (http://tatjanakovacev.wordpress.com/2012/12/24/%D1%83%D1%87%D0%B8%D1%82%D0%B0%D1%98-%D0%BA%D1%9A%D0%B8%D0%B3%D1%83/)
Dakle, ovi rezultati s takmičenja su jednostavna potvrda toga šta sve može motivisana i inventivna učiteljica da postigne sa decom. :-|
Тјах. Сад ће још да испадне да је мој цинични и суморни поглед на свет неоснован. :lol:
@
Bravo mac i Midoto. Ja sam ga isto riješio tako što sam napravio mrežu pravougaonika (odnosno jedan jaako visok pravougaonik). Zašto? Zato što se pokaže da zrak koji prolazi kroz dno (kad kažem dno mislim na AB) i udara u novi pravougaonik formira trougao podudaran sa onim koji formira zrak koji se odbija od dna i vraća nazad u pravougaonik po uslovu zadatka. Tako je onda ovaj problem ekvivalentan probleu da li će zrak ikada udariti u jedno od dva tjemena produženog pravougaonika, pri čemu ga nadograđujemo do kada hoćemo. Dakle, ako pravougaonik visine b nadogradimo k puta ispod osnovnog pravougaonika, da li će to ikada biti jednako a*n, jer ova linija koja ide "cik-cak" na stranicama pravougaonika formira udara u tačke koje formiraju niz dužina a+2a+2a sa lijeve strane pravougaonika, odnosno 2a+2a+2a... sa desne strane (zašto ovaj niz? Zato što su ovo dužine stranica koje se dobijaju primjenom Pitagorine teoreme na trouglove u pravougaoniku prije odbijanja o njegovo dno). Dalje je lako: bk=an isto kao i a/b=k/n, pri čemu je k/n racionalan broj (jer su k i n prirodni), ali a/b ne mora biti, jer su proizvoljni. I stvarno, ako je a/b iracionalno, zrak vječno udara u zidove pravougaonika (slučaj kada je, recimo, a korijen iz dva, a b jedan, jer korijen iz dva nije racionalan broj, kao što znamo iz Pitagorinog dokaza). Ako je a/b racionalno, kao što reče mac, udara, i to je lako izračunati i broj odbijanja. Svaka čast.
Čestitam Maretu, treba se izboriti pored Karavukovaca 8-)
ali baš mi nije jasno to da roditelji prijavljuju djecu, otkad je to praksa, kad sam ja išao nastavnik je birao učesnike takmičenja, realno me nije ni pitao da li hoću da idem
Quote from: Джон Рейнольдс on 01-04-2013, 00:47:27
Такмичење су локално организовале саме школе, те се десило оно што сам некако и предвидео. Нпр, у Маретовој конкуренцији чак 12 деце освојило је максималан број бодова из неке школе из Каравукова. Е сад, можда је то Каравуково (Где је то? Кара-вуково - вуко-јебина, WTF?!? нашао сам, мало западно од Оџака) заиста расадник младих математичара, а можда им је неко решавао задатке.
Maretu čestitam, a ovo što sam citirao je potpuno
odvratno, na mnogo (odvratnih) načina.
The magic of Vedic math - Gaurav Tekriwal (http://www.youtube.com/watch?v=grkWGeqW99c#ws)
Маре вицешампион Србије. Прва награда, али један дечак испред њега за пола бода. Само су њих двојица освојили прве награде.
Маре пао на највећој могућој глупости, ладно није довољно прецизно прочитао најлакши задатак. :x Фалило му је једно просто 6-1 да га уради до краја.
:| :| :|
Svaka čast!
Bravo Mare!
za sve je kriva Sagita, samo teške zadatke ste mu davali, nije se spremao za lake xnerd
Čestitam!
Rispekat za Marea.
Čestitamo!
Хвала свима. Веома сам задовољан резултатом јер, колико год успех био, такође је и опомена. Скоро се уопште није спремао уз изјаву "пих, колико тешки задаци за други разред могу да буду". А солидно тешки задаци тражили су и исцрпна објашњења (укупно на три А4 странице), тако да му је концентрација пала до задатка 5д, последњег иначе. Вероватно је и тактика била лоша, требало је да уради сва решења, па онда напише образложења. Овако је предао у самом финишу и збрзао тај лак, последњи. Али добро, свака школа се плаћа. Прошле године није било ниједне прве награде; ове године, две. Две друге; ове године чак пет.
свака част!
http://www.wimp.com/germansuperbrain/ (http://www.wimp.com/germansuperbrain/)
E tako se to radi :lol:
(Osim ako nije fejk, sto me mrzi da guglam)
sinoć sam zakačio delić emisije na rubu znanosti posvećen vedskoj matematici. gost emisije je pokazao neke fore&trikove kod računanja, koji su, pretpostavljam, matematistima ovdašnjim dobro poznati, ali mene su oduševili.
npr. kako lako sračunati 103x104
na prvi broj 103 dodati 4 , 103+4=107
i pomnožiti 3x4=12
u konačnom to je 10712
Quote from: Alexdelarge on 18-06-2013, 09:57:13
sinoć sam zakačio delić emisije na rubu znanosti posvećen vedskoj matematici. gost emisije je pokazao neke fore&trikove kod računanja, koji su, pretpostavljam, matematistima ovdašnjim dobro poznati, ali mene su oduševili.
npr. kako lako sračunati 103x104
na prvi broj 103 dodati 4 , 103+4=107
i pomnožiti 3x4=12
u konačnom to je 10712
To nije lakši način čim ima isti broj koraka kao i klasično množenje: 103x100, 103x4 i sabiranje ova dva proizvoda.
slicno kao ovo:
Simple graphical multiplication trick (http://www.youtube.com/watch?v=kzEOcZ-oulc#)
Nije ništa drugo do grafička reprezentacija klasičnog množenja.
http://mathwithbaddrawings.com/2013/06/16/ultimate-tic-tac-toe/ (http://mathwithbaddrawings.com/2013/06/16/ultimate-tic-tac-toe/)
(https://www.znaksagite.com/diskusije/proxy.php?request=http%3A%2F%2Fmathwithbaddrawings.files.wordpress.com%2F2013%2F06%2F20-end-game1.jpg%3Fw%3D300%26amp%3Bh%3D225&hash=c91f4caf0cb13818b8ae088b1f171f62bb690182)
Imamo tri novčića i bacimo ih. Koja je verovatnoća da dva oda ta tri budu glava?
37,5%
na testu su opcije bile: 30, 50, 90 posto. Kako si došao do toga?
Svi mogući ishodi kod bacanja 3 kocke su PPP, PPG, PGP, PGG, GPP, GPG, GGP, GGG, pa je verovatnoća da padnu 2 glave 3/8 tj. 37,5% (0,375). A o kakvom se testu radi?
Quote from: Mouchette on 29-06-2013, 12:11:07
Svi mogući ishodi kod bacanja 3 kocke su PPP, PPG, PGP, PGG, GPP, GPG, GGP, GGG, pa je verovatnoća da padnu 2 glave 3/8 tj. 37,5% (0,375). A o kakvom se testu radi?
Очигледно - лошем! :lol:
Quote from: Mouchette on 29-06-2013, 12:11:07
Svi mogući ishodi kod bacanja 3 kocke su PPP, PPG, PGP, PGG, GPP, GPG, GGP, GGG, pa je verovatnoća da padnu 2 glave 3/8 tj. 37,5% (0,375). A o kakvom se testu radi?
Neki na netu, očito je fejk...
Quote from: Truman on 29-06-2013, 11:47:54
na testu su opcije bile: 30, 50, 90 posto. Kako si došao do toga?
Onda su, verovatno, računali i onu kombinaciju GGG - gde su prisutne dve glave (mada je i treća). Iako je neprecizno pitanje, često ponuđeni odgovori upućuju kakva je zamisao.
492, 366, 189, 810,?
a) 1260
b) 675
c) 88
d) 1015
88
objašnjenje?
Ako je poslednji broj xyz to jest 100*x+10*y+z onda je sledeći broj 10*x*y+x+z
boles'
Pametnjakovići, šta mislite - može li iq da skoči rađenjem logičkih zadataka? Pazario sam zbirku zadataka ''Povećajte svoj iq'', izdavač Mladinska knjiga. Nadam se da bi mi to moglo pomoći pri rađenju testova za posao...
Vežbom zadataka bićeš sve bolji u rešavanju zadataka. Ali IQ se ne svodi samo na rešavanje zadataka.
na šta se još svodi? IQ i inteligencija naravno nisu isto, ali generalno iq se meri preko tih zadataka.
Ma nije meni ni cilj da budem inteligentniji, samo hoću što bolje da odrađujem te testove.
hoćeš da varaš, recimo? 8-)
Quote from: mac on 22-09-2013, 22:01:45
Ako je poslednji broj xyz to jest 100*x+10*y+z onda je sledeći broj 10*x*y+x+z
ok, to je formula, ali kako se dolazi do te formule?
Naravno da neću da varam. Ovo što ja radim nije varanje. To bi bilo kao kad bi rekao da varam u pokeru, jer čitam knjige o toj igri.
Ili da varaš u trčanju time što trčiš :lol: :lol: :lol:
Ali, Trumane, mislim da si se ovde mudro postavio. Otkad domaći HR profesionalci smatraju da je IQ test značajan deo procesa regrutovanja nove radne snage, zaista je pametno malo vežbati te testove u slobodno vreme.
Ja lično smatram da su ti testovi irelevantni za uspeh u poslu, al ako oni tako kažu...ko će im se suprotstaviti...
realno, ako ih uradiš dobro i nisu garancija za uspjeh
ali ako ih uradiš loše garancija su za neuspjeh 8-)
za ovo prvo se slažem, za drugo ne znam. Al da me ne shvatite pogrešno- jednom kad sam radio dobro sam prošao. 8-)
ljudi za koje iskreno smatram da su ispod prosjeka dobijali su na Mensinom testu oko 120 bodova
realno samo budale na kvadrat dobijaju ispod 100
tako da 120 u stvari ne znači gotovo ništa, ali 95 itekako znači
Normalno je to što si primetio. Mensini testovi su baždareni za svetski prosek, ali prosek IQa u Srbiji je malo veći, pa se to na testu tako vidi. Uzgred, to što Srbi imaju malo veći prosek takođe ništa ne znači, jer statistički gledano pola država u svetu ima veći prosek od celog sveta.
Ma, da se razumemo, za HR je ovo samo lak način da uspostave donji kriterijum za kandidate a da ne moraju da sami osmisle ikakav relevantan test. Kako Bata reče
Quoteako ih uradiš dobro i nisu garancija za uspjeh
ali ako ih uradiš loše garancija su za neuspjeh
Quote from: mac on 03-10-2013, 19:15:06
Normalno je to što si primetio. Mensini testovi su baždareni za svetski prosek, ali prosek IQa u Srbiji je malo veći, pa se to na testu tako vidi. Uzgred, to što Srbi imaju malo veći prosek takođe ništa ne znači, jer statistički gledano pola država u svetu ima veći prosek od celog sveta.
Jesi li siguran da smo iznad proseka? Prema istraživanju onog Engleza kod nas je prosečan iq 89. Ne verujem da je baš toliko nizak, ali s druge strane odokativno posmatrano u Srbiji ima puno baš ograničenih ljudi ( zapravo, primitivnih, zatucanih i neobrazovanih - ne znam koliko je to u korelaciji sa iq-om ).
a misliš da u Engleskoj samo piju čaj? 8-)
Ne, samo mi je teško da poverujem da su Srbi iznad proseka. Na jednog genijalca iz matematičke gimnazije imaš barem 1000 dileja.
pa ni Englezi nisu prosjek, i oni su iznad prosjeka
uopšte ne znamo da li su Srbi iznad ili ispod Engleza
a govori se o ljudima koji su radili test, ne o kompletnom stanovništvu
mada realno baš ta Engleska ima i veće seljačine, i navijače-huligane, a i intelektualce od Srbije, izgleda da to tako ide zajedno
Mislim da nema veće seljačine. Ja ne pričam o hiliganima, već o običnom svetu.
Ta fora sa prosečnim srpskim IQ od 89 je bila neka podmetačina. Smunjivo je mnogo toga. Članak je iz 2006. godine ( http://www.amren.com/news/2006/03/germans_are_bra/ (http://www.amren.com/news/2006/03/germans_are_bra/) ), spominju nekakvu European League of IQ scores, a taj "entitet" se posle ovog članka nigde više ne spominje. U članku se spominje neki Ričard Lin (http://en.wikipedia.org/wiki/Richard_Lynn), koji pokušava da uvede rasizam u nauku, a optužuju ga za nenaučne metode.
Taj članak nije merodavan, i ne treba ga uzimati u obzir. Daj neki drugi izvor.
Što se tiče rasizma...više istraživanja je pokazalo da najniži iq imaju afrički crnci. Tako da pitanje je da li smo zainteresovani za istinu ili za političku korektnost.
Ja ti kažem da je Ričard Lin kritikovan da koristi nenaučne metode, i da je ovo gore istraživanje sproveo Ričard Lin, a IQ crnaca mi nije trenutno u centru pažnje.
Koliko se ja sećam pisanja novina iz tog perioda taj Lin je napravio spisak nacija sa svih kontinanata gde su Afrikanci na dnu, a od Evropljana smo mi na začećju pa Hrvati s bodom više. :mrgreen:
Quote from: Truman on 03-10-2013, 22:30:03
Mislim da nema veće seljačine. Ja ne pričam o hiliganima, već o običnom svetu.
pa preporučio bih ti da se upoznaš sa engleskom književnošću u posljednja dva vijeka, čisto da bi se uvjerio kakva su stoka
kakvih boleština je u toj Engleskoj bilo i još uvijek ima, mislim da kad bi neko javio Dikensu kakva je savremena situacija ovaj bi počeo da piše preslatke romane o 19. vijeku
naravno, imamo i mi pisce koji nam objašnjavaju kakva smo mi stoka, al još nismo prevazišli gospodu secikese s ostrva
Koliko se ja sećam Dositej Obradović je pisao kakva su gospoda Englezi i koliko su iznad drugih.
Josif Tatic - Velsani (http://www.youtube.com/watch?v=iif-1AU6X98#)
Bruka i sramota, ja se već na drugom nivou zablokirah ( a ima ih 20! xuzi ). Takođe. ulažem oštar protest što mnogi zadaci imaju latinična slova a ja pošto ne znam njihov redosled ne umem ni da ih rešim.
I evo ga jedan s prvog nivoa koji nisam znao - kako ćete sa 5 šibica da formirate br. 16.
Quote from: Truman on 04-10-2013, 13:44:12I evo ga jedan s prvog nivoa koji nisam znao - kako ćete sa 5 šibica da formirate br. 16.
XVI
Ma to su retardirani zadaci. Mislim, lateralno mišljenje nam kaže da je šibice lako izlomiti na koliko god nam treba delova i to je jednako tačan odgovor kao i bilo koji drugi. Onaj ko tako nešto koristi da premeri nečiju "inteligenciju" je nevaljao.
Quote from: Milosh on 04-10-2013, 13:50:43
Quote from: Truman on 04-10-2013, 13:44:12I evo ga jedan s prvog nivoa koji nisam znao - kako ćete sa 5 šibica da formirate br. 16.
XVI
Tačno, mada to bi značilo ''šesnaesti'' a ne 16 ako ćemo da cepidlačimo. U pravu je i Meho.
Mogu tek da zamislim koliko su teški zadaci na narednim nivoima... xdrinka
Ali da podvučemo da konkretno taj zadatak nema nikakve veze sa testiranjem inteligencije nego sa poznavanjem određenih kulturnih obrazaca. Dakle, dizajnirao ga je neko nestručan ili zloban. :lol:
ili neko ko hoće da ti testira ''out of box'' razmišljanje. :)
Što je legitimno, ali to onda valjda nije test IQ-a?
Na višim nivoima mislim da jeste. Sačekajmo da čujemo Batino mišljenje.
Bata je neki autoritet na tom polju???? :-? :-? :-?
Koliko sa čuo iz nezvaničnih izvora jeste...
jedini test koji autoritet 8-) priznaje izvan Mense je Ajnštajnov test, te šibice nisu ni za cigare 8-)
inače, evo Mensinog testa, ispoštovana je forma, sadržinski je nešto lakši
http://www.mensa.hr/eu_iq_test/eu_iq_test.php (http://www.mensa.hr/eu_iq_test/eu_iq_test.php)
poznat mi je nešto, davno sam ga rešavao pa ne bi bilo korektno da sad nabudžim rezultat i pravim se kako sam mnogo pametan.
U kom brojnom sistemu važi ova jednakost i koje su vrednosti slova (isto slovo - ista vrednost; različita slova - različite vrednosti):
K Y T O
+ K Y O T O
____________
T O K Y O ?
Baš simpatičan.
Ako sam dobro izračunao, brojna osnova je sedam, a jednakost je:
4350 +
43050
--------
50430
Logika je bila da je 0 (slovo O) nula, zato što zbir dvije iste cifre u bilo kom brojnom sistemu ne može dati istu cifru (osim ako nemamo prenos u prethodnom koraku, a i tada samo ako je binarni sistem). Onda se postavi niz jednačina sa prenosom, pri čemu je prenos maksimalno jedan bez obzira na brojni sistem (p0 prenos iz prvog koraka, p1 iz drugog itd.). Lako se utvrdi da je p3 jedinica, zato što K i T nisu isti simboli. Kako je p2 nula (jer Y i K nisu isti simboli, a Y + 0 bi trebalo da je Y, ali kako ispod imamo K, to je prenos p1 = 1), onda nam K+Y daje bazu sistema koja je, očigledno, neparna, jer su K i Y uzastopni brojevi. Sad se to izrovari i dobije se rješenje. Stvarno fin zadatak :)
Tačno.
I meni se svideo zadatak. Tu i tamo prija kada čovek iskorači iz kolotečine :)
'Well, we fooled around with puzzles and palindromes for a while, until he wrote out this series of numbers: 1, 11, 21, 1211, 111221. "Complete that series, Cliff." I looked at it for five minutes and gave up. I'm sure it's easy, but to this day, I still haven't solved it.'
Cuckoo's Egg - Clifford Stoll
Upravo čitam (bolje da ne napišem po koji put), pokušah da riješim i kao uspio, ali jako nategnuto. Onda otišao na Internet i vidio da je totalno pogrešno :). O problemu i rješenju imate ovdje http://goo.gl/1yu4vr (http://goo.gl/1yu4vr) , a a ime linka je toliko samoobjašnjavajuće da sam ga provukao kroz url shortener. Ko provali svaka čast, ja nisam mogao (a djeluje mi i da ne bi).
Znam zadatak i, kada sam ga videla, nisam uspela da ga rešim.
Lakše je sa sledećim članom - tako sam dala deci kad su bili mali i neko do njih je rešio.
Mogao bih se zakleti da sam ovaj niz već video na ovom topiku, ali nema ga. (Malo guglovanja koje sam trebao odma', a ne da ručno češljam ceo topik...) Elem, ima ga na forumu, na drugom topiku
http://www.znaksagite.com/diskusije/index.php?topic=11124.0 (http://www.znaksagite.com/diskusije/index.php?topic=11124.0)
Svaka čast. Ja sam izgleda jedini kome je ovaj niz bio nepoznat. Fibonačija sam i ja prepoznao, ali nisam znao šta sa njim. Onda sam se zamajavao nekakvim rotacijama (da li su parne ili neparne), ali uvijek je ostajao taj meni neobjašnjivi korak kada 11 postaje 21. Eto, zadatak sa topikom, čisto da ne bude da toga nema :)
Odradio sam treći nivo, ovde već pola nisam uspeo da rešim...:D
Kakav nivo? Gledam prethodne poruke i ne vidim na šta se to odnosi.
Pisah o tome - kupio sam knjigu ''Povećajte svoj iq'', ima 20 nivoa :D
Juče imao testiranje u Delta Đeneraliju, danas mi zakazali razgovor. :) Izgleda da se isplatilo rađenje onih zadačića, baš sam odvalio testove sposobnosti.
Mala pomoć ako može ko...Pre jedno 6-7-8 godina sam na netu naleteo na jedan test inteligencije koji je bio u simbolici vanzemaljaca. Znači slika aliena u uglu, a na testu su bili neki neobični logički problemi i na taj tip kasnije nisam nailazio. I skala je bila drugačija i objašnjenje u nekom posebnom fazonu. Uglavnom, ovaj test pamtim po tome što mi je bio jako težak i atipičan i nisam se baš na njemu proslavio. Ako neko zna o čemu je reč i ako ima link bio bih mu zahvalan...
Na osnovu tvojih sturih podataka, guglovao sam malo, naravno nisam nasao to sto trazis,
ali sam naisao na jedan simpatican IQ test (fles igrica), pa ako neko hoce nek baci pogled:
http://www.flashgames247.com/play/16205.html (http://www.flashgames247.com/play/16205.html)
Jedan od ponudjenih jezika je srpski.
U međuvremenu naleteh na jedan drugi koji mi se čini da nije loš: http://www.iqtest.dk/main.swf (http://www.iqtest.dk/main.swf)
Ako pravougli trougao čija je hipotenuza 5 i duža kateta 4 zarotiramo oko duže katete, dobićemo geometrijsko telo koje ima osnovicu površine:
a) 3
b) 9
c) 9π
d) 3π
e) 20
Živo me kopka, je l odgovor 9Pi?
9Pi je tacan odgovor. sa hipotenuzom 5 i jednom katetom 4, druga kateta je 3. Posto rotiramo trougao oko duze katete, kraca kateta postaje poluprecnik opisane kruznice cija se povrsina trazi. P=Pi*r^2 dakle 9Pi
Iskočila mi u glavi formula ''r na kvadrat pi'' pa rek'o da proverim da l je to za površinu kruga. Mislim da sam zaboravio i najosnovnije formule iz matematike...
Ovo nije baš matematički test, ali treba ti neka sposobnost, koju ili imaš ili nemaš: http://en.wikipedia.org/wiki/Bongard_problem (http://en.wikipedia.org/wiki/Bongard_problem)
Dobiješ dva skupa crteža, i treba da utvrdiš i opišeš šta je to što je zajedničko crtežima iz prvog skupa, a što nema ni jedan crtež iz drugog skupa. Ponekad je to rečima i teško opisati :)
Na kraju Vikipedijinog članka je link ka indeksu takvih problema, ali nema rešenja, pa za teže probleme možemo samo da nagađamo rešenje, http://www.foundalis.com/res/bps/bpidx.htm (http://www.foundalis.com/res/bps/bpidx.htm)
Quote from: mac on 22-07-2014, 01:54:27
Ovo nije baš matematički test, ali treba ti neka sposobnost, koju ili imaš ili nemaš
Pre ovo drugo :(
Teško zapažam očigledne stvari. Na primer, u nekom testu inteligencije na pitanje šta je zajedničko za 25 i 144 rekla sam da su oba broja kvadrati (tačan odgovor je da su oba brojevi). Ali zanimljivo je ovo. I zabavno.
Evo male igre. Već par dana se mučim, ali i napredujem polako. Može da se igra i oflajn na brauzeru mobilnog telefona.
http://gameaboutsquares.com/ (http://gameaboutsquares.com/)
Kul je, ja već zablokirao na 4. nivou. :D
Ljudi, kako da rešim 4. nivo???
Moraš nekako da poguraš plavi kvadrat dva puta na dole, ali nigde ne piše da za ta dva mesta moraš da ga guraš istim kvadratom...
Hvala! Širiš mi vidike :)
Do desetog nivoa prođoh lako, sad sledi novi izazov. Inače, ove poruke koje igrač dobija između nivoa su mi kul.
stigoh do 13. nivoa. Do kog ste vi?
Neću ti kažem, sneveselićeš se. Igraj svoju igru :)
Mene samo zanima koliko ima nivoa...
Interesantno.
I zarazno, umesto da radim druge stvari osećam potrebu stalno da buljim u kvadratiće.
Igra u jednom trenutku počne da te zeza u vezi s tim što ne znaš koliko još ima do kraja, pa je i nepoznanica o broju novoa u neku ruku deo igre :)
35
Po broju tačkica, 6x6, trebalo bi da ima 36 nivoa. To se vidi kad klikneš na broj koji označava tvoj trenutni nivo.
21, nemam živaca više...
pa nastavi sutra, pamti se dokle si stigao :)
dajte rješenje za level 23, poluđeh od Borisa
Za ovo prvo ti ne mogu pomoci, a za ovo drugo, imas resenje u opciji ignore, provereno radi posao.
не знам како бих ти презентовао решење :(
provalio sam, i još je mnogo jednostavno a ja sve komplikovao...
Quote from: Boris on 31-07-2014, 14:25:10
Za ovo prvo ti ne mogu pomoci, a za ovo drugo, imas resenje u opciji ignore, provereno radi posao.
uzo sam snikers!
Za mene je 15 misterija...
p.s. ovo više ne važi, ajmo dalje...Rešenje je tako prosto da me je sramota. :mrgreen:
Muči me ovaj 23. nivo...Je l neko prešao sve?
vjerovatno, ja stigoh do 34, al nisam igrao zadnja 24h...
eo sad 35, zadnja
edit
ispade 35 najlakše
Kad završite prethodnu igru pogledajte ovo: http://www.puzzle-nonograms.com/ (http://www.puzzle-nonograms.com/)
Počnite sa 5x5, pa kad vidite da mehanički završavate tablu onda pređite na veću...
Stranica ima linkove i ka drugim glavolomkama, ali tamo za sad nisam zalazio.
Не пада ми на памет. Потрошио сам два дана на ове квадрате, легао јутрос у 6, сад сам на 35-ом нивоу и кад ово завршим чим видим да неко поставља линкове за сличне главоломке, тужићу га суду.
Ma ovo ti je kao sudoku, nema nivoa. Rešiš tablu i završio si. Osim ako se ne navučeš na sledeću i sledeću tablu, ali to već nije moj problem...
po netu su neki ljudi prijetili drugima smrću, jer su ih navukli na kvadrate :)
како можеш да се навучеш на нешто што одрадиш за пола сата-сат?
ko je to završio za sat vremena svaka mu čast
ja sam radio po 2-3 nivoa i onda odustajao pa kasnije ponovo malo, i realno nijednom nisam ni pola sata sastavio već sve na prekide riješavao u periodu od par dana
Pitaš (u šali) za game about squares, ili (ozbiljno) za nonograms? Ako je ovo drugo, to ti je kao što ljudi rešavaju ukrštene reči. Neki ne stanu dok sve ukrštenice u novinama ne popune. Mogu da stanu posle prve, i ostave ostalo za drugi dan, ali neće :)
питао сам за квадрате, нонограмс су друга прича.
Upozorenje! Među ovim nonogramima ima i nerešivih. Nikako da uklopim jedan, pa reko da ga provučem kroz neki onlajn solver, i zaista ni mašina ne može da ga reši.
Probajte da rešite: http://www.telegraf.rs/zanimljivosti/1374183-dokaz-da-ste-natprosecno-inteligentni-ovu-zagonetku-ne-moze-da-resi-ni-10-odsto-ljudi-foto (http://www.telegraf.rs/zanimljivosti/1374183-dokaz-da-ste-natprosecno-inteligentni-ovu-zagonetku-ne-moze-da-resi-ni-10-odsto-ljudi-foto)
Ja sam pao na ovom testu inteligencije. :) Zahteva laterarno razmišljanje.
Simpatična, mada je trebalo naglasiti da su sve osobe dovoljno inteligentne, jer ako nije tako, osoba ... neće zaključiti koji šešir je na njenoj glavi (a i malo upućuje kako se rešava ovaj tip zadataka).
Ono što se meni čini zanimljivim ( radi analize sopstvenog procesa razmišljanja ) je kako sam odmah krenu da računam verovatnoće, pa sam izračunao da jedan od njih ima 66% šanse da pogodi. Taj put kojim je moj mozak krenuo je verovatno posledica obrazovnog usmerenja dok s druge strane ''out of box'' razmišljanje nisam imao prilike mnogo da razvijam.
Mislim da je navlakusa kad kazu "blablabla ni 10% ne moze da resi". Uz malo mozganja mislim da svako moze da je resi, jos ako ste upuceni u sistem razmisljanja koji ide uz ovakve zagonetke, mislim da vam ne bi ni trebala natprosecna inteligencija. Sad, mozda se ja tako samo reklamiram jer mi je trebalo manje od minuta da resim pricicu, ali je poenta da se kod ovakvih zadataka polazi obrnutim redosledom pri resavanju. Umesto kombinatorike uvek se polazi od idealne situacije koja bi davala najlakse resenje. Posto situacija nije takva, osobe iz price razmisljaju zasto nije tako, i mic po mic dodje se do resenja. Ove je recimo polazna tacka to zasto se osoba D nije odmah oglasila, jer bi to bilo sasvim logicno u slucaju da osobe B i C (koje su vidljive osobi D) imaju sesire iste boje. I onda se zadatak maltene sam resava od te polazne tacke...
Quote from: Berserker on 03-01-2015, 16:33:36
Mislim da je navlakusa kad kazu "blablabla ni 10% ne moze da resi".
Slažem se. Za ovakve pitalice je presudnije iskustvo u rešavanju sličnih, od pameti. A da je sam zadatak veoma loše sročen - i to stoji, tako da nije ni čudo što se samo 10% izbori sa ovakvim tekstom.
Ajnštajnov test je pojam za ovo
a sročen je stvarno očajno, npr ovo što reče Berserker
''zasto se osoba D nije odmah oglasila''
to se uopšte ne podrazumijeva
nigdje ne piše da postoji neko vremensko ograničenje, pa kao ''sačekajte desetak sekundi'' i znaćete odgovor
nači u nultoj sekundi niko ne zna, a sve ostalo je varanje na testu, prepisivanje :)
Ko drži ribice, e to je prava stvar, to nađi, Truki
Quote from: Pizzobatto on 04-01-2015, 00:28:54
Ajnštajnov test je pojam za ovo
a sročen je stvarno očajno, npr ovo što reče Berserker
''zasto se osoba D nije odmah oglasila''
to se uopšte ne podrazumijeva
Ko drži ribice, e to je prava stvar, to nađi, Truki
Bato, to se ne podrazumeva ali je to polazna osnova od koje se resavaju ovakvi zadaci. Neko dodje do toga razmisljanjem, a neko zna za jadac pa odmah pocne tako da razmislja i resi ga ocas posla. Recimo za Ajnstajnov test (evo ga ovde http://www.iqtest-center.com/einstein-test.php (http://www.iqtest-center.com/einstein-test.php)), izgleda kao da je uzasno tezak za resavanje, a ustvari postoji citava klasa problema koji su isto tako formulisani (ne mogu da se setim kako se zovu na srpskom, jbg, odavno sam to resavao u nekoj novini koja ih je redovno objavljivala, ali se na engleskom zovu Grid puzzles, evo primera http://www.thepuzzleclub.com/logicpuzzlessolve.php (http://www.thepuzzleclub.com/logicpuzzlessolve.php) ). I kad tako postavis i Ajnstajnov problem postaje bar 3x laksi za resavanje. Dakle svaki problem moze da se resi sirovom inteligencijom, kad mozda i vazi da samo 2% ljudi mogu da je uposle na taj nacin. Takodje masa problema vec ima oproban metod za resavanje koji, kada se primeni, cini taj problem resivim mnogo za mnogo siru populaciju, tj. za normalce :)
PS
sad mi pade na pamet da mozda mislis na to da je tekst zadatka ocajno srocen, sa cim se potpuno slazem. Posteno, prepoznao sam ga jer sam vec resavao slicne zadatke pa sam znao sta da ocekujem, al nista bolje nisam ni ocekivao od nasih prepisivaca "novinara". Btw evo jos jednog problemcica koji trazi out of the box razmisljanje, bas mi juce pustio drugar :)
A dragon and knight live on an island. This island has seven poisoned wells, numbered 1 to 7. If you drink from a well, you can only save yourself by drinking from a higher numbered well. Well 7 is located at the top of a high mountain, so only the dragon can reach it.
One day they decide that the island isn't big enough for the two of them, and they have a duel. Each of them brings a glass of water to the duel, they exchange glasses, and drink. After the duel, the knight lives and the dragon dies.
Why did the knight live? Why did the dragon die?
ma jasno, ali to sa prikrivanjem informacija nije logički problem
Ajnštajnov jeste
ovdje moraš mnogo pretpostavki da prihvatiš, ne možeš precizno da odgovoriš
no, jeste bolje sročen
Pizzobatto, ne usuđejem se raditi Ajnštajnov...
ma probaj, on se svakako može riješiti, mada računaju da kad prešišaš 20 min u rješavanju onda si izgubio
Quote from: Berserker on 04-01-2015, 12:37:16
Btw evo jos jednog problemcica koji trazi out of the box razmisljanje, bas mi juce pustio drugar :)
A dragon and knight live on an island. This island has seven poisoned wells, numbered 1 to 7. If you drink from a well, you can only save yourself by drinking from a higher numbered well. Well 7 is located at the top of a high mountain, so only the dragon can reach it.
One day they decide that the island isn't big enough for the two of them, and they have a duel. Each of them brings a glass of water to the duel, they exchange glasses, and drink. After the duel, the knight lives and the dragon dies.
Why did the knight live? Why did the dragon die?
Da li se računa u varanje ako vitez pre dvoboja otpije iz svoje čaše?
rešenje ovog vitez-zmaj zadatka podrazumeva da postoji i nezatrovana voda, pa vitez da zmaju nezatrovanu, a on sam popije pre duela vodu sa izvora 1.
Kratak online test Are You Smart Enough To Work For Google? (http://www.clickhole.com/quiz/are-you-smart-enough-work-google-1656)
Juče i danas se održava SMO (Srpska matematička olimpijada) i upravo sam se domogao zadataka. :) Treći zadatak me podsjetio na jedan problem koji je
Midoto postavila ovdje nekad davno:
Quote from: Midoto on 29-01-2013, 10:02:25
Evo vam jedan "orah". Opet stražar i opet zatvorenici (i opet sam ga preradila u manje brojeve, jer mislim da je odličan za dodatnu nastavu :) )
Stražar ujutru okupi deset zatvorenika i na leđa im zalepi jedan od brojeva iz skupa od 1 do 10. Neki zatvorenici mogu imati i isti broj na leđima, jer stražar ima po 10 komada od svakog broja, pa nasumično stavlja nalepnicu, a mogu imati i svi različite. Tek - svaki zatvorenik ima na leđima jedu nalepnicu i svaki može da vidi ostalih devet, pa da (bez ikakvog domunđavanja, znakova ili sličnih prevarica) zapiše broj od 1 do 10, za koji misli da je na njegovim leđima. Ako bar jedan zatvorenik pogodi broj - svi bivaju nagrađeni. Ako svi promaše - svi će biti kažnjeni. Elem, zatvorenici naprave strategiju (matematičku) zahvaljujući kojoj svakodnevno dobijaju nagrade. Kakva im je strategija?
A evo tog zadatka sa SMO:
Stražar predlaže zatvorenicima sledeću igru. Svi će biti izvedeni u dvorište gde će svakom od njih biti stavljen na glavu šešir u jednoj od 5 mogućih boja. Svaki zatvorenik videće sve šešire sem sopstvenog. Stražar će ih potom poređati u vrstu i pitati prvog zatvorenika u vrsti da li zna boju svog šešira. Zatvorenik glasno odgovara "da" ili "ne". Ako odgovori "ne", biće odmah zaključan u samicu. Ako odgovori "da", stražar će ga pitati koje je boje njegov šešir, na šta zatvorenik treba da odgovara na takav način da ostali zatvorenici ne čuju odgovor. Ukoliko je odgovor pogrešan, taj zatvorenik biće odmah zaključan u samicu pred svima, a ako je odgovor tačan, taj zatvorenik biće odmah oslobođen pred svima. Stražar potom prilazi sledećem zatvoreniku u redu i ponavlja isti postupak, i tako sve do poslednjeg zatvorenika. Zatvorenici imaju mogućnost da osmisle strategiju pre početka igre, ali kad igra počne, nikakva komunikacija među zatvorenicima nije dozvoljena. Ako u zatvoru ima 2015 zatvorenika, koji je maksimalan broj zatvorenika koji će zagarantovano biti oslobođeni ukoliko zatvorenici primenjuju optimalnu strategiju?
Mislim da je 1995...
Ako znaju kojih će 5 boja biti u igri, može da ih bude i 2000.
Htedoh da napišem 2010 (5 manje), a i to bi bila greška, jer ih je najmanje 2011, a sada pokušavam da dođem i do boljeg rešenja.
Ne mogu da smislim taktiku u kojoj bi odgovor "ne" bio korisniji od odgovora "da", posle kog bi zatvorenik nešto lupio.
Valjda se računa da će svi zatvorenici raditi u interesu većine (jednom će i oni biti u sredini kolone :) ). Za sada sam našla strategiju u kojoj trojica mogu, ali ne moraju da odu u samicu, a ostali će biti osobođeni. Ne mogu da dokažem da je nemoguća taktika u kojoj stradaju samo dva zatvorenika. Baš me zanima rešenje. Otišla sam i na Imomath, pa po rezultatima vidim da niko nije uradio taj zadatak, te da je maksimalan broj poena na njemu - 2. Većina nije uspela ni da ga načne.
A za Mouchette ima lep geometrijski - onaj prvi.
I ja za ovaj zadatak imam taktiku po kojoj stradaju najviše četvorica (a možda i niko). Pretpostavljam da je i vaša ista: ako boje označimo od 1 do 5 onda zbir svih boja na šeširima može po modulu 5 biti 0, 1, 2, 3 ili 4. Prvi onda odgovara sa 'da' i dopunjuje svoju boju do 0, drugi kaže 'da' i dopunjuje do 1 itd. Prvi od njih četvorice koji bude spašen daće informaciju ostalima. Ako prva četvorica završe u samici (najgori slučaj) onda peti i ostali znaju da se radilo o 4. Ali, ako prvi bude oslobođen i svi ostali će biti oslobođeni, pa niko ne strada... ako negde ne grešim.
I da, koji je taj geometrijski? :)
Ja sam radila preko zbirova parnosti (šešire sam numerisala od 1 do 5). Oni znaju da je ukupan broj neparan; prvi zatvorenik odgovara da li se razlikuje zbir parnosti 1. i 2. grupe, od zbira 3. i 4. grupe. Posle se pojednostavljuje, ali ne uspevam da "žrtvujem" manje od 3 zatvorenika.
Stiže geometrijski :)
Dat je tetivni četvorougao ABCD. Tačke M, N, P Q su središta stranica DA, AB, BC, CD redom. Tačka E je presek dijagonala AC i BD. Kružnice opisane oko trouglova EMN i EPQ seku se u različitim tačkama E i F. Dokazati da je EF normalno na AC.
Evo lepog zadatka za fiskulturu mozga (u slobodnom prevodu i adaptaciji):
Pera i Mika su u zatvoru. Čuvar im kaže da će im zadati igru u kojoj svaki od njih baci novčić, zapiše da li je dobio pismo ili glavu, a onda zapiše i šta misli da je onaj drugi dobio. Imaju vremena da se dogovore oko eventualne strategije, a onda ih smeštaju u izdvojene gluve sobe - nema dogovaranja, nema varanja... Ako posle prvog bacanja novčića neki od njih dvojice pogodi ispravno, baca se drugi put, pod istim uslovima; onda treći put... Ako deset puta budu uspešni - slobodni su. Ako bilo kada obojica pogreše, obojici se zatvorska kazna poduplava. Mogu da prihvate ove uslove (ako imaju sigurnu strategiju) ili da odbace (ako strategija nije potpuno sigurna). Šta odlučuju Pera i Mika i zašto?
Rešio :)
Pokušao, neslavno propao.
Može li neko da napiše rješenje? Hvala.
Samo to sam čekao! Evo hint (hehe): umesto da gledamo ishode "pismo" i "glava" (sa četiri moguća ishoda) bolje je ako gledamo ishode "isto" i "različito", jer tada vidimo da postoje samo dva moguća ishoda. Sve što treba naša dva junaka da urade je da pokriju ova dva ishoda, i tada će uvek jedan biti u pravu.
a kako da to pokriju?
Pa jedan treba da gađa "isto", a drugi "različito"...
skonto!
Prosto me sramota kako sam ja to zakomplikovao.
Gledao sam 16 mogućih ishoda, pa su problematični parovi PP-GG, PG-PG, GP-GP i GG-PP. Svi ostali (12) daju ishod koji nam treba. I onda sam uzeo da gledam vjerovatnoću ishoda ako uvijek kažu isto i u kakvoj je to vezi sa produženjem kazne (to me nije odvelo nigdje, zato što je nemoguće predvidjeti niz ishoda bacanja novčića).
Međutim ako prvi stalno ponovi ono što je dobio bez obzira da li je bilo pismo ili glava (PP, GG), a drugi uvijek kaže suprotno (PG, GP) dobijaju se četiri kombinacije (PP-PG, PP-GP, GG-PG, GG-GP) koje sve daju bar jedan pogodak.
Svaka čast. Da sam se ja pitao, Pera i Mika bi zaglavili doživotnu :)
Sviđa mi se ovaj tip zadataka gde je bitno da je bar jedan u pravu - obično ispadne da je optimalno da tačno jedan pogađa, a onaj drugi greši.
@BladeRunner - I meni je u prvi mah bila misao da nema nikakve strategije. Nekako mi je bilo krajnje neintuitivno da jedan obavezno pogreši :) Ove nove generacije (vrlo pragmatične) rešavaju zadatak za nekoliko sekundi. Depresivno.
ukucati u gugl zbog radi rešenja
exp(-((x-4)^2+(y-4)^2)^2/1000) + exp(-((x+4)^2+(y+4)^2)^2/1000) + 0.1exp(-((x+4)^2+(y+4)^2)^2)+0.1exp(-((x -4)^2+(y-4)^2)^2)
GIMPS (http://www.mersenne.org/) Project Discovers
Largest Known Prime Number: 2
74,207,281-1
http://www.mersenne.org/primes/?press=M74207281 (http://www.mersenne.org/primes/?press=M74207281)
Quote
RALEIGH, North Carolina -- On January 7th at 22:30 UTC, the Great Internet Mersenne Prime Search (GIMPS) celebrated its 20th anniversary with the math discovery of the new largest known prime number, 274,207,281-1, having 22,338,618 digits (http://www.mersenne.org/primes/digits/M74207281.zip), on a university computer volunteered by Curtis Cooper for the project. The same GIMPS software just uncovered a flaw in Intel's latest Skylake CPUs (http://arstechnica.com/gadgets/2016/01/intel-skylake-bug-causes-pcs-to-freeze-during-complex-workloads/)[1], and its global network of CPUs peaking at 450 trillion calculations per second remains the longest continuously-running "grassroots supercomputing"[2] project in Internet history.
The new prime number, also known as M74207281, is calculated by multiplying together 74,207,281 twos then subtracting one. It is almost 5 million digits larger than the previous record prime number (http://www.mersenne.org/primes/?press=M57885161), in a special class of extremely rare prime numbers known as Mersenne primes. It is only the 49th known Mersenne prime ever discovered, each increasingly difficult to find. Mersenne primes were named for the French monk Marin Mersenne (http://www-groups.dcs.st-and.ac.uk/%7Ehistory/Mathematicians/Mersenne.html), who studied these numbers more than 350 years ago. GIMPS, founded in 1996, has discovered all 15 of the largest known Mersenne primes. Volunteers download a free program (http://www.mersenne.org/freesoft.htm) to search for these primes with a cash award offered to anyone lucky enough to compute a new prime. Prof. Chris Caldwell maintains an authoritative web site on the largest known primes (http://www.utm.edu/research/primes/largest.html) and is an excellent history of Mersenne primes (http://primes.utm.edu/mersenne/index.html).
The primality proof took 31 days of non-stop computing on a PC with an Intel I7-4790 CPU. To prove there were no errors in the prime discovery process, the new prime was independently verified using both different software and hardware. Andreas Hoglund and David Stanfill each verified the prime using the CUDALucas (http://www.mersenneforum.org/showthread.php?t=16142) software running on NVidia Titan Black GPUs in 2.3 days. David Stanfill verified it using ClLucas (http://www.mersenneforum.org/showthread.php?t=18297) on an AMD Fury X GPU in 3.5 days. Serge Batalov also verified it using Ernst Mayer's MLucas (http://hogranch.com/mayer/README.html) software on two Intel Xeon 18-core Amazon EC2 servers in 3.5 days.
Dr. Cooper (http://www.math-cs.ucmo.edu/%7Ecurtisc/primes.html) is a professor at the University of Central Missouri (http://ucmo.edu). This is the fourth record GIMPS project prime for Dr. Cooper and his university. The discovery is eligible for a $3,000 GIMPS research discovery award. Their first record prime was discovered in 2005, eclipsed by their second record in 2006. Dr. Cooper lost the record in 2008, reclaimed it in 2013, and improves that record with this new prime. Dr. Cooper and the University of Central Missouri is the largest contributor of CPU time to the GIMPS project.
Dr. Cooper's computer reported the prime in GIMPS on September 17, 2015 but it remained unnoticed until routine maintenance data-mined it. The official discovery date is the day a human took note of the result. This is in keeping with tradition as M4253 is considered never to have been the largest known prime (http://primes.utm.edu/notes/by_year.html) number because Hurwitz in 1961 read his computer printout backwards and saw M4423 was prime seconds before seeing that M4253 was also prime.
GIMPS Prime95 client software was developed by founder George Woltman. Scott Kurowski wrote the PrimeNet system software that coordinates GIMPS' computers. Aaron Blosser is now the system administrator, upgrading and maintaining PrimeNet as needed. Volunteers have a chance to earn research discovery awards of $3,000 or $50,000 (http://www.mersenne.org/legal/#awards) if their computer discovers a new Mersenne prime. GIMPS' next major goal is to win the $150,000 award administered by the Electronic Frontier Foundation (http://www.eff.org/awards/coop-prime-info.php) offered for finding a 100 million digit prime number.
Credit for GIMPS' prime discoveries goes not only to Dr. Cooper for running the Prime95 software on his university's computers, Woltman, Kurowski, and Blosser for authoring the software and running the project, but also the thousands of GIMPS volunteers that sifted through millions of non-prime candidates. Therefore, official credit for this discovery shall go to "C. Cooper, G. Woltman, S. Kurowski, A. Blosser, et al."
About Mersenne.org's Great Internet Mersenne Prime Search
The Great Internet Mersenne Prime Search (GIMPS) (http://www.mersenne.org/) was formed in January 1996 by George Woltman to discover new world record size Mersenne primes. In 1997 Scott Kurowski enabled GIMPS to automatically harness the power of hundreds of thousands of ordinary computers to search for these "needles in a haystack". Most GIMPS members join the search for the thrill of possibly discovering a record-setting, rare, and historic new Mersenne prime. The search for more Mersenne primes is already under way. There may be smaller, as yet undiscovered Mersenne primes, and there almost certainly are larger Mersenne primes waiting to be found. Anyone with a reasonably powerful PC can join GIMPS and become a big prime hunter, and possibly earn a cash research discovery award. All the necessary software can be downloaded for free at www.mersenne.org/freesoft.htm (http://www.mersenne.org/freesoft.htm). GIMPS is organized as Mersenne Research, Inc., a 501(c)(3) science research charity. Additional information may be found at www.mersenneforum.org (http://www.mersenneforum.org/) and www.mersenne.org (http://www.mersenne.org); donations are welcome.
For More Information on Mersenne Primes
Prime numbers have long fascinated amateur and professional mathematicians. An integer greater than one is called a prime number if its only divisors are one and itself. The first prime numbers are 2, 3, 5, 7, 11, etc. For example, the number 10 is not prime because it is divisible by 2 and 5. A Mersenne prime is a prime number of the form 2P-1. The first Mersenne primes are 3, 7, 31, and 127 corresponding to P = 2, 3, 5, and 7 respectively. There are only 49 known Mersenne primes.
Mersenne primes (http://www.utm.edu/research/primes/mersenne.shtml) have been central to number theory since they were first discussed by Euclid about 350 BC. The man whose name they now bear, the French monk Marin Mersenne (http://www-groups.dcs.st-and.ac.uk/%7Ehistory/Mathematicians/Mersenne.html) (1588-1648), made a famous conjecture on which values of P would yield a prime. It took 300 years and several important discoveries in mathematics to settle his conjecture.
Previous GIMPS Mersenne prime discoveries were made by members in various countries.
In January 2013, Curtis Cooper et al. discovered the 48th known Mersenne prime (http://www.mersenne.org/57885161.htm) in the U.S.
In April 2009, Odd Magnar Strindmo et al. discovered the 47th known Mersenne prime (http://www.mersenne.org/various/42643801.htm) in Norway.
In September 2008, Hans-Michael Elvenich et al. discovered the 46th known Mersenne prime (http://www.mersenne.org/m45and46.htm) in Germany.
In August 2008, Edson Smith et al. discovered the 45th known Mersenne prime (http://www.mersenne.org/m45and46.htm) in the U.S.
In September 2006, Curtis Cooper and Steven Boone et al. discovered the 44th known Mersenne prime (http://www.mersenne.org/various/32582657.htm) in the U.S.
In December 2005, Curtis Cooper and Steven Boone et al. discovered the 43rd known Mersenne prime (http://www.mersenne.org/various/30402457.htm) in the U.S.
In February 2005, Dr. Martin Nowak et al. discovered the 42nd known Mersenne prime (http://www.mersenne.org/various/25964951.htm) in Germany.
In May 2004, Josh Findley et al. discovered the 41st known Mersenne prime (http://www.mersenne.org/various/24036583.htm) in the U.S.
In November 2003, Michael Shafer et al. discovered the 40th known Mersenne prime (http://www.mersenne.org/various/20996011.htm) in the U.S.
In November 2001, Michael Cameron et al. discovered the 39th Mersenne prime (http://www.mersenne.org/various/13466917.htm) in Canada.
In June 1999, Nayan Hajratwala et al. discovered the 38th Mersenne prime (http://www.mersenne.org/various/6972593.htm) in the U.S.
In January 1998, Roland Clarkson et al. discovered the 37th Mersenne prime (http://www.mersenne.org/various/3021377.htm) in the U.S.
In August 1997, Gordon Spence et al. discovered the 36th Mersenne prime (http://www.mersenne.org/various/2976221.htm) in the U.K.
In November 1996, Joel Armengaud et al. discovered the 35th Mersenne prime (http://www.mersenne.org/various/1398269.htm) in France.
Euclid proved that every Mersenne prime generates a perfect number. A perfect number is one whose proper divisors add up to the number itself. The smallest perfect number is 6 = 1 + 2 + 3 and the second perfect number is 28 = 1 + 2 + 4 + 7 + 14. Euler (1707-1783) proved that all even perfect numbers come from Mersenne primes. The newly discovered perfect number is 274,207,280 x (274,207,281-1). This number is over 44 million digits (http://www.mersenne.org/primes/perfect/perfect74207281.zip) long! It is still unknown if any odd perfect numbers exist.
There is a unique history to the arithmetic algorithms underlying the GIMPS project. The programs that found the recent big Mersenne finds are based on a special algorithm. In the early 1990's, the late Richard Crandall (http://en.wikipedia.org/wiki/Richard_Crandall), Apple Distinguished Scientist, discovered ways to double the speed of what are called convolutions -- essentially big multiplication operations. The method is applicable not only to prime searching but other aspects of computation. During that work he also patented the Fast Elliptic Encryption system, now owned by Apple Computer, which uses Mersenne primes to quickly encrypt and decrypt messages. George Woltman implemented Crandall's algorithm in assembly language, thereby producing a prime-search program of unprecedented efficiency, and that work led to the successful GIMPS project.
School teachers from elementary through high-school grades have used GIMPS to get their students excited about mathematics. Students who run the free software are contributing to mathematical research. David Stanfill's verification computation for this discovery was donated by Squirrels (airsquirrels.com (http://airsquirrels.com)) which services K-12 education and other customers.
[1] [url=http://hardwareluxx.de]http://hardwareluxx.de (http://hardwareluxx.de)[/url] ; [url=http://arstechnica.com/gadgets/2016/01/intel-skylake-bug-causes-pcs-to-freeze-during-complex-workloads]http://arstechnica.com/gadgets/2016/01/intel-skylake-bug-causes-pcs-to-freeze-during-complex-workloads (http://arstechnica.com/gadgets/2016/01/intel-skylake-bug-causes-pcs-to-freeze-during-complex-workloads)[/url]
[2] Science (American Association for the Advancement of Science), May 6, 2005 p810.
Quote from: Meho Krljic on 22-01-2016, 10:54:28
GIMPS (http://www.mersenne.org/) Project Discovers
Largest Known Prime Number: 274,207,281-1
Evo i nekoliko zanimljivih videa o tome:
http://youtu.be/tlpYjrbujG0 (http://youtu.be/tlpYjrbujG0)
http://youtu.be/lEvXcTYqtKU (http://youtu.be/lEvXcTYqtKU)
http://youtu.be/jNXAMBvYe-Y (http://youtu.be/jNXAMBvYe-Y)
Kad smo već kod prostih brojeva:
Mathematicians Discover Prime Conspiracy (https://www.quantamagazine.org/20160313-mathematicians-discover-prime-conspiracy/)
QuoteA previously unnoticed property of prime numbers seems to violate a longstanding assumption about how they behave.
Two mathematicians have uncovered a simple, previously unnoticed property of prime numbers — those numbers that are divisible only by 1 and themselves. Prime numbers, it seems, have decided preferences about the final digits of the primes that immediately follow them.
Among the first billion prime numbers, for instance, a prime ending in 9 is almost 65 percent more likely to be followed by a prime ending in 1 than another prime ending in 9. In a paper posted online today (http://arxiv.org/abs/1603.03720), Kannan Soundararajan and Robert Lemke Oliver of Stanford University present both numerical and theoretical evidence that prime numbers repel other would-be primes that end in the same digit, and have varied predilections for being followed by primes ending in the other possible final digits.
"We've been studying primes for a long time, and no one spotted this before," said Andrew Granville, a number theorist at the University of Montreal and University College London. "It's crazy."
The discovery is the exact opposite of what most mathematicians would have predicted, said Ken Ono, a number theorist at Emory University in Atlanta. When he first heard the news, he said, "I was floored. I thought, 'For sure, your program's not working.'"
This conspiracy among prime numbers seems, at first glance, to violate a longstanding assumption in number theory: that prime numbers behave much like random numbers. Most mathematicians would have assumed, Granville and Ono agreed, that a prime should have an equal chance of being followed by a prime ending in 1, 3, 7 or 9 (the four possible endings for all prime numbers except 2 and 5).
"I can't believe anyone in the world would have guessed this," Granville said. Even after having seen Lemke Oliver and Soundararajan's analysis of their phenomenon, he said, "it still seems like a strange thing."
Yet the pair's work doesn't upend the notion that primes behave randomly so much as point to how subtle their particular mix of randomness and order is. "Can we redefine what 'random' means in this context so that once again, [this phenomenon] looks like it might be random?" Soundararajan said. "That's what we think we've done."
Prime Preferences
Soundararajan was drawn to study consecutive primes after hearing a lecture at Stanford by the mathematician Tadashi Tokieda, of the University of Cambridge, in which he mentioned a counterintuitive property of coin-tossing: If Alice tosses a coin until she sees a head followed by a tail, and Bob tosses a coin until he sees two heads in a row, then on average, Alice will require four tosses while Bob will require six tosses (try this at home!), even though head-tail and head-head have an equal chance of appearing after two coin tosses.
Soundararajan wondered if similarly strange phenomena appear in other contexts. Since he has studied the primes for decades, he turned to them — and found something even stranger than he had bargained for. Looking at prime numbers written in base 3 — in which roughly half the primes end in 1 and half end in 2 — he found that among primes smaller than 1,000, a prime ending in 1 is more than twice as likely to be followed by a prime ending in 2 than by another prime ending in 1. Likewise, a prime ending in 2 prefers to be followed a prime ending in 1.
Soundararajan showed his findings to postdoctoral researcher Lemke Oliver, who was shocked. He immediately wrote a program that searched much farther out along the number line — through the first 400 billion primes. Lemke Oliver again found that primes seem to avoid being followed by another prime with the same final digit. The primes "really hate to repeat themselves," Lemke Oliver said.
Lemke Oliver and Soundararajan discovered that this sort of bias in the final digits of consecutive primes holds not just in base 3, but also in base 10 and several other bases; they conjecture that it's true in every base. The biases that they found appear to even out, little by little, as you go farther along the number line — but they do so at a snail's pace. "It's the rate at which they even out which is surprising to me," said James Maynard, a number theorist at the University of Oxford. When Soundararajan first told Maynard what the pair had discovered, "I only half believed him," Maynard said. "As soon as I went back to my office, I ran a numerical experiment to check this myself."
Lemke Oliver and Soundararajan's first guess for why this bias occurs was a simple one: Maybe a prime ending in 3, say, is more likely to be followed by a prime ending in 7, 9 or 1 merely because it encounters numbers with those endings before it reaches another number ending in 3. For example, 43 is followed by 47, 49 and 51 before it hits 53, and one of those numbers, 47, is prime.
But the pair of mathematicians soon realized that this potential explanation couldn't account for the magnitude of the biases they found. Nor could it explain why, as the pair found, primes ending in 3 seem to like being followed by primes ending in 9 more than 1 or 7. To explain these and other preferences, Lemke Oliver and Soundararajan had to delve into the deepest model mathematicians have for random behavior in the primes.
Random Primes
Prime numbers, of course, are not really random at all — they are completely determined. Yet in many respects, they seem to behave like a list of random numbers, governed by just one overarching rule: The approximate density of primes near any number is inversely proportional to how many digits the number has.
In 1936, Swedish mathematician Harald Cramér explored this idea using an elementary model for generating random prime-like numbers: At every whole number, flip a weighted coin — weighted by the prime density near that number — to decide whether to include that number in your list of random "primes." Cramér showed that this coin-tossing model does an excellent job of predicting certain features of the real primes, such as how many to expect between two consecutive perfect squares.
Despite its predictive power, Cramér's model is a vast oversimplification. For instance, even numbers have as good a chance of being chosen as odd numbers, whereas real primes are never even, apart from the number 2. Over the years, mathematicians have developed refinements of Cramér's model that, for instance, bar even numbers and numbers divisible by 3, 5, and other small primes.
These simple coin-tossing models tend to be very useful rules of thumb about how prime numbers behave. They accurately predict, among other things, that prime numbers shouldn't care what their final digit is — and indeed, primes ending in 1, 3, 7 and 9 occur with roughly equal frequency.
Yet similar logic seems to suggest that primes shouldn't care what digit the prime after them ends in. It was probably mathematicians' overreliance on the simple coin-tossing heuristics that made them miss the biases in consecutive primes for so long, Granville said. "It's easy to take too much for granted — to assume that your first guess is true."
The primes' preferences about the final digits of the primes that follow them can be explained, Soundararajan and Lemke Oliver found, using a much more refined model of randomness in primes, something called the prime k-tuples conjecture. Originally stated by mathematicians G. H. Hardy and J. E. Littlewood in 1923, the conjecture provides precise estimates of how often every possible constellation of primes with a given spacing pattern will appear. A wealth of numerical evidence supports the conjecture, but so far a proof has eluded mathematicians.
The prime k-tuples conjecture subsumes many of the most central open problems in prime numbers, such as the twin primes conjecture, which posits that there are infinitely many pairs of primes — such as 17 and 19 — that are only two apart. Most mathematicians believe the twin primes conjecture not so much because they keep finding more twin primes, Maynard said, but because the number of twin primes they've found fits so neatly with what the prime k-tuples conjecture predicts.
In a similar way, Soundararajan and Lemke Oliver have found that the biases they uncovered in consecutive primes come very close to what the prime k-tuples conjecture predicts. In other words, the most sophisticated conjecture mathematicians have about randomness in primes forces the primes to display strong biases. "I have to rethink how I teach my class in analytic number theory now," Ono said.
At this early stage, mathematicians say, it's hard to know whether these biases are isolated peculiarities, or whether they have deep connections to other mathematical structures in the primes or elsewhere. Ono predicts, however, that mathematicians will immediately start looking for similar biases in related contexts, such as prime polynomials — fundamental objects in number theory that can't be factored into simpler polynomials.
And the finding will make mathematicians look at the primes themselves with fresh eyes, Granville said. "You could wonder, what else have we missed about the primes?"
Academics Make Theoretical Breakthrough in Random Number Generation (https://threatpost.com/academics-make-theoretical-breakthrough-in-random-number-generation/118150/)
QuoteTwo University of Texas academics have made what some experts believe is a breakthrough in random number generation that could have longstanding implications for cryptography and computer security. David Zuckerman, a computer science professor, and Eshan Chattopadhyay, a graduate student, published a paper in March that will be presented in June at the Symposium on Theory of Computing. The paper describes how the academics devised a method for the generation of high quality random numbers. The work is theoretical, but Zuckerman said down the road it could lead to a number of practical advances in cryptography, scientific polling, and the study of other complex environments such as the climate.
"We show that if you have two low-quality random sources—lower quality sources are much easier to come by—two sources that are independent and have no correlations between them, you can combine them in a way to produce a high-quality random number," Zuckerman said. "People have been trying to do this for quite some time. Previous methods required the low-quality sources to be not that low, but more moderately high quality. "We improved it dramatically," Zuckerman said. The technical details are described in the academics' paper "Explicit Two-Source Extractors and Resilient Functions." The academics' introduction of resilient functions into their new algorithm built on numerous previous works to arrive at landmark moment in theoretical computer science. Already, one other leading designer of randomness extractors, Xin Li, has built on their work to create sequences of many more random numbers. "You expect to see advances in steps, usually several intermediate phases," Zuckerman said. "We sort of made several advances at once. That's why people are excited." In fact, academics worldwide have taken notice. Oded Goldreich, a professor of computer science at the Weizmann Institute of Science in Israel, called it a fantastic result. "It would have been great to see any explicit two-source extractor for min-entropy rate below one half, let alone one that beats Bourgain's rate of 0.499," Goldreich said on the Weizmann website. "Handling any constant min-entropy rate would have been a feast (see A Challenge from the mid-1980s), and going beyond that would have justified a night-long party." MIT's Henry Yuen, a MIT PhD student in theoretical computer science, called the paper "pulse-quickening." "If the result is correct, then it really is — shall I say it — a breakthrough in theoretical computer science," Yuen said. The study of existing random number generators used in commercial applications has intensified since the Snowden documents were published; sometimes random numbers aren't so random. Low quality random numbers are much easier to predict, and if they're used, they lower the integrity of the security and cryptography protecting data, for example. Right now, Zuckerman's and Chattopadhyay's result is theoretical and work remains in lowering the margins of error, Zuckerman said. Previous work on randomness extractors, including advances made by Zuckerman, required that one sequence used by the algorithm be truly random, or that both sources be close to random. The academics' latest work hurdles those restrictions allowing the use of sequences that are only weakly random. Their method requires fewer computational resources and results in higher quality randomness. Today's random number systems, for example, are fast, but are much more ad-hoc. "This is a problem I've come back to over and over again for more than 20 years," says Zuckerman. "I'm thrilled to have solved it."
i need help.
imam tri nezavisne grupe u eksperimentu (6 tačaka po grupi) a rezultati mi ne ukazuju na normalnu distribuciju, što bi valjda značilo da treba da koristim neki neparametrijski test za poređenje? pitanje: da li je kruskall wallis OK?
a da za post-poređenje između grupa koristim dunn's multiple comparisons test?
A da možda nađeš nekoga pouzdanijeg od Sagite da to pitaš? :lol:
Mada me teši da i naučnici evropskog ranga imaju poteškoća sa statistikom. <3
Ako ti nije kasno, mogu da pitam sutra moje matematičare u Institutu...
Quote from: Father Jape on 22-05-2016, 10:32:38
A da možda nađeš nekoga pouzdanijeg od Sagite da to pitaš? :lol:
Mada me teši da i naučnici evropskog ranga imaju poteškoća sa statistikom. <3
ma pitala sam al je rano :lol:
mislim, 99.99% sam sigurna da može ovako kako sam pitala al konsultacije su uvek bolje nego one man show :)
Quote from: Mica Milovanovic on 22-05-2016, 10:40:45
Ako ti nije kasno, mogu da pitam sutra moje matematičare u Institutu...
nije kasno. hvala. ako te ne mrzi preterano :)
Nije mi teško. Pošalji mi na privatni email detalje o problemu. Ovo nije dovoljno da bih pitao.
Quote from: lilit on 22-05-2016, 09:47:01
i need help.
imam tri nezavisne grupe u eksperimentu (6 tačaka po grupi) a rezultati mi ne ukazuju na normalnu distribuciju, što bi valjda značilo da treba da koristim neki neparametrijski test za poređenje? pitanje: da li je kruskall wallis OK?
a da za post-poređenje između grupa koristim dunn's multiple comparisons test?
odma da se ogradim i kažem da mi je poznavanje statistike vrlo vrlo ograničeno.
ako se dobro sećam, prema ovome kako si opisala problem (a što reče Mića, ne bi bilo zgoreg da pošalješ više detalja), trebalo bi da može. Mann-Whitney U je valjda za poređenje dve grupe (kada nema normalne raspodele), a Kruskall-Wallis za višestruko poređenje, odnosno tri ili više grupa.
kad sam ja nešto prčkao oko toga, naleteo sam da Kruskall-Wallis takođe zahteva proveru homogenosti varijanse, samo se koristi neparametarska verzija Levenovog testa, gde se provera homogenost rangova ili tako nešto, ne mogu tačno da se setim.
Computer generated math proof is largest ever at 200 terabytes (http://phys.org/news/2016-05-math-proof-largest-terabytes.html)
Quote(Phys.org)—A trio of researchers has solved a single math problem by using a supercomputer to grind through over a trillion color combination possibilities, and in the process has generated the largest math proof ever—the text of it is 200 terabytes in size. In their paper uploaded to the preprint server arXiv, Marijn Heule with the University of Texas, Oliver Kullmann with Swansea University and Victor Marek with the University of Kentucky outline the math problem, the means by which a supercomputer was programmed to solve it, and the answer which the proof was asked to provide.
The math problem has been named the boolean Pythagorean Triples problem and was first proposed back in the 1980's by mathematician Ronald Graham. In looking at the Pythagorean formula: a2 + b2 = c2, he asked, was it possible to label each a non-negative integer, either blue or red, such that no set of integers a, b and c were all the same color. He offered a reward of $100 to anyone who could solve the problem.
To solve this problem the researchers applied the Cube-and-Conquer paradigm, which is a hybrid of the SAT method for hard problems. It uses both look-ahead techniques and CDCL solvers. They also did some of the math on their own ahead of giving it over to the computer, by using several techniques to pare down the number of choices the supercomputer would have to check, down to just one trillion (from 102,300). Still the 800 processor supercomputer ran for two days to crunch its way through to a solution. After all its work, and spitting out the huge data file, the computer proof showed that yes, it was possible to color the integers in multiple allowable ways—but only up to 7,824—after that point, the answer became no.
While technically, the team, along with their computer did create a proof for the problem, questions remain, the first of which is, is the proof really a proof if it does not answer why there is a cut-off point at 7,825, or even why the first stretch is possible? Strictly speaking, it is, the team used another computer program to verify the results, and the proof did give a definitive answer to the original question—which caused Graham to make good on his offer by handing over the $100 to the research team—but, nobody can read the proof (or other similar but smaller proofs also generated by computers but which are still too large for a human to read) which begs the philosophical question, does it really exist?
Explore further: Computer generated math proof is too large for humans to check
More information: Solving and Verifying the boolean Pythagorean Triples problem via Cube-and-Conquer, arXiv:1605.00723 [cs.DM] arxiv.org/abs/1605.00723
Abstract
The boolean Pythagorean Triples problem has been a longstanding open problem in Ramsey Theory: Can the set N = {1,2,...} of natural numbers be divided into two parts, such that no part contains a triple (a,b,c) with a2+b2=c2 ? A prize for the solution was offered by Ronald Graham over two decades ago.
We solve this problem, proving in fact the impossibility, by using the Cube-and-Conquer paradigm, a hybrid SAT method for hard problems, employing both look-ahead and CDCL solvers. An important role is played by dedicated look-ahead heuristics, which indeed allowed to solve the problem on a cluster with 800 cores in about 2 days.
Due to the general interest in this mathematical problem, our result requires a formal proof. Exploiting recent progress in unsatisfiability proofs of SAT solvers, we produced and verified a proof in the DRAT format, which is almost 200 terabytes in size. From this we extracted and made available a compressed certificate of 68 gigabytes, that allows anyone to reconstruct the DRAT proof for checking.
via Nature
Read more at: http://phys.org/news/2016-05-math-proof-largest-terabytes.html#jCp (http://phys.org/news/2016-05-math-proof-largest-terabytes.html#jCp)
Mathematicians finally starting to understand epic ABC proof (https://www.newscientist.com/article/2099534-mathematicians-finally-starting-to-understand-epic-abc-proof/)
Quote
It has taken nearly four years, but mathematicians are finally starting to comprehend a mammoth proof that could revolutionise our understanding of the deep nature of numbers.
The 500-page proof was published online by Shinichi Mochizuki (http://www.kurims.kyoto-u.ac.jp/~motizuki/top-english.html) of Kyoto University, Japan in 2012 (https://www.newscientist.com/article/dn22256-fiendish-abc-proof-heralds-new-mathematical-universe/) and offers a solution to a longstanding problem known as the ABC conjecture, which explores the fundamental relationships between numbers, addition and multiplication beginning with the simple equation a + b = c.
Mathematicians were excited by the proof but struggled to get to grips (https://www.newscientist.com/article/dn26753-mathematicians-anger-over-his-unread-500-page-proof/) with Mochizuki's "Inter-universal Teichmüller Theory" (IUT), an entirely new realm of mathematics he had developed over decades in order to solve the problem. A meeting held last year at the University of Oxford, UK with the aim of studying IUT ended in failure (https://www.newscientist.com/article/dn28682-mathematicians-left-baffled-after-three-year-struggle-over-proof/), in part because Mochizuki doesn't want to streamline his work to make it easier to comprehend, and because of a culture clash between Japanese and western ways of studying mathematics.
Now a second meeting, held last month at his home ground in Kyoto (https://www.maths.nottingham.ac.uk/personal/ibf/files/kyoto.iut.html), has proved more successful. "It definitely went better than expected," says Ivan Fesenko (https://www.maths.nottingham.ac.uk/personal/ibf/) of the University of Nottingham, UK, who helped organise the meeting.
The breakthrough seems to have come from Mochizuki explaining his theory in person. He refuses to travel abroad, only speaking via Skype at the Oxford meeting, which had made it harder for mathematicians outside Japan to get to grips with his work. "It was the key part of the meeting," says Fesenko. "He was climbing the summit of his theory, and pulling other participants with him, holding their hands."
Glimmer of understanding At least 10 people now understand the theory in detail, says Fesenko, and the IUT papers have almost passed peer review so should be officially published in a journal in the next year or so. That will likely change the attitude of people who have previously been hostile towards Mochizuki's work, says Fesenko. "Mathematicians are very conservative people, and they follow the traditions. When papers are published, that's it."
"There are definitely people who understand various crucial parts of the IUT," says Jeffrey Lagarias (http://www.math.lsa.umich.edu/~lagarias/) of the University of Michigan, who attended the Kyoto meeting, but was not able to absorb the entire theory in one go. "More people outside Japan have incentive to work to understand IUT as it is presented, all 500 pages of it, making use of new materials at the various conferences."
But many are still not willing devote the time Mochizuki demands to understand his work. "The experts are still on the fence," says Lagarias. "They are waiting for someone else to read the proof and asking why it cannot be made easier to understand."
It is likely that the IUT papers will be published in a Japanese journal, says Fesenko, as Mochizuki's previous work has been. That may affect its reception by the wider community. "Certainly which journal they are published in will have something to do with how the math community reacts," says Lagarias.
The glimmer of understanding that has started to emerge is well worth the effort, says Fesenko. "I expect that at least 100 of the most important open problems in number theory will be solved using Mochizuki's theory and further development."
But it will likely be many decades before the full impact of Mochizuki's work on number theory can be felt. "The magnitude of the number of new structures and ideas in IUT will take years for the math community to absorb," says Lagarias.
(https://www.znaksagite.com/diskusije/proxy.php?request=http%3A%2F%2Fantenam.net%2Fmedia%2Fk2%2Fitems%2Fcache%2Fb4f8aa0e64f234ba028215bbcd334cb6_L.jpg&hash=f48b7aadfc9f11dcfed52b5bb95692596afefb4b)
This ancient Babylonian tablet may contain the first evidence of trigonometry (http://www.sciencemag.org/news/2017/08/ancient-babylonian-tablet-may-contain-first-evidence-trigonometry)
Mathematicians Race to Debunk German Man Who Claimed to Solve One of the Most Important Computer Science Questions of Our Time (https://motherboard.vice.com/en_us/article/evvp34/p-vs-np-alleged-solution-nortbert-blum)
Quote from: Meho Krljic on 01-09-2017, 09:06:19
Mathematicians Race to Debunk German Man Who Claimed to Solve One of the Most Important Computer Science Questions of Our Time (https://motherboard.vice.com/en_us/article/evvp34/p-vs-np-alleged-solution-nortbert-blum)
Comments: (https://arxiv.org/abs/1708.03486v2) | The proof is wrong. I shall elaborate precisely what the mistake is. For doing this, I need some time. I shall put the explanation on my homepage (https://arxiv.org/abs/1708.03486v2) |
https://courses.edx.org/courses/course-v1:Microsoft+DEV262x+1T2018/progress
Топло препоручујем овај курс из логике и рачунарског размишљања.
У којој се школи уче Диофантове квадратне једначине?
https://www.nytimes.com/crosswords/game/set
Мало занимације за сваки дан, није лоше да се вежба...
Jednom kad skapiraš sistem pretrage malo izgubi draž.
Мислим да сам га већ скапирао. Данас сам овај први проблем решио за једно два минута. Ал зато крећем од сутра да вежбам напредније проблеме. Него прочитах на блогу једне програмерке да она свако јутро вежба вијуге тако па рек'о да пробам.
Bugari napravili par sajtova sa dovoljno glavolomki da ti traju ceo dan. Svaka glavolomka ima posebnu adresu, što mi je neobično, ali možda ima veze sa većom vidljivošću s gugla. Najlepše je što povremeno dodaju nove tipove glavolomki. Trude se ljudi svojski. Ako vam neki problem bude pretežak dajte ovde tip problema i identifikator problema, pa možemo zajedno da se mučimo. Evo adrese jedne od glavolomki, a do ostalih možete da dođete sa ove stranice:
https://www.puzzle-bridges.com/
Walter Bradley Center Fellow Discovers Longstanding Flaw in an Aspect of Elementary Calculus (https://mindmatters.ai/2019/04/walter-bradley-center-fellow-discovers-longstanding-flaw-in-elementary-calculus/)
QuoteFor those interested in the technical details, the second derivative of y with respect to x has traditionally had the notation "d2 y/dx 2". While this notation is expressed as a fraction, the problem is that it doesn't actually work as a fraction. The problem is well-known but it has been generally assumed that there is no way to express the second derivative in fraction form. It has been thought that differentials (the fundamental "dy" and "dx" that calculus works with) were not actual values and therefore they aren't actually in ratio with each other. Because of these underlying assumptions, the fact that you could not treat the second derivative as a fraction was not thought to be an anomaly. However, it turns out that, with minor modifications to the notation, the terms of the second derivative (and higher derivatives) can indeed be manipulated as an algebraic fraction. The revised notation for the second derivative is "(d 2 y/dx 2) – (dy/dx)(d 2 x/dx 2)".
Rešenje je ovde gore od problema, rekli bi cinici :lol: :lol: :lol: :lol:
Ali opet, pošto ima mnogo ludaka koji samo čekaju da pokažu prstom da "matematika nije tačna" i obruše se na nauku kao na izmišljotinu novog sveckog poretka, masona, Jevreja, feministkinja i pedera, onda je dobro da se ovakve stvari rešavaju.
Што каже Веља Абрамовић, сва та описна математика не вреди ништа!
Šta je opisna matematika i kako izvodi spadaju u nju?
Описна је оно што описује неку математичку форму. Нпр. Интеграли описују површ графикона ако се добро сећам. И то су људи измислили
Веља сматра да је права математика она генеричка, тј. она која одражава законе природе попут златног пресека.
Pošto sam ja izrazito slab s matmatikom, priznajem da ne razumem tu distinkciju. Šta bi bio izraz za zapreminu lopte? On opisuje geometrijsko telo ali to telo je sveprisutno u prirodi.
И Веља у својој физици бави запремином лопте ( https://www.youtube.com/watch?v=PES6G9a2UZE ) тако да верујем да је то генеричка математика. Ја сам рецимо изразито слаб у физици тако да не могу да проценим колико су његови прорачуни тачни. Веља тврди да су интеграли, диференцијали, статистика описна математика нижег реда која нема везе са природним законима.
kako onda gomila prirodnih zakona opisana izvodima i integralima?
Питај Вељу!!
До формуле (1) на 219 страни коју аутори зову екстремно неинтуитивном, долази се простом заменом променљивих
(https://www.znaksagite.com/diskusije/proxy.php?request=http%3A%2F%2Foi67.tinypic.com%2F20h6mmu.jpg&hash=321ab48d07c2976d5d4cb3305985565d1c3d7650)
Вреди приметити замену диференцијала у оператору диференцирања.
Немој Васо, подсећа ме на Теорије цена! :cry:
радим неки курс из дата сајнс па би ми користила помоћ. Имате две четворостране коцке ( личи највише на пирамиду ). Задатак је следећи:
Assume we roll 2 four sided dice. What is P({first roll larger than second roll})? Answer in reduced fraction form - eg 1/5 instead of 2/10.P је вероватноћа изражена у разломку.
Napravi matricu gde je jedna strana rezultat jedne kocke, a druga rezultat druge. Ostatak rešenja sam izbelio:
Ćelije matrice su verovatnoće da se desi taj rezultat, i trebalo bi da svaka ćelija ima verovatnoću 1/16. Slučajevi kada je rezultat jedne kocke veći od rezultata druge su svi slučajevi iznad (ili ispod, svejedno) glavne dijagonale. Tih slučajeva ima 6, pa je ukupna verovatnoća 6/16=0.375
хехе, браво мек! Зна се који главни форумски мозак. Иначе сам мало варао, тј. у међувремену сам нашао одговор на овом сајту:
https://stackexchange.com/ (https://stackexchange.com/)Овај сајт му дође као мајка stackoverflow. Регистровао сам се истим налогом који имам на stackoverflow.
мек, твој супер мозак ће ми опет требати!
Given two decks of 52 playing cards, you flip one over from each deck. Assuming fair decks, what is P({the two cards are the same suit})?
решење је 1/4 али га не капирам. То је шанса да у једном шпилу извучеш једну боју. А за два шпила да је иста зар не би требало да је вероватноћа 1/16?
Verovatnoća da dve karte budu pik je 1/16, a toliko je i da budu herc, tref, i karo. Kad sabereš ovo dobiješ četvrtinu.
Видиш, ово је једини задатак који нисам умео да решим. Капирам да је то зато што делује контраинтуитивно. И даље ми делује након твог објашњења.
Može i jednostavnije do istog rešenja: verovatnoća da druga karta bude neka od četiri boje je 1/4. A to je upravo i verovatnoća da se složi s bojom prve karte, pošto je ona u nekoj od četiri boje. Trivijalno. :)
Тривијално, али треба наштеловати мозак да размишља на тај начин.
Da nisi rekao rešenje ja bih opet krenuo od tabele. Tako je manja šansa da pogrešiš, ali se malo više pomučiš. I to bih možda krenuo od tabele 54x54, za svaki slučaj. Redovi su ishod prvog izvlačenja a kolone ishod drugog. Ćelije su verovatnoće da se taj ishod desi. Pošto su izvlačenja međusobno nezavisna u svakoj ćeliji je ista vrednost, proizvod verovatnoća prvog i drugog, 1/(52x52). U zbiru daju 1, kao što i treba da bude. Nas interesuju ishodi sa istim bojama, a to su 4 regije u tabeli veličina 13x13. Zbir njihovih verovatnoća je (4x13x13)x(1/(52x52))=1/4.
хехе, ипак ми је простије оно претходно решење. Само да не мора да се узима оловка у руке. :)
Mda, lako je sad kad su događaji nezavisni, jer se karte vuku iz različitih špilova. Videću te kod povezanih događaja, recimo karte iz istog špila.
Било је до сада пар задата и са тим. Само умањим број карата у другом извлачењу у разломку, уместо 52, 51. Ал могуће да тежи задаци тек предстоје!
Мек, опет сам забаговао. Један проблем никако да решим.
You pick three balls in succession out of a bucket of 3 red balls and 3 green balls. Assume replacement after picking out each ball. What is the probability of each of the following events?
Any sequence with 2 reds and 1 green. Answer in reduced fraction form - eg 1/5 instead of 2/10.
Не, одговор није 1/8.
1/6
Ovo "assume replacement" interpretiram kao da loptu vraćamo u korpu kad je odaberemo, i svaki izbor ima iste verovatnoće kao i svaki prethodni. Biranje jedne lopte ima dva ishoda, a pošto je sve nezavisno biranje tri ima osam, pa svaki pojedinačni ishod ima verovatnoću 1/8. Od tih osam ishoda tri imaju dve crvene i jednu zelenu, pa je verovatnoća 3/8.
мек, кад ти објашњаваш то изгледа тако просто!!
Мек, опет ми је потребан твој бриљантан ум
Let's say Alvin will catch the flu with probability of 1/10 during any given month. Let's also assume that Alvin can catch the flu only once per month, and that if he has caught the flu, the flu virus will die by the end of the month. What is the probability of the following events?
He catches the flu exactly once in the three months from September through November.
Pošto je nazebavanje nezavisno svakog meseca onda možemo da gledamo samo tri tražena meseca, a ignorišemo bilo šta što se dešavalo ranije. Postoji osam elementarnih ishoda (dobio ili nije dobio grip svakog od tri meseca, 2^3=8), ali ovi ishodi nemaju istu verovatnoću. Svaki ishod je rezultat tri nezavisna događaja, pa je verovatnoća bilo kog ishoda jednaka proizvodu verovatnoća odgovarajuća tri događaja. Nama su interesantna tri od ovih osam ishoda (slučajevi 100, 010, 001, ako me razumeš).
Neka je P(m) verovatnoća da Alvin dobije grip u mesecu m∈[1,12]. Za svako m verovatnoća je P(m)=1/10, a verovatnoća da se ne desi P(m) je 1-P(m)=9/10. Verovatnoća bilo kog od tri interesantna ishoda je P*(1-P)*(1-P)=81/1000 (verovatnoća da jednog meseca dobije grip, ali da ga u druga tri meseca ne dobije). Verovatnoća da se desi bilo koji od ova tri ishoda je njihov zbir, 243/1000.
мек, хвала. Ја сам стигао до 81/1000 и ту стао. А у ствари треба сабрати вероватноће за сва три случаја. кад ти објасниш то делује тако просто ( мада ово сам ти већ написао ).
Decades-Old Computer Science Conjecture Solved in Two Pages (https://www.quantamagazine.org/mathematician-solves-computer-science-conjecture-in-two-pages-20190725/)
Two Mathematicians Just Solved a Decades-Old Math Riddle — and Possibly the Meaning of Life (https://www.livescience.com/diophantine-42-solved-meaning-of-life.html)
Quote from: Meho Krljic on 14-09-2019, 07:45:18
Two Mathematicians Just Solved a Decades-Old Math Riddle — and Possibly the Meaning of Life (https://www.livescience.com/diophantine-42-solved-meaning-of-life.html)
Није да ће ово баш да открије смисао живота, али је макар забавно.
Dobro, to je aluzija na Daglasa Adamsa, jelte :lol:
This Guy Just Found a Faster Way to Multiply (https://www.popularmechanics.com/science/a29514208/faster-way-multiply/)
Mathematician Finds Easier Way to Solve Quadratic Equations (https://www.popularmechanics.com/science/math/a30152083/solve-quadratic-equations/)
Mathematician Proves Huge Result on 'Dangerous' Problem (https://www.quantamagazine.org/mathematician-terence-tao-and-the-collatz-conjecture-20191211/)
Quote
The Collatz conjecture is quite possibly the simplest unsolved problem in mathematics — which is exactly what makes it so treacherously alluring.
"This is a really dangerous problem. People become obsessed with it and it really is impossible," said Jeffrey Lagarias (http://www.math.lsa.umich.edu/~lagarias/), a mathematician at the University of Michigan and an expert on the Collatz conjecture.
(...)
Lothar Collatz likely posed the eponymous conjecture in the 1930s. The problem sounds like a party trick. Pick a number, any number. If it's odd, multiply it by 3 and add 1. If it's even, divide it by 2. Now you have a new number. Apply the same rules to the new number. The conjecture is about what happens as you keep repeating the process.
Intuition might suggest that the number you start with affects the number you end up with. Maybe some numbers eventually spiral all the way down to 1. Maybe others go marching off to infinity.
But Collatz predicted that's not the case. He conjectured that if you start with a positive whole number and run this process long enough, all starting values will lead to 1. And once you hit 1, the rules of the Collatz conjecture confine you to a loop: 1, 4, 2, 1, 4, 2, 1, on and on forever.
Over the years, many problem solvers have been drawn to the beguiling simplicity of the Collatz conjecture, or the "3x + 1 problem," as it's also known. Mathematicians have tested quintillions of examples (that's 18 zeros) without finding a single exception to Collatz's prediction. You can even try a few examples yourself with any of the many "Collatz calculators (https://www.dcode.fr/collatz-conjecture)" online. The internet is awash in unfounded amateur proofs that claim to have resolved the problem one way or the other.
xrotaeye xrotaeye xrotaeye
Researchers Solve One of the Most Notorious Open Problems in Math (https://www.popularmechanics.com/science/a31977621/math-problem-random-walks-solved/)
(https://uploads.tapatalk-cdn.com/20200724/e0b72c65a43b8d6bd56fc0e2490f71af.jpg)
Fali točka.
Točno!
Pa, da, mada, da bi bilo jasnije, trebalo je da 5! stavi pod navodnike. Ali dobro, zgodna dosetka.
Ne fali ništa. Faktorijel 5.
Fali tačka na kraju rečenice "You probably won't believe it but the answer is 5!", da bi rečenica bila gramatički ispravna. A fali i zarez u sredini.
Treba me jebati što se zajebem sa budalama.
A Mathematician Has Finally Solved the Infamous Goat Problem (https://www.popularmechanics.com/science/math/a34962164/mathematician-solves-goat-problem/)
Dobar primer naučnog rada koji, za sada, nema nikakvu praktičnu aplikaciju :lol: :lol:
Mathematicians Discovered a New Kind of Prime Number (https://www.popularmechanics.com/science/math/a36014795/mathematicians-discover-new-kind-of-prime-number/)
Dobar fraktalni trip od dva i po sata, s odgovarajućom muzikom
https://youtu.be/LhOSM6uCWxk
Dobar (al podugačak) video koji komplikovanu logičko-matematičku problematiku paradoksa približi običnom svetu kroz - jezik:
https://youtu.be/ymGt7I4Yn3k (https://youtu.be/ymGt7I4Yn3k)
Zabavan je Džefri Kaplan.
A geometric shape that does not repeat itself when tiled (https://phys.org/news/2023-03-geometric-tiled.html)
https://twitter.com/edfrenkel/status/1746704468153143321
Mda, poštujemo pravila koja se odnosena zagrade, a ignorišemo pravila koja se odnose na priotitet operacija. Sve operacije moraju imati jednak prioritet, school be damned! Čovek je ili komunista, ili feminista. Ili neoliberal.
Ovi tvrde da množenje nema prioritet. Zato i jeste dilema...
Ne, ovaj lik tvrdi da pravila kojima nas uče u školi "mogu da bidnu, al' ne moraju da budu". U školi nas uče da množenje i deljenje imaju isti prioritet i da se primenjuju s leva na desno. Ali njemu se to nešto ne sviđa. On bi to drugačije, jer mu se to nešto ne sviđa.
Quote
I imagine a school teacher telling students what those rules are, and then testing whether the students memorized those rules. This is NOT the right way of teaching math, in my opinion.
Bukvalno nema veze kojim ćeš redosledom deliti ili množiti i oduzimati ili dodavati, rezultat će uvek biti isti jer su to dve međusobno potpuno inverzne operacije. To pravilo sa levo na desno je budalaština. Verovatno čovek to i pokušava da objasni.
Uradi desno na levo i imaćeš isti rezultat. Uradi random redosledom, ni levo ni desno, i imaćeš isti rezultat.
Ma nemoj.
S leva na desno: 8 / 4 * 2 = (8 / 4) * 2 = 2 * 2 = 4
S desna na levo: 8 / 4 * 2 = 8 / (4 * 2) = 8 / 8 = 1
Da, ali bitno je šta množiš / deliš.
S leva na desno: 8 / 4 * 2 = (8 / 4) * 2 = = 2 * 2 = 4
S desna na levo: 8 / 4 * 2 = (8 * 2) / 4 = 16 / 4 = 4
Dude... Ti mi daš tvrdnju. Ja ti dam kontra-primer koji pobija tvoju tvrdnju. Ti mi daš primer koji potvrđuje tvoju tvrdnju. Ali šta ćemo sa mojim kontra-primerom? Dovoljan je jedan kontra-primer da bi tvrdnja bila pobijena. Pobijen si, batice...
Može i ovako: Podeliš množitelj i delitelj najmanjim zajedničkim sadržateljom 2. To se naziva skraćivanje razlomka.
8 / 4 * 2 = 8 / (4 / 2) * (2 / 2) = 8 / 2 * 1 = 8 / 2 = 4
Tvoj primer nije primer jer radiš nepostojeću operaciju između množitelja i delitelja prvo, a to je ono što ne može da se radi. Izvršavaš operacije na brojem 8, a ne između množitelja i delitelja.
Napiši svoj primer kao razlomak i videćeš da nikakve dileme tu nema, vrlo je jednostavno. Množitelj je iznad razlomačke crte, a delitelj ispod. Da li ćeš pomnožiti, ili podeliti prvo, apsolutno je nebitno.
Ali lik sa tvitera kaže da se može. Sve se može kad nema pravila. Dosta sam vam vremena potrošio. Idite radite nešto.
To su pravila za decu koja nisu stigla do razlomaka, gde se rade naprednije operacije.
8 / 4 * 2 = 2 * 8 / 4 (transformacija izraza po pravilima sređivanja razlomaka) = 16 / 4 = 4
Pravilo je da množitelji uvek idu iznad razlomačke crte, a delitelji ispod. Tvoj primer je netačan, jer si množitelj gurnuo ispod razlomačke crte. Rezultat je uvek samo 4.
Help me settle an argument. What is 8 divided by 2 times 4? (https://youtu.be/py2lzZoeepM)
Da, viđao sam ovakve linkove ranije.
To je neko nešto pobrkao. To pravilo prioriteta množenja u odnosu na deljenje ima smisla samo kod rešavanja pravilno napisanog razlomka.
Npr. 8 / 4 * 2 ako je razlomak gde je 8 brojilac, a 4 * 2 su deo imenilac, znači ispod razlomačke crte, onda ima smisla prvo pomnožiti 8 / 8 pa deliti i dobije se 1.
Ali, generalno ovako napisano 8 / 4 * 2 deluje da imamo razlomak gde je 8 brojilac, a imenilac 4 i taj razlomak se naknadno množi sa 2, dakle množi brojilac razlomka 2 * 8 / 4 i dobije 16 /4 tj 4.
Neko je nešto teško pobrkao. Moguće da se radi o lošem nastavnom kadru u nižim razredima. Koji je u nekim slučajevima i izdavao udžbenike sa pogrešnim pravilom prioriteta množenja nad deljenjem. I eto konfuzije.
Quote from: mac on 16-01-2024, 14:21:08
Help me settle an argument. What is 8 divided by 2 times 4? (https://youtu.be/py2lzZoeepM)
Množenje nema prioritet u odnosu na dijeljenje, kako su mene učili, ajd i nekako.
Ali zašto lijevo ima prioritet u odnosu na desno!?!!? 8-)
Neoliberali!
Ljudi raspravljaju u prioritetima. Da li su operacije množenja i deljenja istog ili različitog prioriteta. To o čemu ti pričaš nije ono o čemu drugi ljudi raspravljaju. Uostalom evo drugog primera:
1024 / 32 / 4 / 2
Koja je sad tu tvoja interpretacija? Jer mene su učili u školi da se ovo računa s leva na desno, dakle ((1024 / 32) / 4) / 2 = 4. Ti pričaš da to liči na razlomke, pa sad ispada da po tebi to treba da se računa (1024 / 32) / (4 / 2) = 16
Ne, ja baš pričam da nisu razlomci, a da ljudi koriste pravila koja se koriste u radu sa razlomcima, tj. prioritet množenja u odnosu na deljenje kad rešavaš razlomak, jer su pobrkali osnovno i napredno.
Jeste, 16 bi bio rezultat da se radi o razlomku, a ne radi se. Ipak, može se prevesti u razlomak, tj. 1024 / 32 / 4 / 2 = 1024 / 32 * 4 * 2 (kad sekvencijalno deliš, onda se umnožava imenilac) = 1024 / 256
Kako prevodiš svaki izraz u razlomak? Jednostavno, prvo što ti dođe pre razlomačke crte je brojilac, ono što dođe posle je imenilac. Svako sekvencijano moženje posle toga množi samo brojilac, a svako sekvencijalno deljenje množi imenilac time sa čim deliš. Prosto kao pasulj.
Prevedeš izraz u razlomak i rešavaš bez tog pravila s leva na desno, jer je to samo olakšanje da deca koja ne znaju razlomke reše elementarne izraze. Kad imaš razlomak, svejedno ti je da li ga rešavaš iz sredine, levo ili desno.
Ljudi su jednostavno matematiku zaboravili ili poluzaboravili, pa sad imaš ovakve konfuzije po internetu, kad se poluprisećaju stvari.
Bukvalno nikakve konfuzije nema. Ako hoćeš te proste izraze da, radi sa leva na desno i bez prioriteta množenja u odnosu na deljenje i dobićeš tačno rešenje.
Ipak, razlomci su bolji vizuelno i omogućavaju napredne tehnike poput uprošćavanja izraza, deljenja sa najmanjim zajedničkim sadržateljem i sl.
https://mondo.rs/Magazin/Stil/a1877690/Srpski-profesori-objasnili-kako-se-racuna-zadatak-iz-matematike.html
Besmrtno:
(https://i.imgur.com/3YOpAVu.jpeg)
Viđao sam ovo, al šta ćemo recimo sa 74% od 2569?
Je l može 2569% od 74?
Ma može, i sve to lepo radi, al recimo u ovom slučaju treba digitron i za jedno i za drugo.
Naravno. Ali to da Timoti daje ovaj odgovor a onda dobija, jelte, onakvu reakciju, to je ovde besmrtno.
To neki moj imenjak Truman ima afekciju na mladog i velikog glumca. :|
A New Formula for Pi Is Here. And It's Pushing Scientific Boundaries. (https://www.popularmechanics.com/science/math/a66026240/new-quantum-pi-formula-revolutionizes-mathematics/)
QuotePhysicists are now using principles from quantum mechanics to build a new model of the abstract concept of pi. Or, more accurately, they built a new model that happens to include a great new representation of pi. But what does that mean, and why do we need different representations of pi?
Because quantum mechanics looks at the tiniest particles, one at a time, even simple questions can have complex answers that require massive computing power. Rendering high-tech video games and movies like Avatar can take days or more, and that's still not at the level of reality. In this paper, published in the peer-reviewed journal Physical Review Letters, physicists Arnab Priya Saha and Aninda Sinha describe their new version of a quantum model that reduces complexity, but maintains accuracy.